You are on page 1of 78

Laws of Exponents:

1. a m a n = a m +n QUADRATIC EQUATION
m
a Generally, an equation is said to be of quadratic form if it has the form
2. n
= a m-n
a ax2n + bxn + c = 0, where n is an integer or a fraction; such as x4 – 5x2 + 6 = 0 and
3. (a )m n
= a mn
y-3 + y-3/2 + 6 = 0
− b ± b 2 − 4ac
4. ( ab ) n = a nb n Quadratic Formula: X=
n 2a
a an
5.   = n
b  b The expression b2 – 4ac is called the discriminant

Laws of Radicals: 1. When b2 – 4ac > 0, the roots are real and unequal.
1. n a n = a 2. When b2 – 4ac = 0, the roots are real and equal (or quadratic equation is a perfect
trinomial)
2. n
ab = n a n b 3. When b2 – 4ac < 0, the roots are imaginary and unequal (complex conjugates)
a na
3. n = The roots:
b nb
4. m n
a = mn a − b + b 2 − 4ac − b − b 2 − 4ac
X1 = X2 =
2a 2a
Laws of Logarithms: Sum of the roots, x1 + x2 = - b/a
1. log b MN = logb M + logb N
M Product of the roots, x1 ∙ x2 = c/a
2. logb = logbM - log bN
N THE BINOMIAL THEOREM
3. logbMN =NlogbM
EXPANSION PASCAL’S
Important Properties: BINOMIAL TRIANGLE
1: a 0 = 1 provided a ≠ 0 (x + y)0 = 1 (x ≠ -y) 1 1
(x + y)1 = x+y 1 1
1
2: a
-n
= (x + y)2 = x2 + 2xy + y2 1 2 1
an (x + y)3 = x + 3x2y + 3xy2 + y3
3
1 3 3 1

( a)
m
m (x + y)4 = x + 4x3y + 6x2y2 +4xy3 + y4
4
1 4 6 4 1
3: a n = n am = n
(x + y)5 = x + 5x4y + 10x3y2 +10x2y3 + 5xy4 + y5
5
1 5 10 10 5 1
(x + y)6 = x + 6x5y + 15x4y2 +20x3y3 + 15x2y4 + 6xy5 + y6
6
1 6 15 20 15 1
4: a logab = b or e lnb = b
5: a m = a n implies that m = n This array of numbers is called the Pascal’s Triangle. Any lower row is formed by adding
6: log M =N implies that M = b N any two adjacent numbers of the upper row and place 1 at both ends so as to form a triangle.
b
7: log M =logbN implies that M = N Pascal’s Triangle is used to easily recall the numerical coefficients of the expansion of the
b
powers of a binomial. But for large powers of a binomial, Pascal’s Triangle becomes
logbN inconveniently to use. For such, use Binomial Theorem.
8: log aN =
logb a
n(n-1)(n-2)…(n-r+2)
9: log b =1
b provided b > 0, b ≠ 1 The rth term of (x + y)n =
(r – 1)!
xn-r+1yr-1
10: log 1 =0 provided b > 0, b ≠ 1
b
LOWEST COMMON MULTIPLE (LCM) Arithmetic Mean
The lowest common multiple (LCM) of several natural numbers is the smallest natural The arithmetic mean between two numbers is the number which when placed between
number of which each of the given numbers is a factor. It mat be found by taking the product the two numbers, forms with them an arithmetic progression.
of all the different prime factors of the numbers, each taken the greatest number of times that
it occurs in any of those numbers. In general, for n terms, arithmetic mean (AM) = a1 + a2 + a3 + … + an
n
Example: Find the lowest common multiple of 24, 10, 18, and 25.
Solution: Geometric Progression (G.P.)
24 = 2x2x2x3, 10 = 2x5, 18 = 2x3x3, 25 = 5x5 - a sequence of terms in which each term after the first is found by
LCM = 2x2x2x3x3x5x5 = 1800 multiplying the preceding term by a fixed number called common ratio.
- The sequence a1, a2, a3 are in G.P. if and only if:
HIGHEST COMMON FACTOR (HCF) a2/a1 = a3/a2 = r
The highest common factor (HCF) of several natural numbers is the largest natural number The nth term, an
which is a factor of all the given numbers. It may be found by taking the product of all the an = a1rn-1
different prime factors common to the given numbers, each taken the smallest number of
times that it occurs in any of those numbers. If the given numbers have no prime
factors in common, the HCF is defined to be 1, in this case the numbers are said to Sum of the first n terms in G.P.
be relatively prime. Sn = a1 1-rn
1-r
Example: Find the highest common factor of 24, 30, 18 and 150. where a1 = first term
Solution: r = common ratio
24 = 2x2x2x3, 30 = 2x3x5, 18 = 2x3x3, 150 = 2x3x5x5 n = number of terms
HCF = 2x3 = 6
Infinite Geometric Progression
The sum of terms in geometric progression can be found if the common ratio
PROGRESSION | r |<1, -1 < r < 1

Arithmetic Progression (A. P.) a1


- a sequence of terms in which each term after the first is obtained by S=
1 −r
adding a fixed number to the preceding term.
- a sequence of terms in which any two consecutive terms has a common Geometric Mean
difference. The term in between the first and last terms of the geometric sequence.
That is, the sequence a1, a2, a3 are in arithmetic progression if and only if: Let x = geometric mean,
a2 – a1 = a3 – a2
a1, x2, a2 → geometric progression
Let: a1 = first term of an A. P.
Then, x/a1 = a2/x →common ratio
an = nth term of an A. P.
d = common difference Solving for x: x2 = a1a2
n = number of terms
Sn = the sum of n terms x= a1 a2 → geometric progression

Then,
an = a1 + (n – 1)d Harmonic Progression
Sn = n/2 (a1 + an)
Sn = n/2 [ 2a1 + (n – 1)d] Sequence of terms whose reciprocal forms an arithmetic progression
That is, a1, a2, a3…an are in harmonic progression 3. VARIATION
If 1/a1, 1/a2, 1/a3…1/an form an arithmetic progression
i. Direct Variation (also direct proportion)
The Five Statements Below Have Same Meaning
As x increases y increase proportionately
Harmonic Mean
y is proportional to x
Let x = harmonic mean between a and b y is directly proportional to x
y varies as x
a, x, b → in H. P. y varies directly as x
1/a, 1/x, 1/b → in A. P. In symbols the above statements mean,
yαx
Then, In mathematical terms,
y = kx
1/x – 1/a = 1/b – 1/x → common difference
where k is called the constant of proportionality or also called the constant of
Solving for x: variation
2/x = 1/a + 1/b ii. Inverse Variation (also indirect variation)
2/x = a + b/ab The following Statements Below Have Same Meaning
As x decreases y increase (and vice versa)
x = 2ab/(a + b) y is inversely proportional to x
y varies indirectly as x
Harmonic Mean (HM) = _ n In symbols the above statements mean,
1/a1 + 1/a2 + 1/a3 + … + 1/an y α 1/x
In mathematical terms,
y = k/x, (x not equal to zero)
RATIO, PROPORTION AND VARIATION
Examples
1. RATIO 1. Boyle’s Law: “When the temperature of a confined gas is held constant, the pressure of the
The ratio of a number a to another number b is the fraction a/b usually as a:b (read a “is gas varies inversely as its absolute pressure.”
to” b). Where a is called antecedent and b is called consequent. 2. Ohm’s Law: “The current is directly proportional to the impressed emf and inversely to the
resistance
2. PROPORTION
Proportion is a statement that two ratios are equal. Usually written as a:b = c:d or a/b = iii. Joint Variation
c/d. Where a and d are called the extremes and b and c are called the means. D is called y varies jointly as x and w
the fourth proportional to a, b, and c. If the means of a proportion are equal, as in a/x = x/b, In symbols,
the number b is called the third proportional to a and x, while the number x is called the mean y α xw
proportional between a and b. It is obvious that the mean proportional between a and b is Mathematically, Y = kxw; Warning: Not y = k(x+w)
equal to their geometric mean. A proportion may be altered in four different ways summarized EQUATION OF THE HIGHER DEGREE
in the tabular form below. • Rational integral term – a constant, or a positive integral power of any variable, or the
product of such qualities.
Basic Proportion Transformation by Transformed Form Ex. 5, 2x4, - 6y3, 15x2y5
Alternation a:c = b:d
a:b = c:d Inversion b:a = d:c • Degree of a term
Addition (a+b):b = (c+d):d -the term Cxn, where C is a constant and n is a positive integer is said to be of degree n
Subtarction (a-b):b = (c-d):d in terms of x.
-the term Cxnyn is said to be of the degree n in terms of x, degree p in terms of y, and if f(x) is divided by (x – r), the remainder is f(r).
degree n+p in terms of x and y.
• Factor theorem
• Polynomial Function
if r is a root of the equation f(x), then (x – r) is factor of f(x).
-An algebraic sum of rational integral term
-A series of a power of a base where he exponents are positive integer
• Converse of factor theorem
-Also called rational integral function.
f(x) = a0xn + a1xn-1 + a2xn-2 + …+ an polynomial function in degree n if (x – r) is a factor of f(x), then r is a root of f(x) = 0
where: n = non-negative integer
a0, a1, a2,….,an are any constants • Depressed equation
a0 ≠ 0 if r is a root of the equation f(x) = 0, then (x – r) is a factor of f(x). Thus, f(x) = (x – r). The
equation Q(x) = 0 is called depressed equation of f(x) = 0, and the roots of Q(x) = 0 are
• Zero of a function the remaining roots of f(x) = 0
any value of the unknown x, that will make a function f(x) equal to zero
also called root of f(x) = 0 • Descartes’ rule of signs
Let f(x) be a polynomial with real coefficients.
• Fundamental theorem a) The number of positive real roots of f(x) is either equal to the number of variations in
every equation f(x) = 0 has at least one root sign of f(x), or that number diminished by a positive integer.
there exist at least one number either real or complex which will satisfy f(x) =0 b) The number negative with real zeros of f(x) is either equal to the number of variations
in sign of f(-x) or that number diminished by positive even integer.
• Number of roots
every equation f(x) = 0 of degree n, has n roots and no more if root of order k is counted • Relationship Between Coefficients and Zeros of Polynomial
as k roots. Given an integral rational function:

• Multiple roots f(x) = a0xn + a1xn-1 + a2xn-2 + … + an-1x + an


consider that the roots r1,r2, r3, …, rn of f(x) = 0 are equal The coefficients for the polynomial function in terms of its zeros can be given as:
if f(x) is exactly divisible by x – r but not by (x – r) 2, then x – r is simple root of f(x) = 0.
-a1/ a0= sum of roots
If f(x) = 0 is exactly divisible (x –r) 2 but not by (x – r1)3, then r1 is called double root of f(x) =
0.
a2 / a0 = sum of product of the roots taken two at a time
In general, if f(x) = 0 is divisible by (x – r) k but not by (x – r1)k+1, then r1 is a k – fold root of
order k.
-a3/ a0 = sum of product of the roots taken three at a time
• Multiplicity
(-1)n an/ a0 = product of all roots
the number of times that r appears as roots of f(x) = 0
Supplementary Problems
• Theorem of complex roots 1. Determine m so that x3 – 2x2 + mx + 8 shall be divisible by x + 3. Ans. m = 9
complex roots always occur in pair. 2. If 3x4 = kx3 + x2 – 16 + 4 is divided by (x – 2), for what value of k will the
That is if b = bi is root, then a – bi is also a root (-conjugate zeros) where a, b are real but remainder be 8 ? Ans. k = 2
b≠0 3. For what value of k will x = 3 be a factor of x3 + 7x2 + kx – 12? Ans. k = 88
4. Find the remainder when 149x1592 – 375x375 + 10 is divided by x + 1. Ans.
• Theorem on quadratic surd roots 1877
surd root always occur in pair 5. One of the roots of 3x3 – mx2 + 23x – 14 = 0 is 2. Determine the value of m.
That is, if a +√bi is a root, then a –√b is also a root where a and b are rational and √b is Answer: m = 14
irrational. 6. 2x3 –x2 + mx + n is to be exactly divisible by x 2 – 2x – 8. Determine the
• Remainder theorem values of m and n. Ans. m = -2, n = -24
7. The sum of the two roots of x3 + 6x + c = 0 is 2; find c. Ans. c = 20
8. What is the sum and product of the roots of the equation 3x 4 – 2x2 + 8x – 6 = 7. Two prime numbers which differ by 2 are called prime twins. Which of the following
0? pairs of numbers are prime twins?
Answer: sum = 0; product = -2 a. (1,3) b. (13, 15) c. (7,9) d. (9,11)
9. Given the equation x3 – 4x2 + 3x – 5 = 0. from the equation whose roots are :
a) negative of the roots of the given equation 8. A relation in which every ordered pair (x,y) has one and only one value of y
b) thrice the roots of the given equation corresponds to the values of x is called
c) the roots of the given equation diminished by 2. a. term b. function c. coordinated d. abscissa
Answer:
a) x3 + 4x2 + 3x + 5 = 0 9. Tossing a coin is generally called
b) x3 + 12x2 + 27x – 135 = 0 a. an experiment b. an event c. an outcome d. a trial
c) x3 + 2x2 – x – 7 = 0
10. The sum of the roots of 2x3 + mx2 – 5x - 3 = 0 equals twice the product of the 10. A polynomial which is exactly divisible by two or more polynomials is called as:
roots. Determine m. Ans. m = -6 a. least common denominator b. common multiple
11. The product of the two roots of the equation x 3 + 5x + 12 = 0 is 4. Find the c. factors d. binomial
third root. Ans. -3
12. The sum of the roots 3x2 – 2mx2 + 4 = 0 is 6. Find m. Ans. m = 9 11. The roots of the equation 2x2 – 13x + 20 = 0 are
a. real and equal b. real and unequal
c. complex and equal d. complex and unequal
MISCELLANEOUS QUESTIONS
12. Each of two or more numbers which is multiplied together to form a product is called
a. term b. multiplier c. kilogram d. kilowatt
1. A set of elements that is taken without regard to the order in which the elements are
arranged is called:
13. In the SI unit, the small letter k means kilo while the capital letter K means
a. combination b. sequence c. permutation d. series
a. Kilometer b. Kelvin c. Kilogram d. Kilowatt
2. When a logarithm is expressed as an integer plus a decimal (between 0
14. Any number that can be expressed as a quotient of two integers (division of zero
and 1), the integer is called the
excluded) is called
a. Briggs Logarithm c. Napierian Logarithm
a. irrational number c. rational number
b. Characteristic d. Mantissa
b. imaginary number d. odd number
3. Any positive integers as 1, 2, 3, etc. is also called
15. A rectangular array of numbers forming m rows and n columns are called as
a. Real Number c. Natural Number
a. determinants c. elements
b. Rational Number d. Irrational Number
b. Pascal’s triangle d. none of the above
4. The set of integers that does not satisfy the closure property under the operation of
16. In the expression n√a, the letter n represents the
a. addition b. subtraction c. multiplication d. division
a. power b. order c. exponent d. radicand
5. An equation which is satisfied by some, but not all, of the values of the variables for
17. A number of the form a + bi with a and b real constants and i is square root of -1 is
which the members of the equation are not defined is called a
called
a. linear equation c. rational equation
a. imaginary number c. complex number
b. conditional equation d. irrational condition
b. radical d. compound number
6. An equation which is satisfied by all of the values of the variables for which the
18. Which of the following nonterminating decimals is rational
members of the equation are defined is
a. 3.14159265… c. 2.470470…
a. linear equation c. rational equation
b. 2.71828182… d.1.141421356
b. conditional equation d. irrational equation
19. A succession of numbers in which one number is designated as first, another as
second, another as third and so on is called a 35. If 3x = 9y and 27y = 81z, then is equal to
a. series c. order of numbers a. 3/7 b. 3/5 c. 3/4 d. 3/8
b. arrangement d. sequence .
36. A certain work can be done in as many days as there are men in the group. If the
20. An equation in which a variable appears under a radical sign is called number of men in the group is reduced by 3, the job will be delayed by 4 days. The number of
a. irradical equation c. irrational equation men originally in the group is
b. quadratic equation d. linear equation a. 8 men b. 10 men c. 12 men d. 14 men

21. If 1/4 and – 7/2 are the roots of the quadratic equation Ax2 + Bx + C = 0, what is the 37. How many terms in the progression 3, 5, 7, 9, … must be taken in order that the
value of B? sum is 2,600?
a. -28 b. -7 c. 4 d. 26 a. 53 b. 52 c. 51 d. 50
38. The other form of logaN = b is
22. If 1/4 and – 7/2 are the roots of the quadratic equation Ax2 + Bx + C = 0, what is the a. N = ab b. N = ba c. N = a/b d. N = ab
value of C?
a. -28 b. -7 c. 4 d. 26 39. In the quadratic equation Ax2 + Bx + C = 0, the product of the root is:
a. C/A b. –B/A c. –C/A d. B/A
23. Radicals can be added if they have the same radicand and
a. exponent b. power c. order d. coefficients 40. Two students were solving a problem that would reduce it to a quadratic equation. The
first student committed an error in the constant term and found the roots to be 5 and 7 while
24. If the roots of ax2 + bx + c = 0, are real and equal, then the second student made an error in the first degree term and gave the roots as 2 and 16. if
a. b2 – 4ac > 0 c. b2 – 4ac < 0 you were to check their solutions, the right equation is:
2
b. b – 4ac = 0 d. b2 – 4ac < 0 a. x2 + 12x + 35 = 0 b. x2 + 18x + 32 = 0
2
c. x + 7x – 14 = 0 d. x2 – 12x + 32 = 0
25. The sum of the integers between 288 and 887 that are exactly divisible by 15 is
a. 23,700 b. 22,815 c. 21,800 d. 24,150 41. Determine the value of k so that the equation x2 + (k-5)x + k – 2 = 0 is a perfect
trinomial square.
26. The sum of the prime numbers between 1 and 15 Ans. 3 or 11
a. 42 b. 41 c. 39 d. 38
42. The expression x4 + ax3 + 5x2 + bx + 6 when divided by (x – 2) leaves the remainder 16,
27. What is the sum to infinity of the sequence 1 + 1/3 + 1/9 + … and when divided by (x – 1) leaves the remainder 10. Find the values of a and b.
a. 2/5 b. 5/6 c. 2/3 d. 3/2 Ans. a = - 11/3, b = 5/3

31. The term free of y in the expansion of is 43. Given the equation x4 + x2 + 1 = 0. Which of the following is not a root?
a. 46 b. 84 c. 47 d. 49 a. 1 /120° b. 1 /135° c. 1 /240° d. 1 /300°

32. If f(x) = x + 2 and g(y) = y + 2, then f[g(2)] equals 44. The area of a square field exceeds another square by 56 square meters. The perimeter
x–2 of the larger field exceeds one half of the smaller by 26 meters. What are the sides of each
a. 6 b. 5 c. 4 d. 3 field?
Ans. larger field, 9m or 25/3m; smaller field, 5m or 11/3m
33. If x3 + 3x2 + (K + 5)x + 2 – K is divided by x + 1 and the remainder is 3, then the value
of K is 45. The sum of the areas of two unequal square lots is 5,200 square meters. If the lots
a. -2 b. -4 c. -3 d. -5 were adjacent to each other, they would require 320 meters of fence to enclose the combined
area formed by them. Find the dimensions of each lot. Ans. 60m and 40m or 68m and 24m
34. The value of K which will make 4x2 – 4kx + 5k a perfect square trinomial is
a. 6 b. 5 c. 4 d.3 BINOMIAL EXPANSION
1. A father is twice older than his son and the sum of their ages is 48. How old is each?
1. Solve the following equations: a. 8, 40 b. 12, 36 c. 16, 32 d. none of these
A) Find the value of x: (a + b)x = (a2 + 2ab + b2)x-1
B) Find the roots of the equation 4x4 + 1 = 0 2. Maria is 36 years old, Maria was twice as old as Anna was when Maria was as old as
Anna is now. How old is Anna now? Ans. 24 years old
2. Without expanding, find the term involving x4 of (3x2 – 2x-1)8. Ans. 90,720x4
3. The sum of the ages of the father and his son is 99. If the age of the son is added to
3. A) Expand to 4 terms (x2/3 – ½)x13 the inverted age of the father, the sum is 72. If the inverted age of the son is subtracted from
B) Find the 9th term in the expression of (x2 + ½)13 the age of the father, the difference is 22. What are their ages? Ans. 74 & 25
C) Write the first four terms and the last term of the expansion of
(3/x – x/3)65 4. Maria is 24 years old now. Maria was twice as old as Ana when Maria was as old as
D) Find the term independent of y in the expansion of (y 2 – y-1)9. Anna is now. How old is Anna now? Ans. 16 years old
Ans. 84
5. A father is twice as old as his son and the sum of their ages is 48. How old is each?
4. Which of the following has no middle term? a. 8, 40 b. 12, 36 c. 16, 32 d. nota
a. (x + y)3 b. (a – b)4 c. (u + v)6 d. (x – y)8
6. Pedro is as old as Juan was when Juan is twice as old as Pedro was. When Pedro will
5. Find the middle term of (x2 – 2y)10 be as old as Juan is now, the difference between their ages is 6 years. Find the age of each
Ans. -8,064x10y5 now. Ans. Juan, 24 years old and Pedro, 18 years old

6. If the middle term in the expansion of (x + 2y) n is kx4y4, find k and n. 7. I am three times as old as you were when I was as old as you are now. When you got
Ans. n = 8, k = 1,120 to be my age together our ages will be 84. How old are we now?
a. 24 & 36 b. 24 & 8 c. 16 & 8 d. 18 & 27
7. If the rth term of (x2 – 2y3)n is Cx8y12, find the value of C. Ans. 1120
8. The sum of the ages of two boys is four times the sum of the ages of a certain number
8. Without expanding, find the 10th term of the expansion of (S – 2t2)14 of girls. Four years ago, the sum of the ages of the girls was one eleventh of the sum of the
ages of the boys and eight years hence, the sum of the ages of the girls will be one half that
9. In the expansion of (x2 + 1/x)12 of the boys. How many girls are there? Ans. 4 girls
find: a) the 6th term
b) the middle term 9. In a family, there are 8 children, two of them are twins. The youngest is 3 years old and
c) the term involving x6 the eldest is 21 years old. Their ages are in arithmetic progression. There are three children
d) the term free of x younger than the twins. How old are the twins? Ans. 12 years

10. Find the sixth term in the expansion of (x/2 + y) 9 10. The sum of the ages of two men equals 99. If the inverted age of the elder is added to
Ans. 63/8x4y5 the age of the younger, the sum is 108. However, if the age of the younger is inverted and
subtracted from the age of the older, the difference is 44. Find the age of the older man.
11. Find the term containing x26 in the expansion of (x-2 + x3)12 a. 67 b. 32 c. 53 d. 46
Ans. 66x26

12. The term containing x9 in the expansion of (x3 + 1/x)15 INTEGER AND DIGIT PROBLEMS

13. Find the coefficient of the expansion of (x2 + y)10 containing x10y5 1. Separate 132 into 2 parts such that the larger divided by the smaller the quotient is 6
a. 149 b. 252 c. 105 d. 10,818 and the remainder is 13. What are the parts? Ans. 17 and 115

AGE PROBLEMS
2. A number of two digits divided by the sum of the digits the quotient is 7 and the
remainder is 6. If the digits of the number are interchanged, the resulting number exceeds 2. How much silver and how much copper must be added to 20kg of an alloy containing
three times the sum of the digits by 5. What is the number? Ans. 83 10% silver and 25% copper to obtain an alloy containing 36% silver and 38%
copper?
3. Six times the middle of a three digit number is the sum of the other two. If the number is a. 14kg, 12kg b. 16kg,14kg c. 12kg,10kg d. 16kg, 18kg
divided by the sum of the digits, the answer is 51 and the remainder is 11. If the digits are
reversed, the number becomes smaller by 198. Find the number. Ans. 725 3. A tank full of alcohol is emptied of one third of its content and then filled up with water
and mixed. If this is done six times, what fraction of the volume (original) of alcohol remains?
4. Find the number such that their sum multiplied by the sum of their squares is 65, and Ans.64/729
their difference multiplied by the difference of their squares is 5. Ans. 2 and 3
4. How much tin and how much iron must be added to 50 kilograms of an alloy containing
5. Three numbers are in the ratio 2:5:8. If their sum is 60, find the numbers. Ans. 8, 20, 32 10 percent tin and 25 percent iron to obtain an alloy containing 25 percent tin and 50 percent
iron? Ans. 27.5 kg(tin), 52.5 kg (iron)
6. The sum of the digits of a three-digit number is twelve. The sum of the squares of the
hundreds’ digit and the tens’ digit is equal to the square of the units’ digits. If the hundreds’ 5. How many liters of water must be added to 45 liters of solution which is 90% alcohol in
digit is increased by two, the digits will be reversed. Find the number. Ans. 345 order to make the resulting solution 80% alcohol? Ans. 5.63L

7. April 1978. The square of a number increased by 16 is the same as 10 times the 6. A 40-gram solution of acid and water is 20% acid by weight. How much pure acid must
number. Find the number. Ans. 8, 2 be added to this solution to make it 30% acid? Ans. 5.71 grams

8. The sum of the digits of a three-digit number is 12. The middle digit is equal to the sum 7. How much water must be evaporated form 80 liters of 12% solution of salt in order to
of the other two digits and the number shall be increased by 198 if its digits are reversed. obtain a 20% solution of salt? Ans. 32 L 39. How many liters of water must be added to 100
Find the number. Ans. 264 liters of 85% sulfuric acid solution to produce 60% sulfuric acid solution? Ans. 41.67 L

9. Find three consecutive odd integers such that twice the sum of the first and the 8. A certain solution should contain 8% alcohol. If it was mistakenly mixed to contain 6%
second integers plus four times the third is equal to 60. Ans. 5, 7, 9 alcohol, how much must be drawn from a 5-liter tank and replaced by 10 percent alcohol
solution to provide the proper concentration? Ans. 2.5 L
10. The sum of the digits of a three-digit number is 12. The sum of the squares of the
9. A certain amount of 80% sugar solution added to another amount of 40% sugar
hundreds digit and the tens digit is equal to the square of the units digit. If the hundreds digit
solution yields a solution that contains 14 kg of sugar. Had the amount been reversed, the
is increased by 2 and the units digit is decreases by 2, the digits of the original number will be solution would have contained 16 kg sugar. How much of the 80% solution was there? Ans.
10 kg
reversed. Find the number. Ans. 345
11. The excess of the sum of the fifth and the seventh parts over the difference of the half 10. Ten liters of 25% salt solution and 15 liter of 35% salt solution are poured into a drum
originally containing 30 liters of 10 % salt solution. What is the percent concentration of salt in
and third parts of number is 259. What is the number? Ans. 1470
the mixture?
12. The sum of the digits of the three-digit number is 6. The middle digit is equal to the sum of a.19.55% b. 22.15% c. 27.05% d. 25.72%
the two other digits and the number shall be increased by 99 if the digits are reversed. Find
the number. Ans. 132
RATE AND MOTION PROBLEMS
MIXTURE PROBLEMS
1. A motorist is traveling from town A to town B at 60 kph and returns from town B to town
A at 30 kph. His average velocity for the roundtrip is
1. The tank of a car contains 50 liters of alcogas 25% of which is pure alcohol. How
A. 45 kph B. 40 kph C. 35 kph D. NOTA
much of the mixture must be drawn off which when replaced by pure alcohol will yield a 50-
50% alcogas?
a. 16 2/3 b. 15 1/3 c. 14 d. 20
2. At the recent Olympic games in Montreal, Canada, a team which participated in per hour. The airplane can fly 280 kilometers per hour in still air. If the package carrier takes 3
1600 meters relay event had the following individual speed. First runner, 24 kph, second 2/3 hours in going from A to C and 3 1/6 hours for the return trip, what is the total distance of
runner, 20 kph, third runner, 22 kph and fourth runner 23 kph. What was the team’s speed. travel covered by the man? D = 605 km. t1 = 1.5 hr., t 2 = 5/3 hr.
Ans. 22.149 kph
12. A motorcycle messenger left the rear of a motorized troop 8 kilometers long and rode
3. A troop of soldiers marched 15 km, going to the concentration camp after they were to the front of the troop, returning at once to the rear. How far did he ride, if the troop traveled
forced to surrender, at the same time that the victorious general who is supervising the 15 kilometers during this time and each traveled at a uniform rate? Ans. 25kms.
“march” rode from the rear of the troop to the front and back at once to the rear. If the
distance covered by the victorious general is 25 km. and both the troop and general traveled 13. An army officer made the first part of the trip on a plane which flew at the rate of
at uniform rate, how long is the troop? Ans. 8 km. 210 kilometers per hour. At the landing field, he was met by a jeep which took him the rest of
the way to his destination at a rate of 40 kilometers per hour. The trip required 3 hours and 15
4. Two cyclists are practicing on a circular tract of circumference 276 meter. Starting at minutes. On his return trip, the jeep traveled at the rate of 50 kilometers per hour and the
the same instant and from the same place, when they run in opposite directions they pass plane which he took flew at the rate of 200 kilometers per hour. The return journey required
each other every 6 seconds and when they run the same direction the faster passes the same amount of time, but this included a minute which he spent waiting for the plane to
the slower at every 23 seconds. Determine their rates. Ans. F= 29 m/s, S= 17m/s take off. Find the total distance that he flew and the total distance that he traveled by jeep.
Ans. 532km (by plane) and 28 2/3km (by jeep)
5. Two cars A and B start at the same point and at the same time and travel in opposite
directions, car B traveling 20 km/hr slower than A. If they are 420 kilometers apart after 3 14. A BMW car drives from A toward C at 30 miles/hr. Another car starting from B at the
hours, find the rate of each. Ans. 60 kph, 80 kph same time, drives towards A at 20 mi/hr. If AB = 20 miles, find when the cars will be nearest
together. Ans. 24 min.
6. Two cars A and B are to race around a 1,500-meter circular track. If they will start at
the same point and travel opposite directions, they will meet for the first time in 3 minutes. But 15. Two boats started their voyages is in a straight line towards each other. One has an
if they will travel in the same direction, with the same starting point, car A will reach the average navigational speed of 30 km/h and the other one has an average of 20kph.
starting point with car B trailing behind by 500 meters. What should be the rates of each? Assuming that they can not avoid a collision, how long will it take before the collision occurs?
Ans. 300 m/min, 200 m/min How far would each boat have traveled before the collision? Ans. 4 hrs, 120 km, 80 km

7. A one kilometer long caravan of men is walking at a constant rate. A man from the 16. A man traveling 40 km finds that by traveling one more km per hour, he would made
rear ends walk towards the head and back to the rear at the instant when the caravan has the journey in 2 hours less time. How many kilometer per hour did he actually travel?
covered a distance of one kilometer. Find the total distance traveled by the man. Ans. 2.414 a. 4 b. 8 c. 18 d. 6
km
PROGRESSION
8. A boy started one hour and twenty minutes earlier than a man. If the man ran at 6
kph faster than the boy and overtook the boy in 40 minutes, find the rate of each. Ans. 3 kph 1. Two numbers differ by 40 and their arithmetic mean exceeds their positive geometric
for the boy and 9 kph for the man mean by 2. The numbers are
a. 45, 85 b. 64, 104 c. 81, 121 d.100, 140
9. A man walked 24 km in time T. During the first part of this time, he walked at 6 kph and
the last part at 4 kph. Had he reversed his rates, he would have walked two km more. Find 2. A sets out to walk at the rate of four km per hour. After he had been walking for 2-3/4
the time. Ans. 5 hrs. hours, B sets out to overtake and went 4-1/2 km the first hour, 4-3/4 km the second hr., 5 km
the third hr and so on gaining 1 quarter of a km. every hour. In how many hours would B
10. A traffic check counted 390 cars passing a certain spot on one day and 430 cars at the overtake A? Ans. 8 hours
same spot on the second day. On the first day, there were three times as many cars going
east and half as many going west on the second day. What was the total number of west 3. A besieged fortress is held by 5700 men who have provisions for 66 days. If the
bound cars for the two days? garrison loses 20 men each day, for how many days can the provisions hold out? Ans. 76
Ans. 280 east, 540 west days
11. A man is sent to deliver an important package ant travels by car 75 kilometers per
hour from point A to B and then by airplane to point C against a wind blowing 40 kilometers
4. The sum of three numbers in arithmetic progressions is 60. If the numbers are
increased by 2, 1, and 28, respectively, the new numbers will be in geometric progression. 17. In the series 1.01, 1.0, .099, .098… Find the 80th term.
Find the arithmetic progression.
18. Find the sum of 3+0.4+0.05+0.004+0.0005+… Ans. 38/11
5. Three numbers are in arithmetic progression. Their sum is 15, and the sum of their
squares is 83. Find the numbers. 19. An arithmetic progression starts with 1, has 9 terms, and the middle term is 21.
Determine the sum of the first 9 terms.
6. A 20-liter container is filled with pure acid. Five liters are drawn off and replaced with
water; then 5 liters of the mixture drawn off and replaced with water, and so on until 5 20. A pendulum swings 24 inches for the first time. It is swinging 11/12 of its previous
drawings and 5 replacements have been made. Find the amount of acid in the final mixture. swing. What would be the total distance traveled when the pendulum
stopped?
7. A rubber ball is dropped from a height of 27 meters. Each time that it hits the ground it a. 246 b.264 c.288 d. 312
bounces to a height 2/3 of that from which it fell. Find the distance that it travels up to the time
that it hits the ground for the 5 th time. The total distance traveled by the ball until it comes to 21. A small line truck hauls poles from substation stockyard to pole sites along a proposed
rest. distribution line. The truck can handle only one pole at a time. The first pole site is 150 meters
from the substation and the poles are to be 50 meters apart. Determine the total
8. A man wishes to buy a piece of land worth 150,000 pesos. If it were possible for him distance traveled by the line truck, back and forth, after returning from delivering
to save one centavo on the first day, two centavos on the second day, 4 centavos on the third the 30th pole.
day and so on, in how many days would he save to be able to buy the land? Ans. about 24 a. 35.0km b. 30.0km c. 37.5km d. 40.0km
days
22. Two positive numbers may be inserted between 3 and 9 such that the first three are
9. The sum of the two numbers is 20 and their positive geometric mean is one greater in geometric progression, while the last three are in arithmetic progression. What is the sum
than one half of their arithmetic mean. Find their difference. of these two positive integers?
10. A man cuts a piece of paper 0.03 mm thick into three equal parts. Then he cuts each of a.1.25 b. 12.25 c. 11.25 d.6.25
these parts into three equal parts again and the process is repeated 10 times after which he
piles together the pieces of paper. How thick is the pile? 23. A man piles 150 logs in layers so that the top layer contains 3 logs and each lower
layer has one more log than the layer above. How many logs are at the bottom? Ans. 17 logs
11. A rich man called his seven sons. He had with him a number of pebbles, each pebble
representing a gold bar. To his first son, he gave half the pebbles that he had and one pebble 24. A body falls 16.1 meters during the first second, 48.3 meters during the second, 80.5
more. To his second son, he gave half the remaining pebbles and one pebble more. He did meters during the third second and so on. How long will it take the body to reach the ground if
the same to each to his five other sons and then found out that he had one pebble left. How it was released at an altitude of 15,000 meters? Ans. 30.5 seconds
many pebbles were there initially? Ans. 382
25. The 18th and the 52nd terms of an arithmetic progression are 3 and 173, respectively.
12. A man receives a salary of P36,000 per annum for the first year and a 10% raise The 25th term is
every year for ten years. What is his salary during the fifth year? Ans. P52,707.60 a. 38 b. 35 c. 28 d. 25

13. A car running at 25 kilometers per hour can cover a certain distance in 8 hours. By how 25. Find the number of terms of a geometric progression in which the first term is 48, the
many kilometers per hour must its rate be increased in order to cover the same distance in last term is 384 and the sum of the terms is 720. Ans. 4 terms
three hours less? Ans. 15km/hr
26. The sum of three numbers in A.P. is 27. If the first number is increased by 2, the
14. Find the harmonic mean 7, 1, 5, 2, 6 and 3 second by 7, and the third by 20, the resulting numbers will be in G.P. Find the original
a. 2.36 b. 2.46 c. 2.56 d.2.66 numbers.
a. 3, 9, 15 b. 4, 9, 14 c. 5, 9, 13 d. 6, 9, 12
15. The 8th term of an AP is 3 while its 84th term is 273. Find the 35th term.

16. Find the sum to infinity of 1/3, 1/27,1/243… Ans. 3/8 WORK AND DISCHARGE PROBLEMS
12. A steel company has three blast furnaces of varying sizes. If furnaces A, B, C are
1. If 4 men can plow 12 hectares in 8 hours, how many men are needed to plow 24 hectares used full time, 800 metric tons of steel are produced per day. If A and B are used half time
in 24 hours? Ans. 6 men and C full time, 545 metric tons are produced. If A is not used, B is used full time, and C half
dime, 410 metric tons are produced. How many metric tons per day does each furnace
2. A garden can be cultivated by 8 boys in 5 days. The same job can be done 5 men in 6 produce?
days. How long will it take to finish the job if A) the 8 boys and 5 men will work together? B)
only 6 boys ands 3 men will work together? C) two days after the 5 men were working the 8 13. Three observation planes A, B, and C, working together, can map the region in 4 hours.
boys arrived to help? Planes A and B can map the region in 6 hours, planes B and C can map it in 6 hours and 40
minutes. How long would it take each of the planes working alone to map this region?
3. A, B and C can do a piece of work in 10 days, A and B can do it in 12 days, A and C in
20 days. How many days would it take each to do the work alone? Ans. 30, 20 ,60 14. A pump discharging 9 gpm requires 36 hours to fill a tank. If the pump is replaced by
one that will discharge 16 gpm, how long will it take to fill the tank?
4. A boat’s crew rowing at half their usual rate can negotiate 2km. down a river and back a. 64 hr b. 16 hr. c. 20.25 hr d. 40.5 hr
in one hour and 40 minutes. At their usual rate in still water, they would have gone over the
same course in 40 min. Find their rate of rowing in still water. Ans. 32/5 km/hr 15. Two pipes running simultaneously can fill a swimming pool in 6 hours. If both pipes
run for 3 hours and the first pipe is then shut off, it requires 4 hours more for the second to fill
5. Two pipes running simultaneously can fill a tank in 3 hours and 20 minutes. If both the pool. How long does it take each pipe running separately to fill the pool? Ans. 8 & 24
pipes run for 2 hours and the first is then shut off, it requires 2 hours more for the second to
fill the tank. How long does it take each pipe to fill it alone? 16. A man and a boy can dig a trench in 20 days. It would take the boy 9 days longer to
dig it alone than it would take the man. How long would it take the boy to dig alone?
6. A and B can do a job in 12 days. A and C can do the same job in 18 days while B and a. 45 days b. 16 days c.25 days d. 4 days
C can do it in 24 days. How will it take A, B and C to do the job together?
17. A job can be done in as many days as there are men in the group. If the number of men
7. A man can finished a certain job in three-fourths the time that the boy can; the boy is reduced by 3, the job will be delayed by 4 days. How many men are there originally in the
can finish the same job in two-thirds the time that a girl can; and the man and the girl working group?
only jointly can finish the job in 4 hours. How long will it take to finish the job if they all work a. 6 b. 12 c. 20 d. 30
together? Ans. 8/3 hr.
VENN DIAGRAM
8. The intake pipe to a reservoir is controlled by a valve which automatically closes 1. A certain part can be defective because it has one or more out of three possible
when the reservoir is full and opens again when four-fifths of the water had been drained off. defects; insufficient tensile strength a burr or diameter outside of tolerance limits. In a lot of
The intake pipe can fill the reservoir in 4 hours and the outlet pipe can drain it in 10 hours. If 500 pcs:
the outlet pipe remains open, how much time elapses between the two instants that the 19 have a tensile strength defect
reservoir is full? Ans. 13.3 hr. 17 have a burr
11 have an unacceptable diameter
9. Two brothers washed the family car in 24 minutes. Previously, when each had 12 have tensile strength and burr defects
washed the car alone, the younger boy took 20 min. longer to do the job than the older boy. 7 have tensile strength and diameter defects
How long did it take the older boy to wash the car alone? Ans. 40 minutes 5 have burr and diameter defects
2 have all three defects
10. A swimming pool holds 54 cubic meters of water. It can be drained at a rate of one a. how many have four defects
cubic meter per minute faster than it can be filled. If it takes 9 minutes longer to fill it than to b. how many pcs have only a burr defect
drain it, find the drainage rate. Ans. 3 cu.m/min c. how many pcs have exactly two defects. Ans. 475, 2, 18

11. One input pipe can fill a tank alone in 8 hrs. another input pipe can fill it alone in 6 2. During the election, the total number of votes recorded in a certain municipality was
hours and a drain pipe can empty the full tank in 10 hours. If the tank is empty and all the 12,400 had 2/5 of the supporters of LABAN candidate stampede away from the pools and ½
pipes are wide open, how long will it take to fill the tank? Ans.5.22 hours of the supporters of GAD candidate behaved likewise, the LABAN candidates majority would
have been reduced by 100. How many votes did the LABAN & GAD candidates actually 8. A student left his home to attend a party one morning at past 6 o’clock and returned
received? Ans. 7,000, 5,400 at past 3 o’clock. He noticed the hands of the wall clock have exchanged position. What exact
time did he arrive?
3. The President just recently appointed 25 Generals of the Phil. Army of these 14 have a. 3:26.07 b. 3:34.62 c. 3:31.47 d. 3:32.19
already served in the war of Korea, 12 in the war of Vietnam and 10 in the war of Japan.
Therefore 4 who have served both in Korea and Japan, 6 have served both in Vietnam and 9. How many times in one complete day will the hour and the minute hands coincide
Korea and 3 have served in Japan, Korea, and Vietnam. Ans. 2 generals with each other?
a. 24 b. 23 c. 22 d. 25
4. A survey of 500 T.V. viewers proceed the following result:
285 watch football games RATIO, PROPORTION AND VARIATION
195 watch hockey games
115 watch basketball games 1. The kilowatt that can be transmitted safely by a shaft varies directly as the number of
45 watch football and basketball revolution it makes per minute and the cue of its diameter. If a shaft 3 centimeters in diameter
70 watch football and hockey making 200 revolutions per minute can safely transmit 60 kilowatt, what kilowatt can be safely
50 watch hockey and basketball transmitted by a 2-centimeter shaft making 300 revolutions per minute?
50 do not watch any of the 3 games,
How many watch the basketball games only? 2. The time required for an elevator to lift a weight varies directly with the weight and
a.50 b.40 c.30 d.60 distance through which it is lifted and inversely as the power of the motor. If it takes 30
seconds for a 10 HP motor to lift 100 lbs through a height of 50 ft, what size of motor is
required to lift 800 lbs in 40 sec through the height of 40 ft? Ans. 48 HP
CLOCK PROBLEMS
3. Eight men can excavate 15m3 of drainage open canal in 7 hrs. Three men can
1. At what time between 7 and 8 o’clock are the hands of the clock are backfill 10m3 in 4 hrs. How long will it take 10 men to excavate and back fill 20 m 3 in the
A) at right angles B) straight line C) coinciding project? Ans. 9.87 hrs.

2. The time is 3:00 o’clock and the hands of the clock are at right angles to each 4. A man is sent to deliver an important package and travels by car 75 kilometers per
other. What is the nearest time of the clock such that the hands of it will be at right angles hour from point A to B and then by airplane to point C against a wind blowing 40 kilometers
again? Ans. 32.72 minutes per hour in still air. If the package carrier takes 3 2/3 hours in going from A to C and 3 1/6
hours for the return trip, what is the total distance of ravel covered by the man?
3. How long will it be from the time the hour hand and the minute hand of a clock are
together until they will be together again? Ans. 1 hr. and 5.45 min 5. A sphere 30 cm in diameter is divided into two segments. One of which is two times as
high as the other. Find the volume of the bigger segment.
4. At what time between 4 and 5 o’clock do the hands of the clock coincide? Ans.
4:21.82 o’clock 6. Two flywheels are connected by a belt. The radius of the flywheels are 30 in and 50 in.
5. It is exactly 3 o’clock. In how many seconds will the angle formed by the hour hand and The small flywheel has a speed of 350 rpm. Determine the velocity of the belt in ft/sec. What
the minute hand be twice the angle formed by the hour and the second hand? Ans. 22.4 would be the angular velocity of the larger flywheel?
seconds
7. A cylindrical tin can has its height equal to the diameter of its base. Another cylindrical
6. It is now between 9 and 10 o’clock. In 4 minutes, the hour hand will be exactly tin can with the same capacity has its height equal to twice the diameter of its base. Find the
opposite the position occupied by the minute hand 3 minutes ago. What is the time now? ratio of the amount of tin required for making the two cans with covers. Ans. 0.9524
Ans. 9:20
8. The diameters of two spheres are in the ratio 2:3 and the sum of their volumes is
7. How many minutes after 2:00 o’clock will the hands of the clock extend in the 1,260 cubic meters. Find the volume of the larger sphere.
opposite directions for the first time? Ans. 972 cu. M
a. 40.636 b. 41.636 c. 43.636 d. 42.636
9. If the square root of x varies directly as y and inversely as the square of z and if x = 16 x0 = ½ (x1 + x2) ; y0 = ½ (y1 + y0)
when y = 24 and z = 2, find z when x = 9 and y =2. Ans. 2/3
Angle Between Two Concurrent Lines
10. If a:b = 2:3, b:c= 4:5, what is a:b:c? Let α and β be the inclinations of lines L1 and L2 respectively and let θ be the angle between
a. 2:3/4:5 b. 8:12:16 c. 8:12:16 d. 6:9:12 the two lines
y
L1 θ = β - α ;m1 = tan α, m2 = tan β
ANALYTIC GEOMETRY L2 tan θ = tan (β - α)
θ tan θ = tan β - tan α__
Distance Between Two Points P1 (x1,y1) and P2 (x2,y2) 1 + tan β tan α
y tan θ = m2 – m1
P2(x2,y2)
α β 1 + m1m2
d

P1(x1,y1) SUPPLEMENTARY PROBLEMS


1. A point P(x, 3) is equidistant from points A(1, 5) and B(-1, 2). Find x. Ans. ¾
o x
2. Find the locus of points P(x, y) such that the distance from P to (3, 0) is twice its distance
to (1, 0). Ans. 3x2 – 3y2 – 2x – 5 = 0
d = √ (x2 – x1)2 + (y2 – y1)2
Area of Polygon (Non-overlapping) of n-sides Given Vertices 3. Find the length of the segment joining the two midpoints of the sides of the triangle if the
Given vertices (x1, y1), (x2, y2), ……… (xn, yn) oriented counterclockwise length of the third side opposite to it is 30 cm. Ans. 15 cm.

4. A line from P(1, 4) to Q(4, -1) is extended to a point R so that PR = 4PQ. Find the
x1 x2 x3 …………x1 coordinate of R. Ans. R(13, -16)
A=
y1 y2 y3 …………y1 5. Two vertices of a triangle are (0, -8) and (6, 0). If the medians intersect at (9, -3), find the
third vertex of the triangle. Ans. (-3, -1)
+ + +
A = ½ [ (x1y2 + x2y3 + x3y4 ……. + xny1) – (y1x2 + y2x3 + y3x4 ……. + ynx1)] 6. The area of a triangle with vertices (6, 2), (x, 4) and (0, -4) is 26. Find x.Ans. – 2/3 and 50/3

Division of Line Segment 7. Find the length of the median from A of a triangle ABC given vertices A(1, 6), B(-1, 3) and
Let P(x, y) be a point on the line joining P 1(x1, y1) and P2(x2, y2) and located in such a way that C(3, -3). Ans. 6
segment P1P is a given fraction k of P1P2, that is P1P = kP1P2.
8. If the midpoint of a segment is (5, 2) and one endpoint is (7, -3), what are the coordinates
y of the other end? Ans. (3, 7)
P2(x2,y2)
9. Given vertices of a triangle ABC :
P(x,y) x = x1 + k (x2 – x1) A(1, 5),B(-1, 1) and C(6, 3). Find the intersection of the median. Ans.(2, 3)
y = y1 + k (y2 – y1)
P1(x1,y1) 10. Find the inclination of the line 2x + 5y = 10. Ans. 158.2°

o x
Locus – the curve traced by an arbitrary point as it moves in a plane is called locus of a
point.
If k = ½, then formula above becomes a midpoint formula
– the locus of an equation is a curve containing only those points whose coordinates
satisfy the equation.
y
4. Two- Intercept Form
EQUATION OF A STRAIGHT LINE x +y =1 P2(0, b)
Line – is a locus of points which has constant slope. a b P(x, y)
Theorems : where a = x intercept
• Every straight line can be represented by a first-degree equation. b = y intercept P1(a, 0) x
• The locus of an equation of the first degree is always a straight line.

General Equation of a Line


Ax + By + C = 0 ; A, B, C are constants ; A and B, not zero at the same time 5. Normal Equation of a Straight Line
y
Standard Equation of a Line
1. Two Point Form
Y P2(x2,y2)
P(x,y) By similarity of triangles x
y – y1 = y2 – y1 (x – x1) (1)
P1(x1,y1) x x2 – x1 Given ρ = normal intercept N(ρcosθ,ρsinθ)
= segment from the origin ρ
y perpendicular to the required line θ ρsinθ
θ = normal angle ρcosθ
2. Point-Slope Form P(x, y) = inclination of the normal intercept
In (1) replacing y2 – y1 by m,
x2 – x1
From the point slope form :
y – y1 = m (x – x1) θ
y – y1 = m (x – x1) where x1 = ρcos θ, y1 = ρsin θ
x
mL = -1 / tan θ
y - ρsin θ = (-1/ tan θ ) (x–ρcos θ)
m = tan θ
Simplifying, xcosθ + ysinθ = ρ
0° ≤ θ ≤ 180°

y Reduction to Normal Form :


3. Slope-Intercept Form
y = mx + b (0, b) Given the line Ax + By + C = 0
where m = slope P(x,y) The normal form is :
b = y intercept A x + B y + C =0
x
±√ A2 + B2 ±√ A2 + B2 ±√ A2 + B2 a) find the equation of the median from A
b) find the equation of the altitude from B
Note : The sign of the radicand must be chosen such that the last term will become c) the intersection of medians from B to C
negative since ρ > 0. Ans. a) x – 2y – 2 = 0 b) x + y – 1 = 0 c) (4, 1)
12. Find the normal intercept and the normal angle of line 5x+12y–39 = 0
Special Cases of a Straight Line Ans. ρ =3, θ = 67.38°

Α A. Equation of the x – axis: y=0 y


Equation of a horizontal line : y=b where b is a constant Distance Between Parallel Lines L1
B. Equation of the Y-axis : x=0 Let the parallel lines be given by the equations :
Equation of a vertical line : x=a where a is a constant
L1 : Ax + By + C1 = 0 L2
SUPPLEMENTARY PROBLEMS:
L2 : Ax + By + C2 = 0
Find the equations of the line/s satisfying the given conditions.
The distance between the two
1. Passing through (1, -2) and perpendicular to the line through (2, -1) and (-3, 2) d
Ans. 5x – 3y – 11 = 0 lines is given by the formula

2. With x intercept of 5 and passing through (3, 4) Ans. 2x + y – 10 = 0 d = C2 – C1_


√ A2 + B 2
3. Passing through (-3, 4) and with equal intercepts Ans. x – y + 7 = 0 and x + y – 1 = 0
Sample Problems:
4. Making an angle of 45° with the x-axis and passing through (2, 3) Ans. x – y – 1 = 0 1. Find the distance from point (3, -1) to the line 3x – 4y – 3 = 0
Solution :
5. With slope -12/5 crosses the first quadrant and forms with the axes a triangle with Here, A = 3, B = -4, C = -2 P0(x0, y0) ↔ (3, -1)
perimeter of 15. Ans. 5x + 12y – 3 = 0
Using the formula d = Ax0 + By0 + C = 3(3) + (-4)(-1) – 3
6. Passing through (7, -4) and at a distance of 1 unit from the point (2, 1) √A +B
2 2
+√ 3 + 42
2

Ans. 4x + 3y – 16 = 0 ; 3x + 4y – 5 = 0
d = 2 units (the point (3, -1) and the origin are on the opposite side of the line)
7. Passing through the midpoint of the segment joining the points (1, 3) and (5, 1) and
parallel to the line 2x – y + 5 = 0 Ans. 2x – 3y – 5
2. Find the distance between parallel lines 8x + 15y + 18 = 0 and 8x + 15y + 1 = 0.
8. Find the value of parameter k so that the line 3x – 5ky + 5 = 0 Solution :
a) will pass through (0, 1)
b) will be parallel to x + 2y = 5 A = 8, B = 15, C2 = 18, C1 = 1
c) will be perpendicular to 4x + 3y = 2 D = C2 – C1 = 18 – 1 = 1 unit Answer
d) has the y-intercept equal to 3
Ans. a) 1 b) –6/5 c) 4/5 d) 1/3 √ A2 + B 2 √ 82 + 152
9. Find the equations of the lines parallel to the line x + 2y – 5 = 0 and passing at a distance Distance from a Point to a Line
2 from the origin Ans. x + 2y + 2√ 5 = 0 and x + 2y - 2√ 5 = 0 The directed distance from a point P(x0,y0) to a line Ax + By + C = 0 is given by the formula:

10. Find the equation of the perpendicular bisector of the segment joining (2, 5) and (4, 3). d = Ax0 + By0 + C
Ans. x – y + 1 = 0 ± √ A2 + B2
11. Given vertices of a triangle ABC, A(2, 0); B(3, -2) and C(7, 5)
where the sign of the radical is chosen to be the opposite that of C. ASYMPTOTE - a straight line which the curve f(x, y) = 0 approaches indefinitely near as its
tracing point approaches to infinity.
Remarks:
• To find the vertical asymptote, solve the equation for y in terms of x and set the
1. If d > 0, the origin and P lie on opposite sides of the given line. linear factors of the denominator equal to zero.
2. If d < 0, the origin and P lie on the same side of the line. • To find the horizontal asymptote, solve the equation for x in terms of y and set
Notes: the linear factors of the denominator equal to zero.
Regardless of the location of the point P0(x0, y0), the distance being always positive the CIRCLE
formula can be expressed using the absolute value as: Circle is the locus of a point which moves so that it is always equidistant from a
fixed point.
d =Ax0 + By0 + C Note: fixed point is called the center
Fixed distance is called the radius
√ A2 + B2
Line Through the Intersection of Two Lines
Let Ax + By + C = 0 and
Equation of a Circle
Dx + Ey + F = 0 be two intersecting lines, where A, B, C, D, E and F are constants
and A = B ≠ 0, E = F ≠ 0. In normal form
Consider a circle of radius r with center at C(u, k)
The equation of the family of lines passing through the intersection of the two given lines is
Let P(x, y) be a point in the circle
given by,
(Ax + By + C) + k (Dx + Ey + F) = 0 y
where k is an arbitrary constant.
P(x,y) By Pythagorean Theorem
r y–k (x – h)2 + (y – k)2 = r2 → standard form
INTERCEPT OF A CURVE C(h,k) x–h
Center at the origin C(0, 0)
• x-intercept – directed distance from the origin to the point where the curve crosses the x-
x 2 + y 2 = r2
axis
To find the x intercept of a curve, set y = 0, then solve for x.
x
• y-intercept – the directed distance from the origin to the point where the curve crosses 0
the y-axis
to find the y- intercept of a curve, set x = 0, then solve for y.

SYMMETRY
• If the equation of a curve does not change upon replacement of y by –y, then the General Form
locus is symmetric with respect to the x-axis. Expanding the form (x – h)2 + (y – k)2 = r2 becomes
f(x, -y) = f(x,y) =0 x2 + y2 – 2xh – 2ky + h2 + k2 – r2 = 0
• If an equation of a curve does not change upon replacement of x by –x, then the This is of the form:
locus is symmetric with respect to the y-axis x2 + y2 + Dx + Ey + F = 0 → general form
f(-x,y) = f(x,y) = 0 where D, E, F are constants not all zero at a time.
• If an equation of a curve does not change upon replacement of x by –x and y by Note: By equation of coefficients:
–y, then the locus is symmetric with respect to the origin. -2h = D ; h = -½ D → abscissa of center
f(-x, -y) = f(x, y) = 0 -2k = E ; k = -½ E → ordinate of center
h2 + k2 – r2 = F ; r = √(h2 + k2 – F)
Radical Axis of Two Circles
Consider the two non-concentric circles D. All tangents drawn to two circles from a point on their radical axis have equal lengths.
x2 + y2 +D1x + E1y + F1 = 0
x2 + y2 +D2x + E2y + F2 = 0 y Radical Axis
The equation: P
x2 + y2 +D1x + E1y + F1 + k (x2 + y2 +D2x + E2y + F2) = 0
represent a circle for any value of k except for k = -1 T1 & T2 are points of
T1 tangency
if k = -1, the equation of the family of circles above becomes: T2
(D1 – D2) x + (E1 – E2) y + (F1 – F2) = 0 PT1 = PT2
This represents a straight line called the RADICAL AXIS of two circles.
x
Properties of the Radical Axis
A. If two circles intersect at two distinct points, their radical axis is the common chord of
the circles. Supplementary Problems
1. Find the center and radius of the circle whose equation is x 2 + y2 – 4x –6y –12 = 0
y
(ECE Board Problem – Oct 1981) Ans: C(2, 3) r = 5

Common Chord 2. Find the area of the circle whose equation x2 + y2 = 6x – 8y


(ECE Board Problem – Mar. 1981) Ans: 25π sq. units

0
Condition for Orthogonality x
3. Find the equation of the circle whose center is at (3, -5) and whose radius is 4 units.
2 2
The two non-concentric circles : x + y + D1x + E1y + F1 = 0 Ans: (x – 3)2 + (y + 5)2 = 16
x2 + y2 + D2x + E2y + F2 = 0,
meet at right angles (orthogonal) if : D1D2 + E1E2 = 2(F1 + F2) For Problems 4 – 9, determine the equation of the circle given the following conditions
4. Passes through the point (2, 3), (6, 1) and (4, -3) Ans: x2 + y2 – 10y = 0
B. If two circles are tangent, their radical axis is the common tangent to the circles at their
point of tangency. 5. Center on the y – axis, and passes through the origin and point (4, 2). Ans: x2 + y2 – 10y
=0
Radical Axis
y
6. Passes through the points of intersection of the circles x 2 + y2 = 5, x2 + y2–x + y = 4, and
through the point (2, -3) Ans: x2 + y2 –2x + 2y –3 = 0

7. Center on the line x – 2y –9 = 0 and passes through the points (7, -2) and ( 5, 0)
Ans: x2 + y2 – 10x + 4y +25 = 0

0 x 8. Circumscribe the triangle determine by the lines x – u – 8 = -y and y = -1.


Ans: x2 + y2 –8x + 2y + 8 = 0
C. The radical axis of two circles is perpendicular to their line of centers.
9. Given the endpoints of the diameter (5, 2) (-1, 2) Ans: x2 + y2 – 4x – 4y – 1 = 0
y Radical Axis
AB – line of centers 10. Find the equation of the line tangent to the circle x 2 + y2 – 8x – 8y + 7 = 0 at the point (1,
A B 0)
Ans: 3x + 4y – 3 = 0

0 x
PARABOLA
The locus of a point that moves in a plane such that its distance from a fixed point
equals its distance from a fixed line. 0 a>0 x

Notes:
• Fixed point is called focus C. Vertex at the Origin, Vertical Axis
• Fixed line is called directrix x2 = 4ay
• Axis – the line passing through the focus and perpendicular to the directrix if a is positive (+a) ----- concave upward
if a is negative (-a) ----- concave downward
• Vertex – The midpoint of the segment of the axis from the focus to the directrix.
Notes: y
• Latus rectum – a segment passing through the focus and perpendicular to the axis of the 1. Axis : the y axis axis
parabola. 2. Focus: F(0,a)
• Focal distance – distance from vertex to focus = a 3. Latus Rectum: /4a/ F
Ends: L(-2a,a) L(-2a,a) R(2a,a)
Standard Equations of Parabola R(2a, a)
A. Vertex at V(h,k), Vertical Axis 4. Equation of directrix
(x-h)2 = 4a(y-k) y = -a V(0,0)
if a is positive (+a) ----- concave upward
if a is negative(-a) ----- concave downward directrix

Notes: D. Vertex at the Origin, Horizontal Axis


1. Equation of axis : x=h y axis y2 = 4ax
2. Focus : F(h,k + a) if a is positive (+a) ----- concave to the right
3. End of Latus Rectum L F R if a is negative (-a) ----- concave tot he left
L(h-2a, k+a)
R(h+2a, k+a)
4. Equation of Directrix V(h,k) Notes: y
y = k-a 1. Axis : the x-axis directrix
directrix 2. Focus: f(a,0) L
0 x 3. Latus Rectum = 4a 
Ends: L(a,2a) V F
B. Vertex at V(h,k), Horizontal Axis R(a,-2a) x axis
(y-k)2 = 4a(y-k) 4. Equation of Directrix
if a is positive (+a) ----- concave to the right x = -a R
if a is negative (-a) ----- concave to the left
Remarks:
1. The vertex and focus always lie on the axis of the parabola.
2. Focus is always located on the concave side of the parabola.
Notes:
1. Equation of the axis: y=k directrix General Equations of Parabola
2. Focus: F(h+a, k) y L 1. Vertical Axis
3. Ends of Latus Rectum: • Ax2 + Dx + Ey + F = 0, E or A must not be zero
L(h+a, k+2a) V F axis
2. Horizontal Axis
R(h+a, k-2a) (h,k)
• Cy2 + Dx + Ey + F = 0, D or C must not be zero
4. Equation of Directrix
x = h-a R
Supplementary Problems  Major axis – the segment cut by the ellipse on the line containing the foci
1. Find the vertex, focus, and end points of the Latus Rectum of each of the following - a segment joining the two vertices of an ellipse of length equal to
parabolas I2aI
a. 4y2 – x + 2y = 0  Diameters– the chords of an ellipse that pass through the center
Ans: V(-1/4, -1/4), F(-3/16, -1/4) EL(-3/16, -1/4 ± 1/8)  Vertices – the endpoints of the diameter through the foci
- the endpoints of the major axis
b. 2y2 – 5x + 3y - 7 = 0 Ans: V(-13/8, -3/4 ), F(-1, -3/4 ), EL(-1, -3/4 ± 5/4 )  Latus Rectum – the segment cut by the ellipse passing through the foci and
perpendicular to the major axis
2. Find the equation of the parabola determined by the given conditions  Eccentricity – measure the degree of flatness of an ellipse
a. focus at (-11/4, 1) and the endpoint of latus rectum is (-11/4, 5/2) Ans: y2 + 3x – 2y + 7 =
0
Standard Equation of Ellipse
b. vertex at (1, -1) and focus at (1, -3/4) Ans: x2 – 2x – y = 0
Center at C(h, k), Horizontal Major Axis
y
c. vertex at (0, 3) directrix x = -1 Ans: y2 – 4x – 6y + 9 = 0
(x – h)2 (y – k)2
------------ + ----------- = 1
d. axis vertical, vertex (-1, -1) and passing through (2, 2) Ans: x2 + 2x – 3y – 2 = 0
a2 b2
Notes: b
3. A chord passing through the focus of the parabola y2 = 16x has one end at the point (1,
1. Major axis : y = k V1 F1 C(h,k) F2 V2
4). Where is the other end of the chord?Ans: (4, 8)
2. Minor axis ; x = h
3. Vertices : V1(h-a, k) b
4. Find the equation of the line tangent to the parabola x 2 + 2x + 3y – 1 = 0
V2(h+a, k)
Ans: 2x + 3y – 1 = 0
4. Foci : F1(h-c, k)
F2(h+c, k)
c c
5. Find the equation of the circle that passes through the vertex and the endpoints of latus a a
0
rectum of the parabola y2 = 8x. Ans: x2 + y2 – 10x = 0
Center C(h, k), Vertical Major Axis y V1
6. Find the equation of parabola whose axis is horizontal, vertex is on the y – axis and
which through (2, 4) and (8, -2) Ans: y2 = 20y –18 +100 = 0, y2 = 4y + 2x + 4 =0
F1
a
7. An arch in the form of parabolic curve, with a vertical axis is 60 m, across the bottom. The (x – h)2 (y – k)2
–––––––– + –––––––– = 1 c
highest point is 16 m above the horizontal base. What is the length of a beam placed
horizontally across the arch 3m below the top? Ans: 26 m a2 b2 b b

8. Assume that water issuing from the end of a horizontal pipe, 25 ft. above the ground Notes:
c
describe a parabolic curve, the vertex of the parabola being at the end of the pipe, the 1. Major axis : x = h C(h,k) a
flow of water has curve outward 10 ft. beyond a vertical line through the end pipe, how far 2. Minor axis ; y = k F2
beyond this vertical line will the water strikes the ground? Ans: 17. 68 ft. 3. Vertices : V1(h, k+a)
V2(h, k-a)
4. Foci : F1(h, k+c) V2
ELLIPSE F2(h, k-c)
Ellipse is the locus of a point P(x, y) in a plane which moves such that the sum of its 0 x
distances from two fixed points is constant.
Center at the Origin, Horizontal Major Axis
Notes:
 The two fixed points are called foci. (x)2 (y)2
------- + ------ = 1 y General Equation of an Ellipse
a2 b2 Ax2 + Cy2 + Dx + Ey + F = 0 where A ≠ C but of the same sign
Notes:
5. Major axis : x = axis Supplementary Problems:
6. Minor axis ; y = axis V1 F1 F2 V2 1. In the ellipse below determine the following: a) Center b) Vertex c)Foci d) Major Axis
5. Vertices : V1(-a, 0) C(0,0) e) Major Axis f) Latus Rectum g) Eccentricity
V2(a, 0)
7. Foci : F1(-c, 0) A. 25x2 + 16y2 – 50x + 32y – 1559 = 0
F2(c, 0) c c B. 144x2 + 169y2 +864x – 23,760 = 0
a a Ans: A. a) C(1, -1); b) V1(1, 9), V2(1, -11); c) F1(1, 5), F2(1, -7)
d) 20 e) 16 f) 12.8 g) 0.6

B. a) C(-3, 0); b) V1(-16, 0), V2(10, 0); c) F1(-8, 0), F2(2, 0)


d) 26 e) 24 f) 288/13 g) 5/13

2. In each of the following find the equation of ellipse satisfying the given conditions
Center at the Origin, Vertical Major Axis V1 A. center at (0, 0), focus at (±√3, 0), and b = 1
Ans: x2 + 4y2 = 4
B. center at (1, 0) focus at (1, √3) e= √3/2
(x)2 (y)2 Ans: 4y2 + y2 = 8x
------- + ------ = 1 F1
a
C. focus at (0, -1), (-4, -1), a = √6
a2 b2 c Ans: x2 + 3y2 + 4x + 6y +1 = 0
b b D. center at (-1/2, 2) a = 5/2 b = 2
Notes: 2 2
Ans: 16x + 25y + 16x + 4 = 100y
8. Major axis : y = axis C(0,0) E. center at (0, 0), vertex (0, 4) e = ½
9. Minor axis ; x = axis
c
a Ans: 4x2 + 3y2 – 48 = 0
10. Vertices : V1(0, a)
V2(0. -a) F2 3. A satellite orbits around the earth in an ellipse orbit of eccentrically of 0.80 and
11. Foci : F1(0, c) semi – major axis of length 20,000 km. If the center of the earth is at one focus, find the
F2(0, -c) V2 maximum altitude (apogee) of the satellite. Ans: 36, 000 km

4. Find the equation of the locus of a point which moves so that the sum of its distance from
General Remarks (-2, 2) and (1, 2) is 5. Ans: 16x2 + 25y2 + 16x – 100y +4 = 0
1. Vertices and foci lie on the major axis
2. IaI is the distance from the center to the vertex 5. Find the eccentricity of an ellipse whose major axis is thrice a long as its minor axis
3. IcI is the distance from the center to the foci ( focal distance) Ans: 2√ 2/3
4. The ellipse is symmetrical to the major, minor axes and the center.
6. What is the quadrilateral formed by joining the foci of an ellipse to the endpoints of the
Important Relations minor axis? Ans: rhombus
1. a > b, a > c
2. a2 = b2 + c2 7. Find the distance of the point (3, 4) to the foci of the ellipse whose equation 4x 2 + 9y2 =
3. e = eccentricity = c/a < 1 36
4. Latus Rectum, LR = 2b2/a Ans: 3 ±√ 5
8. An arch in the form of a semi – ellipse has a span at 45 m and its greatest height is 12m. 1. Transverse axis: y = k
There are two vertical supports equidistant from each other and the ends of the arc. Find 2. Conjugate axis: x = h
the height of the support. transverse
Ans: 8 √ 2 m. 3. Vertices: V1 (h – a, k) axis
V2 (h + a, k) conjugate axis
9. Determine the locus of a point P(x, y) so that the product of the slopes joining P(x, y) to 4. Foci: F1 (h – c, k)
(3, -2) and (-2, 1) is –6. Ans: 6x2 + y2 – 6x +y – 20 = 0 F2 (h + c, k)
5. Asymptotes: y – k = ± b/a ( x – h) 0 x
10. What is the area of the ellipse whose equation is 25x2 + 16y2 = 400. Ans: 20π sq. units

HYPERBOLA
Hyperbola is the locus of point P(x, y) in a plane which moves such that the difference of
Center at C(h, k), Vertical Transverse Axis
its distances from two fixed points is a positive constant.
Asymptote
Notes:
(y – k)2 _ (x – h)2 = 1 Transverse axis
 The two fixed points are called foci F1
a2 b2
 Transverse axis – a line segment joining the two vertices of hyperbola
V1
- the length of the transverse axis is I 2a I Notes: Conjugate C
 Conjugate Axis – the perpendicular bisector of the transverse axis. axis V2
- the length of the conjugate axis is I2b I 1. Transverse axis: x = h
 Center – point of intersection of transverse and conjugate axis 2. Conjugate axis: y = k F2
 Central Rectangle – the rectangle whose area is (2a) (2b) and whose diagonals 3. Vertices: V1 (h, k + a)
are asymptotes of hyperbola V2 ( h, k – a)
 Vertices – the endpoints of the transverse axis 4. Foci: F1 (h, k + a)
 Asymptotes of the hyperbola – two intersecting lines containing the diagonal of F2 ( h, k – c)
the central rectangle 5. Asymptotes: y – k = ± a/b( x – h)
–To find the equation of the asymptote, set the right side of the equation of
hyperbola in standard form to zero then solve for y.
 Central Circle – the circle of radius c with center at the center of the hyperbola
circumscribing the central rectangle Center at the Origin, Horizontal Transverse Axis
 Equilateral Hyperbola – hyperbola whose transverse axis equals its conjugate
axis x2 _ y2 = 1 y
 Conjugate Hyperbolas – hyperbolas whose transverse axis of one is the a2 b2
conjugate axis of the other
Notes:
1. Transverse axis: x–axis
Standard Equations of Hyperbola 2. Conjugate axis:F1 y–axis
V1 V2 F2

Center at C(h,k), Horizontal Transverse Axis 3. Vertices: V1 (-a, 0) C x


V2 (a, 0)
(x – h)2 _ (y – k)2 = 1 y 4. Foci: F1 (-c, 0)
a2 b2 F2 (c, 0)
5. C Asymptotes: y = ± b/a x
F1 V1
V
2
F 2
Asymptote
Center at the origin , Vertical Transverse Axis
Notes:
y2 _ x2 = 1 y
a2 b2 c. C(0,0); V1(–4,0),V2(4,0); F(–√ 41 ,0), F2(√ 41 ,0);
F1 Asymptote ( y = ± 5/4 x ); TA = 8; CA = 10;LR = 25/2; e = √41/4
Notes:
V1
C 2. Find the equation of the hyperbola satisfying the conditions given in each case
1. Transverse axis: y – axis a. Center(3,–1); vertex(1,–1); focus(0,–1)
V2
2. Conjugate axis: x – axis Ans. 5x2–4y2–30x–8y+21=0
F2
3. Vertices: V1 (0, a)
V2 ( 0, -a) b. vertices at(0,4) and (4,4); foci at (5,4) and (–1,4)
4. Foci: F1 (0, c) Ans. 5x2–4y2–20x+32y–64=0
F2 (0, -c)
5. Asymptotes: y = ± a/b x c. Center at (1,1), vertex(1,3), eccentricity=2
Ans. x2–3y2–2x +10 =0

General Remarks d. Directrices: y=±4; asymptotes: y=±3/2x


1. Vertices and foci are on the transverse axis Ans. 9y2–324x2–208 = 0
2. IaI is the distance from the center to the vertex
3. IcI is the distance from the center to the focus. e. Asymptotes: 3y=±4x ; foci (±6,0)
4. The hyperbola is symmetrical to the transverse and conjugate axis and to the center. Ans. 400x2–225y2–5184=0

f. Foci(0,0), (0,10); asymptote: x+y=5


General Relations Ans. 2x2–2y2+20y–25=0
1. c > a, c > b ( a = b or a < b or a > b)
If a = b, then the hyperbola is called equilateral hyperbola g. Asymptotes: x+y=1 and x–y=1 and passing through (–3,4) and (5,6)
2. c2 = a2 + b2 Ans. x2–y2–2x–2=0
3. Length of Latus Rectum = 2b2
a h. Axes along the coordinates axes, passing through (–2,5)
4. Eccentricity, e = c/a > 1 Ans. 4y2–5x2=19
i. Vertices at (0, ±4) passing through (–2,5)
General Equation of Hyperbola Ans. 4y2–9x2 = 64
Ax2 + Cy2 + Dx + Ey + F = 0 where A and C are of opposite signs
3. Find the eccentricity of a hyperbola whose transverse axis and conjugate axis are equal
in length Ans. e= √ 2
Supplementary Problems
1. Find the center, vertices, foci and asymptotes, transverse axis, conjugate axis, latus THE CONIC SECTIONS (a summary)
rectum and eccentricity of hyperbola below. A conic section is the locus of a point which moves such that its distance from a fixed
a. 9x2–16y2+18x+64y=91 point called focus is in constant ratio and called eccentricity to its distance from a fixed
b. 16x2–4y2–62x+24y+92=0 straight line called directrix.
c. 25x2–16y2=400
Ans. A. General Form of a Quadratic Equation in x and y
a. C(–1,2); V(–1± 2, 2); F(–1± 5/2, 2); asymptote (3x+4y–5=0, 3x–4y+11=0); Ax2 + Cy2 + Dx + Ey + F = 0
TA = 4; CA = 3;LR = 9/4; e = 5/4 1. Ellipse : A ≠ C, same signs
2. Circle : A = C, same signs
b. C(2,3); V1(2,1),V2(2,7); F(2,3+√ 20 ), F2(2, 3–√ 20 ); 3. Hyperbola : A and C have opposite signs
asymptote(2x–y–1=0, 2x+y–7=0); TA = 8; CA = 4;LR = 2; e = √5/2
B. Eccentricity e = c/a
1. Circle : e = 0 1. The sum of the digits of a two-digit number is 11. If the digits are reversed, the resulting
2. Parabola :e=1 number is seven more than twice the original number. What is the original number? a. 38
3. Ellipse : e < 1 b. 53 c. 83 d. 44
4. Hyperbola :e>1 2. A metal washer 1-inch in diameter is pierced by ½-inch hole. What is the volume of the
washer if it is 1/8 inch thick.
Note: a. 0.074 b. 0.047 c. 0.028 d. 0.082
 The circle, parabola, ellipse and hyperbola are called conic sections (or conics) because 3. If a regular polygon has 27 diagonals, then it is a,
any one of them can be obtained geometrically by cutting a cone with a plane. a. nonagon b. pentagon c. hexagon d. heptagon
4. Find the probability of getting exactly 12 out of 30 questions on a true or false question. a.
0.12 b. 0.08 c. 0.15 d. 0.04
5. Find the area bounded by the curve y = 9 − x 2 and the x-axis
a. 18 units2 b. 25 units2 c. 36 units2 d. 30 units2
6. It is a measured of relationship between two variables.
a. Function b. Relation c. Correlation d. Equation
7. A central angle of 45 degrees subtends an arc of 12 cm. what is the radius of the circle? a.
15.28 cm b. 12.82 cm c. 12.58 cm d. 15.82 cm
8. Two posts, one 8 m and the other 12 m high are 15 cm apart. If the posts are supported by
a cable running from the top of the first post to a stake on the ground and then back to the top
of the second post, find the distance from the lower post to the stake to use minimum?
a. 6 m b. 8 m c. 9 m d. 4 m
9. A regular octagon is inscribed in a circle of radius 10. Find the area of the octagon.
a. 228.2 b. 288.2 c. 238.2 d. 282.2
10. The volume of the two spheres is in the ratio 27:343 and the sum of their radii is 10. Find
the radius of the smaller sphere.
Circle Ellipse Parabola Hyperbola a. 5 b. 4 c. 3 d. 6
11. Find the approximate change in the volume of a cube of side “x” inches caused by
increasing its side by 1%.
a. 0.30x2 in3 b. 0.02 in3 c. 0.1x3 in3 d. 0.03x3 in3
 If the cutting plane is perpendicular to the axis of the cone, the section is a circle. 12. The time required for the examinees to solve the same problem differ by two minutes.
 If the cutting plane is making an angle ( other than 90 o) with the axis of the cone, the Together they can solve 32 problems in one hour. How long will it take for the slower problem
section is an ellipse. solver to solve a problem?
 If the cutting plane is parallel to one of the elements of a cone, the section is a parabola a. 5 minutes b. 2 minutes c. 3 minutes d.4 minutes
 If the cutting plane is parallel (but not coincident) to the axis of the cone, the section is a 13. If a = b, then b = a. This illustrates which axiom in Algebra?
hyperbola. a. Transitive axiom b. Replacement axiom
c. Reflexive axiom d. Symmetric axiom
In each of the cases, the cutting plane should not pass through the vertex of the cone, 14. Find the distance of the directrix form the center of an ellipse if its major axis is 10 and its
otherwise the section hat will be formed is a degenerate conic. minor axis is 8.
a. 8.5 b. 8.1 c. 8.3 d. 8.7
Degenerate Conic ( one point, one line, two lines) is a conic formed if the cutting plane is 15. A regular hexagonal pyramid has a slant height of 4 cm and the length of each side of the
passing through the vertex along one of its elements base is 6 cm. Find the lateral area.
a. 62 cm2 b. 52 cm2 c. 72 cm2 d. 82
Principal Axis of a Conic – is the line through the focus and perpendicular to the 16. At the surface of the earth 9 = 9.806 m/s 2. Assuming the earth to be a sphere of radius
directrix 6.371 x 106 m, compute the mass of the earth.
a. 5.12 x 1024 kg b. . 5.97 x 1023 kg
Diameter of a Conic – the locus of the midpoints of a system of parallel chords. 24
c. 5.97 x 10 kg d. . 5.62 x 1024 kg
Direction: Encircle the letter corresponding to the correct answer. 17. The perimeter of an isosceles right triangle is 6.6824. Its area is
a. 4 b. 2 c. 1 d. ½ 32. Find the sum of the rots of 5 x 2 − 10 x + 2 = 0.
18. Determine the vertical pressure due to a column of water 85-m high. a. -1/2 b. -2 c. 2 d. ½
a. 8.33 x 105 N/m2 b. 8.33 x 104 N/m2 33. In a box there are 25 coins consisting of quarters, nickels and dimes with a total amount
c. 8.33 x 106 N/m2 d. 8.33 x 103 N/m2 of $ 2.75. If the nickels were dimes, the dimes were quarters and the quarters were nickels,
19. a 40-gm rifle with a speed of 300 m/s strikes into a ballistic pendulum of mass 5 kg the total amount would be $ 3.75. How many quarters arethere?
suspended from a cord 1 m long. Compute the vertical height through which the pendulum a. 12 b. 16 c. 10 d. 5
rises. 34. A point moves so that its distance from the point (2,-1) is equal to its distance from the
a. 28.87 cm b. 29.42 cm c. 29.88 cm d. 28.45 cm axis. The equation of the locus is.
20. What is the area of the largest rectangle that can be inscribed in a semi-circle of radius
a. x 2 − 4 x + 2 y + 5 = 0 b. x 2 − 4 x − 2 y + 5 = 0
10?
a. 2 50units 2 b. 100 units2 c. 1000 units2 d. 50units 2 c. x 2 + 4 x + 2 y + 5 = 0 d. x 2 + 4 x − 2 y − 5 = 0
21. The amount of heat needed to change solid to liquid is 35. You loan from a loan firm an amount of P 100,000 with a rate of simple interest of 20%
a. condensation b. cold fusion but the interest was deducted from the loan at the time the money was borrowed. If at the end
c. latent heat of fusion d. solid fusion of one year you have to pay the full amount of P 100,000 what is the actual rate of interest.
22. Mr J. Reyes borrowed money from the bank. He received from the bank P1,842 and a. 25.0% b. 27.5% c. 30.0% d. 18.8%
promise to repay P 2,000 at the end of 10 months. Determine the simple interest. 36. It is a polyhedron of which two faces are equal polygons in parallel planes and the other
a.19.45% b. 15.70% c. 16.10% d. 10.29% faces are parallelograms.
23. Find the length of the vector (2,4,4) a. Tetrahedron b. Prism c. Frustum d. Prismatoid
a. 7.00 b. 6.00 c. 8.50 d. 5.18 37. A railroad is to be laid –off in a circular path. What should be the radius if the track is to
24. According to this law, “The force between two charges varies directly as the magnitude of change direction by 30 0 at a distance of 157.08 m?
each charge and inversely as the square of the distance between them.” a.150 m b. 200 m c. 250 m d. 300 m
a. Law of Universal Gravitation b. Coulomb’s Law 38. A 200-gram apple is thrown from the edge of a tall building with an initial speed of 20 m/s.
c. Newton’s Law d. Inverse Square Law What is the change in kenetic energy of the apple if it strikes the ground 50 m/s? a. 130
25. A loan of P 5,000 is made for a period of 15 months, at a simple interest rate of 15 %, Joules b. 210 Joules c. 100Joules d. 82 Joules
what future amount is due at the end of the loan period. 39. A machine costs P 8, 000 and an estimated life of 10 years with a salvage value of P 500.
a. P 5,637.50 b. P5,937.50 c. P 5,900.90 d. P5,842.54 What is its book value after 8 years using straight-line method?
a. P 2,000 b. P 4,000 c. P 3, 000 d. P 2, 500
40. The distance between the points AB defined by A( cos A,−sin A) and
d2y
26. If y = x ln x, find . B ( sin A, cos A) is equal to
dx 2
a. cos A b. 1 c. 2 d. 2 tan A
−1 −1 1 1
a. b. c. d. 41. What nominal rate, compounded semi-annually, yields the same amount as 16%
x x2 x x2 compounded quarterly?
27. The integral of any quotient whose numerator is the differential of the denominator is the a. 16.64% b. 16.16 c. 16.32 d. 16.00%
____________. 42. If (2log x to the base 4) – (log9 to the base 4) = 2, find x.
a. cologarithm b. product c. logarithm d. derivative a. 10 b. 13 c. 12 d. 11
28. Find the nominal rate, which if converted quarterly could be used instead of 12% 43. The point of intersection of the planes x + 5 y − 2 z = 9 ,3x − 2 y + z = 3 and x + y + z = 2
compounded semi-annually.
is at
a. 14.02% b. 21.34% c. 11.29% d. 11.83%
29. Evaluate the expression (1 + i2 )10 where I is an imaginary number. a. (1,2,1) b. ( 2,1,-1) c. (1,-1,2) d. ( - 1,-1,2)
a. 1 b. 0 c. 10 d. -1 44. To compute the value of n factorial, in symbolic form (n!); where n is large number, we
30. A VOM has a selling price of P 400. If its selling price is expected to decline at a rate of use a formula called
10% per annum due to obsolence, what will be its selling price after 5 years? a. Richardson-Duchman Formula b. Diophantine Formula
a. P 213.10 b. 249.50 c. 200.00 d. 236.20 c. Stirling’s Approximation d. Matheson’s Formula
31. If tan 4 A = cot 6 A, then what is the value of angle A. 45. A boat can travel 8 miles per hour in still water. What is it velocity with respect to the
a. 90 b. 120 c. 100 d. 140 shore if it heads 350 East of North?
a. 6,743 b. 8,963 c. 5,400 d. 4,588
46. What is the distance in cm between two vertices of a cube which are farthest from each
other, if an edge measures 8 cm? Degree of Differential Equation:
a. 13.86 b. 16.93 c. 12.32 d. 14.33 - The power to which the highest - order derivative is raised , if the equation is written as a
1 π polynomial in the unknown function .
47. If arctan  = , then the value of x is
3 4
Linearity of a Differential Equation
a. 1/3 b. 1/2 c. 1/4 d. 1/5
48. The energy stored in a stretched elastic material such as a spring is - A differential equation is linear if it has the form:
a. mechanical energy b. elastic potential energy
c. internal energy d. kinetic energy Pn(x) dny + Pn-1 (x) dn-1y +……+ P1(x) dy + Po(x)y = Q(x)
49. Given the points (3, 7) and (-4, -7). Solve for the distance between them. dxn dxn-1 dx
a. 15.65 b. 17.65 c. 16.65 d. 14.65 where: y – the unknown function
50. What rate of interest compounded annually is the same as the rate of interest of 8% x – the independent variable
compounded quarterly?
a. 8.48% b. 8.42% c. 8.24% d. 8.86% Q(x) , Pn(x), Pn-1(x)…P0(x) - presumed known and depend only on the variable x

ANSWERS
1A 2A 3A 4B 5C 6C 7A 8 A 9 D 10 C 11D 12D 13D 14C 15C 16C 17B 18A 19D 20B * Differential equations that cannot be reduced or put in this form are nonlinear.

21C 22D 23B 24B 25B 26C 27C 28D 29D 30D 31A 32C 33D 34A 35A 36B 37D 38B Sample Problems:
Determine the order, degree, linearity, unknown function, and independent variable of
39A 40C 41C 42C 43B 44C 45D 46A 47B 48B 49A 50C the differential equation.
1. y’’’ – 2xy ‘ = ln x +2
2. 5x (d2y / dx2 ) 9x3 (dy / dx) tanx (y) = 0
Differential Equation - an equation involving differential coefficient or differentials.
3. (d2b / dp2 ) + p (db / dp)2 = 0
Consider the equation : Answers:
1. third order , first degree, linear (P 3(x) =1 , P1(x) = -2x , P2(x ) = P0 (x) =0 , Q(x) =
(d2y / dx ) + P (x) =Q (x ) lnx +2, the unknown function is y, and the independent variable is x.
2. second order , first degree, linear, unknown function is y, and the independent
* When an equation involves one or more derivatives with respect to a particular variable, variable is x.
that variable is called independent variable. For the given equation, the independent variable 3. second order , second degree , nonlinear ( one of the derivatives is raised
is x. to a power other than the first) unknown function is b and its derivatives, and
the independent variable is p.
* If a derivative of a variable occurs, that variable is a dependent variable. For the given
equation; dependent variable is x. Supplementary Problems:
Determine a)order b) degree c) linearity d) unknown function and e)independent variable for
Types of Differential Equation: the following differential equations.

1. Ordinary Differential Equation – an equation which all differential coefficient have 1. (y’’’)3 - 5x(y’)2 = e-x + 1
reference to a single independent variable.
2. 5y d2z / dy + 3y2 (dz/dy) - (siny) z = 0
2. Partial Differential Equation – an equation which there are two or more independent
variables and partial differential coefficients with respect to any of them. 3. (d4x / dy4)5 + 7(dx / dy)10 + x7 –x5 = y

Order of Differential Equation: 4. ty’’ + t2y’ - (sin t ) √ y = t2 – t + 1


- The order of the highest derivative appearing in the equation.
Families of Curves:
Families of curves may be represented by equation involving parameters. If the 2. Parabolas with foci at the origin and axis Ox.
constants of this equation is treated as an arbitrary constant, the result is called differential Ans. y( y’’)2 + 2xy’ –y =0
equation of the family represented by equation.
3. Circles tangent to the x-axis.
Sample Problems : Ans. [ 1+ (y’ )2 ]3 = [ yy ’’ +1 + (y’)2 ]2
1. Obtain the differential equation of the family of:
4. All tangents to the parabola y2 =2x
a). straight lines with slope and y- intercept equal.
Ans. 2x (y’ )2 –2yy’ + 1 = 0
Solution:
Solution of Differential Equations
y = mx +b - the slope intercept form
A function y = f(x) is a solution of differential equation if it is identically satisfied when
In these case, m = b
y and its derivatives are replaced throughout by f(x) and its corresponding derivatives.
y = mx+m - the constant to be eliminated A differential equation of order n will, in general, possesses a solution involving n
arbitrary constants. This solution is called general solution. It will be necessary to assign
differentiating dy = m
specific values to these arbitrary constants in order to meet the prescribed initial conditions.
dy/dx = m substituting to the original equation
First Order Equations
y = (dy / dx) (x+1) Answer
1. First Order : Variables Separable
b). circles with center on the x-axis A first order differential equation can be solved by integration if it is possible to collect all
y terms with dy and all x terms with dx. That is, if it is possible to write it in the form:
y f(y) dy + g(x) dx = 0 ,
Then the general solution is
∫ f(y) dy + ∫ g(x) dx = C
x where C is an arbitrary constant.

Sample Problems:
1. Find the general solution of the following differential equation
a. y’ =(x+1) / y
b. ds/dt = s2 + 2s + 2
Solution: 2. Obtain a particular solution which satisfies the given initial condition.
C( h,0) the arbitrary constants are h and r, therefore we must
dy/dx = 3x3 / √ y , y = 4 when x = 1
(x-h)2 + y2 = r2 differentiate the equation two times.
2(x-h) + 2yy’ = 0 first differentiation
(x-h) + yy ‘ = 0 Solution:
x+yy’ = h 1. a). The equation maybe written in the form
1+yy’’+ (y ‘)2 = 0 Answer dy/dx = (x+1)/y
ydy = (x+1) dx
The solution is
Supplementary Problems:
For each of the following, obtain the equation of the family of plane curves. ∫ydy = ∫ (x+1) dx
½ y2 = ½ x 2 + x + C
1. Straight lines whose distance is a from the origin.
Ans. (xy’ –y ) 2 = a2 ( 1+ (y’)2) y2 – x2 – 2x = C Ans.
b. Separate the variables to obtain b. dy/dx = sec y tan x x=0, y=0 Ans. sin y + ln cos x = 0
ds
= dt
s2 + 2s + 2 2. First Order : Homogeneous Equation
which is the same as Polynomials in which all terms are of the same degree such as x 2 + y2 and x2 sin (y/x)
ds are called homogeneous polynomial.
= dt
(s + 1)2 + 1 Equations with homogeneous coefficient.
The solution is Consider
ds M (x,y) dx + N (x,y) dy = 0  1
∫ = ∫ dt If the coefficients M and N are of the same degree in x,y , they are called homogeneous
(s + 1)2 + 1 functions.
C + t = arctan (s+1)
Let y = Vx, and dy = Vdx + xdV, substitute this to equation 1 to make the variables separable.
s = tan (C+t) – 1 Answer
Sample Problem:
2. The variables are separable, which can be written in the form; 1. Solve y’ = (y + x )/ x
√ y dy = 3x3 dx Solution: Let y = Vx; dy = Vdx + xdV
integrating both sides dy/dx = (y + x)/ x = (Vdx + xdV ) / dx = (Vx + x) / x
∫ √ y dy = ∫ 3x dx 3
or simply
2
/3 y3/2 = ¾ x4 + C  this is the general solution x (dV/dx) = 1
if y = 4, x = 1 V = ln x + C
C = 2/3 (4)3/2 – ¾ (1)4 V = ln C x *
C = 55/12 But v = y/x
2
/3 y 3/2 4 55
= ¾ x + /12 Therefore, y = x ln C x
or simply * The ln of a constant is still a constant.
8 y3/2 – 9 x4 = 55 Answer Supplementary Problems:
Find the general solution:
Supplementary Problems:
1. Obtain the general solution of the following: 1. y’ = 3xy / y 2 – x2 Ans. (y2 + 2x2)3 = Cy2
a. xy’ + y = 0 Ans. xy + C = 0
b. x dx = y dy = 0 Ans. x2 + y2 = C 2. y’ = (x2 + y2 ) / 4xy Ans. 3y2 - x2 = C√ x
c. x3 dx + (y + 1)2 dy = 0 Ans. (y + 1)3 + ¾ x3 = C
3. y’ = y / ( x + √ xy) Ans. – 2 √ x/y + ln y = C
d. ds/dt = t2 / s2 + 6s + 9 Ans. (s + 3)3 – t3 = C
e. y dy + (y2 + 1) dx Ans. ln (1 + y2) + 2x 3. First Order : Linear Equation
A differential equation of first order, which is also linear, can be written in the form:

2. Obtain the particular solution satisfying the given initial conditions. dy/dx + P(x)y =Q (x) 1
a. dy/dx + 2y = 3 x=0, y=1 Ans. ln (3 – 2y) + 3x = 0
To obtain the general solution we must find a function ∅ = ∅ (x) such that if the equation Integrating factor φ = e∫Pr (x) dx
is multiplied by this function derivative of the product ∅y. This function is called the Where: P(r)(x) = (1-n) P(x) Reduced polynomial in x
integrating factor ∅. Q(r)(x) = (1-n) Q(x)
∅ = e ∫Pdx z = y1-n
And the solution is General Solution: zφ = ∫ φ Qr (x)dx + c

y e∫Pdx = ∫ Q e∫Pdx dx + C  2 Sample Problem:


Solve dy / dx – 2y / x = 4x3 y3
Sample Problem: Solution:
1. Obtain the general solution of the differential equation. dy / dx + y (-2/x) = y3 (4x3)
x dy + y dx = sin x dx
Solution: n=3
The differential equation must be first reduced in the form of equation 1, hence, z = y1-3 z = y –2
dy/dx + y/x = (sin x) / x P(x) = -2 / x
P(x) = 1/x ; Q(x) = (sin x) / x Pr (x) = (1-n) Px
Solving for the integrating factor, Pr (x) = (1-3) Px
∅=e ∫dx/x
Pr (x) = -2 (-2/x) = 4/x
∅ = eln x Q (x) = 4x3

∅=x Qr (x) = (1-n) Q(x)


Qr (x) = (1-3) (4x3)
using the formula,
Qr (x) = -8x3
yx = ∫ sinx /x (x) dx + C
φ = e ∫Pr (x) dx = e ∫ (4/x) dx + e 4 lnx = x4
simplifying,
General solution:
xy + cos x = C Answer
zφ = ∫ φ Qr (x)dx + c
(y-2)(x4) = ∫ x4 (-8x3) dx + c
Supplementary Problems:
Obtain the general solution: y-2x4 = -8∫ x7 dx + c
1. dy/dx + x3y = 0 Ans. y = c e (-x^4)/4
y-2x4 = - x8 + c
2. e-2ydx + 2 (xe2y – y) dy Ans. x e2y = y2 + C x4 = (c –x8) y2
3. dT/dθ = cos θ + y cot θ Ans. T = c sin θ - cos θ
4. First Order: Exact equations
4. y’ = x – 2y cot 2x Ans. 4y sin 2x = c + sin 2x – 2x cos 2x Any equation that can be written in the form
5. y’ = x3 2xy; when x = 1, y = 1 Ans. 2y = x2 – 1 + 2 e(1-x²)
M(x,y)dx + N(x,y)dy = 0

3. Bernoulli’s Equation And we have the property


Standard Form:
dx/dy + y P(x) = yn Q(x) ∂M ∂N
provided, n ≠ 0, 1 ------- = ------- is said to be an exact equation.
∂y ∂x k < 0 – exponential decay
Sample Problem:
The technique is to find a function f(x,y) such that 1. A certain radioactive material is known to decay at a rate proportional to the amount
present. If initially there is 50 mg of the material present and after 2 hrs. it is observed that
f’(y) – the common term of f(x,y) and N(x,y) the material has lost 10% of its original mass. Determine mass of the material after 4 hours?
g’(y) – the common term of f(x,y) and M(x,y)
Solution:
Sample Problem: Given: Qo = 50 mg
Determine whether the equation 2xy dx + (1+x2) dy = 0 is exact. If so, obtain the general
solution. Q(2) = (50 mg) (1-0.1)
Solution: Q(2) = 45 mg
Given in the equation that
Solving for k,
M = 2xy, N = 1+ x2
45 = 50 e2k
∂M / ∂y = ∂N / ∂x = 2x ∴ The equation is exact
k = -0.053
F = ∫ 2xy dx + f(y)
∴ amount of material at any time t
F = x2y + f(y)
Q = 50 e -0.053 t
∂F/ ∂y = x + f’(y) = x + 1
2 2
In the problem t = 4 hrs.
f’(y) = 1
Q = 50 e -0.053 t
f(y) = y + c
Q = 40.5 mg. Answer
x2y + y + c = 0 Answer
2. Newton’s Law of Cooling
Supplementary Problems:
The time rate of change of the temperature of a body is proportional to the
1. (2x ey + ex) dx + (x2 +1) ey dy = 0 Ans. ex + ey(x2 +1) = c
temperature difference between body and its surrounding medium.
2. (2xy + y2) dx + (x2 +2xy - y) dy = 0 Ans. 2x2y +2xy2 – y2 = c T = temperature of the body at any time t
3. (x + sin y) dx + (x cos y – 2y) dy = 0 Ans. ½ x2 + x sin y – y2 = c To = initial temperature
4. dy + (y- sin x) / x dx = 0 Ans. xy + cos x = c Tm = temperature of the surrounding medium
dT / dt = time rate of change of the temp. of the body
Elementary Applications: dT / dt = - k (T – Tm)
1. Law of Exponential Change (Growth /Decay)
(dT / dt) + kT = kTm , k is always positive, dT / dt = negative for cooling
The rate of which the amount of a substance changes, is proportional to the amount
present or remaining at any instant. T = Ce-kt + Tm or T = (To – Tm) e-kt + Tm

Q = amount of substance at any time t


Sample Problem:
dQ / dt = rate of change of the amount Q A body at a temperature off 36oC is placed outdoors where the temperature is 60oC. If
after 5 mm, the temperature of the body is 40 oC. How long will it take to reach a temperature
dQ / dt ∝ Q of 45oC?
dQ / dt = kQ Solution:
Given: Tm = 60oC
Q = Qo ekt k > 0 – exponential growth
To = 36oC into the tank at Ri = (gal/min. , liter/min.) while simultaneously, the well-stirred
o solution leaves the tank at the rate R o ( gal/min.)
T @ 5 mm = 40 C
Solve for k: Ci , Ri
40 = (36-60) e –k(5)
+ 60 ds/dt = Ci Ri - Co Ro
Co = S / [Vo + (Ri , Ro) t]
k = 0.03646 Mixture
Outgoing
o In general, dS/dtVo=,SSoo / [Vo + (Ri , Ro) t] = Ci Ri Ri > Ro , then Vo will overflow
Solving for t at T = 45 C
RCi =o ,RRo o, Vo = c
45oC = (36-60) e –0.03646 t + 60 Ro < RI , then Vo↓
t = 12.9 min. Answer Let S = amount of substance present in the tank
dS/dt = rate of change of substance S
3. Simple Chemical Conversion
In certain reactions in which a substance A is being converted into another substance Sample Problem:
the time of change of the amount x of unconverted substance is proportional to x. A 50 gal. tank contains 10 gal. of fresh water. At t = 0, a brine solution containing 1 lb.
Of salt per gallon is poured into the tank at the rate of 4 gal/min. , while the well-stirred
Let x = xo the uncovered substance at to = 0, then the amount x at any time t > 0, is mixture leaves the tank at the rate of 2 gal/min,a.) find the amount of time required for
given by the differential equation as dx/dt = - kx  ← overflow to occur, b.) Find the amount of salt in the tank at the amount of overflow.

The proportionality is chosen to be –k because x is decreasing as time increases, Solution:


from ← x = ce-kt but x = xo at t = 0, hence At t = 0, Vo = 0 , ci = 1, Ri = 4,Ro = 2
xo= ce-k(1) c = xo
X = xoe-kt The volume of brine in the tank at any time t equals to
Sample Problem:
Suppose that a chemical reaction proceed such that the time rate of change of the Vo + Ri t - Ro t = 10 + 4t – 2t
unconverted substance is proportional to the amount of it. If half of substance A has been = 10+2t
converted at the end of 10 sec. Find when 9/10 of the substance will have been converted. overflow will occur if the volume of the brine in the tank equals to the volume of tank
now.
Solution: T = ½ (50-10) = 20 min.
Given: x = ½ xo
b.) dQ/dt + Q/10+2t = 4
½ xo = xo e k(-10)
dQ/dt + 2Q/10+2t = 4
k = 0.069
integrating factor = e ∫ (dt / 5+t) = e ln (5+t) = 5+t
when 9/10 is converted only 1/10 is unconverted
Q (5+t) = 4 ∫ (5+t) dt
∴ x = 1/10 x
1 Q (5+t) = 4 [ 5t +1/2 t2] + c
/10 xo = xo e –0.069 t
Q = 20t +12 t2 + c / 5+t
solving for t
At t = 0 Q = a = 0
t = 33 sec. Answer
0 = [20(0) + 2(0)2 + c] / (5+t)
c = 0; now
4. Dilution / Flow Problem:
The tank initially hold Vo (gal., liter,etc.) of solution that contains S o (lb.,kg.,N) of a Q = 20t + 2 t2 / 5+t
substance. Another solution containing a substance at C I = lb/gal.; N / L is poured
Req’d: Q =? At t = 20 min.
Q = [20(20) + 2(20)2 / (5+20) 5. If a population of a country doubles in 50 years, in how many years will it treble under the
assumption that the rate of increase is proportional to the no. of inhabitants? Ans. 79
Q = 48 lb. Answer
6. A cylindrical tank contains 40 gal. of a salt solution containing 2 lb. of salt per gallon. A
salt solution of concentration 3 lb/gal flows into the tank at 4 gal/min. How much salt is in
the tank at any time t if the well-stirred solution flows at 4 gal/min? Ans. 120 - 40e –t/10
5. Orthogonal Trajectories
7. An inductance of 1 henry and a resistance of 2 ohms are connected in series with an emf
Given a family of curves given by f(x,y,c) = 0
of 100e-t volts. If the current is initially zero, what is the maximum current attained? Ans.
The curves that intersects a family f(x,y,c) = 0 at right angles, whenever they do
25A
intersect is given by the family of another curve g(x,y,k) and are called orthogonal
8. Find the orthogonal trajectories of x2=cy. Ans. 2y2+x2=c
trajectories. This two curves are said to be orthogonal to each other, because each
point of intersection, the slopes of the curves are negative reciprocals to each other.
DIFFERENTIAL CALCULUS
Let Mdx + Ndy = 0 where M and n are f(x,y)
dy/dx = - M/N LIMIT OF FUNCTIONS
Now the DE of the orthogonal trajectories is dy/dx = N/M Definitions
Sample Problem: • The limit of a function of x or f(x) as x approaches a is L; written as
Find the orthogonal trajectories of all circles whose center is at the origin. lim f(x) = L
x→a

• lim f(x) = L if and only if, for any chosen positive number ∈,
Solution: x→a
however small, there exists a positive number δ such that, whenever 0 <
x2 + y2 = c2 x-a < δ , then ƒ(x) – L  < ∈

Differentiating implicitly, 2xdx + 2ydy = 0 Theorems on Limits


Simplifying 1. Limit of a constant c
lim c = c
dy/dx = - x/y = - M/N x→a

∴ N/M = dy/dx  the D. E. of trajectories 2. Limit of a variable x


lim x = a
∫ y/dy = ∫ x/dx then integrating x→a

3. Limit of sum of two functions


y = kx Answer
lim [ ƒ(x) + g(x) ] = lim ƒ(x) + lim g(x)
x→a x→a x→a

Supplementary Problems: 4. Limit of product of two functions


1. A body at an unknown temperature is placed in a room which is held at a constant lim [ ƒ(x) g(x) ] = lim ƒ(x) [ lim g(x) ]
temperature of 30oC and after 10 min., the temperature of the body is 0 oC and after 20 x→a x→a x→a
min., the temperature of the body is 15oC. Find the expression for the temperature at any
5. Limit of a quotient
time t. Ans. T = - 60 e –0.069t + 30
2. At exactly 10:30 pm, a body at a temperature of 50 oF is placed In an oven whose lim ƒ(x) = lim ƒ(x)
x→a x→a
temperature is kept at 150oF, If at 10:40, the temperature of the body is 75 oF, at what g(x)
time will it reach 100oF. Ans.10:54 p.m. lim g(x)
x→a
3. Find the required for a radioactive substance to disintegrate half of its original mass if
three quarters of it are present after 8 hrs. Ans. 19.3 hrs. 6. Limit of a radical
4. After 2 days, log of a radioactive material is present. Three days later, 5g. is present. lim n √ ƒ(x) = n√ lim ƒ(x)
How much of the chemical was present initially, assuming the rate of disintegration is x→a x→a

proportional to the amount present. Ans. 15.87 g.


Limit to Infinity or Zero INTERMEDIATE FORMS
Given a constant c and variable x A. 0/0 or ∞/∞ ( L’Hospital’s Rule is applicable)
1. lim cx = + ∞ for positive c B. ∞ - ∞ , 0. ∞ ( L’Hospital’s Rule is not directly applicable)
x→∞
= - ∞ for negative c C. 00, ∞0, 1∞
2. lim c/x = 0 Note:
x→∞
If the evaluated function turned out to be in the form of those in B and C, change the
3. lim /c = + ∞ for positive c
x
form of the given function to obtain an evaluated function in the form of that in A.
x→∞
= - ∞ for negative c
4. 4. lim c/x = + ∞ for positive c • The Indeterminate Form 0. ∞
x→0
= - ∞ for negative c If f(x) approaches zero and g(x) approaches infinity as x approaches a (or x
→ ± ∞) , the product ƒ(a). g(a) is undefined and will be of the form 0. ∞.
L’Hospitals Rule If the limit f(x) . g(x) exists as x → a (or as x → ± ∞), it may be found by writing the
If the functions f(x) and g(x) are continuous in an interval containing product as a fraction
x = a, and if their derivatives exist and g’(x) ≠ 0 in this interval (except possibly at x = a), then lim ƒ(x) g(x) = lim ƒ(x) = lim g(x)
when f(a) = 0 and g(a) = 0 x→a x→a 1
/g(x) x→a 1
/f(x)
lim f(x) = lim f ’(x) Then apply L’Hospital’s Rule
x→a x→a
g(x) g ’(x) • The Indeterminate Form ∞ - ∞
Provided that the limit on the right side exists. If ƒ(x) and g(x) both increase without bound as x → a (or x → ± ∞), the difference
ƒ(a) – g(a) is undefined and will be of the form ∞ - ∞
Evaluation of Limits If the limit of ƒ(x) – g(x) exists as x → a (or x → ± ∞), then by algebraic means.
Let lim N(x) = L
x→a
D(x) lim [ ƒ(x) – g(x)] = lim 1
/g(x) - 1/f(x)__
where N (x) and D (x) are polynomials in x x→a x→a
1___
a = any real number L = limit g(x) f(x)

Methods of Evaluation • The form 0 , ∞ , 1


0 0 ∞

If the limit of ƒ(x)g(x) exists as x → a (or as x → ± ∞), then by logarithm,


1. direct substitution ( obvious limit )
2. rationalizing N (x) or D (x) lim ƒ(x)g(x) =y can be evaluated
x→a
3. expanding N(x) or D (x)
4. combining terms in N (x) or D (x) Let L = lim ƒ(x)g(x)
5. factoring N (x) or D (x) x→a

6. applying L’Hospital’s Rule Taking the logarithm of both sides


ln L = ln lim ƒ(x)g(x) = lim ƒ(x)g(x) lim g(x) ln ƒ(x) = k
Limits to Infinity of a Fraction N(x) x→a x→a x→a
D(x) lnL k
In L = k , then e = e or L = ek
Let lim N(x) = L
x→∞
D(x) Supplementary Problems
1. If the degree of the numerator N (x) is less than the degree of denominator D (x),
then L = 0. Evaluate the limits given below
2. If the degree of N (x) equals the degree of D (x) then 1. lim sin 5x
x→0
L = coefficient of N (x) with highest degree x Ans: 5
coefficient of D (x) with highest degree 2. lim 1 – cos θ
θ →∞
3. if the degree of N (x) > D (x), then L = ∞ 2θ Ans: 0
3. lim (1 + 1/x)3x
x→∞
Ans: e3 Slope of tangent line
1
4. lim /3 – /x 1 =- 1
x→3
x–3 Ans: 1/9 dy/dx
5. lim sinθ =- 1
f’(xo)
θ →0
θ Ans: 1
ANGLE BETWEEN TWO CURVES (Angle of Intersection) – is defined as the angle between
6. lim (1/x – 1/3sinx)
x→0
their tangents at their point of intersection.
Ans: 0 To determine the angle of intersection of two curves, f(x) and g(x)
7. 7. lim (x+1) ln x 1. Solve the equations simultaneously to find the points of intersection.
x→0
Ans: 1 2. Find the slopes m1 = f’ (xo) and m2 = g’ (xo)
8. 8. lim x csc 5x
x→0
Ans: 1/5 Then the acute angle θ of intersection is given by
9. ECE Board exam 1987
Evaluate lim x3 – 2x + 9
x→∞
2x3 – 8 Ans: ½ m1 – m 2
Tan θ = ---------------
10. ECE Board Exam 1987 1 + m1m2
Evaluate lim 2x4 – 2x3 + 9x2 – x + 7
x→∞
x3 – 8 Ans: ∞
Interpretation of Derivative 2. Derivative as Rate of Change
1. Derivative as Slope dy /dx = lim ∆y / ∆x
x→0

y The value of the derivative of the function is the instantaneous rate of change of the
y=ƒ(x) function with respect to the independent variable.

Tangent Line Rectilinear Motion – motion in a straight line


Assumption:
1. Motion will always be assumed to take place along straight line, although the
object in motion may go either direction.
Po(xo,yo) 2. The body in motion is idealized to be a point or a particle.

Let S = displacement of a particle at any time t


θ • Speed (v) – of a particle moving along a curve is the absolute value of the time
0 x rate of change of the displacement (or distance), measured along the curve, of
the point with reference to some fixed point on the curve.
Definitions V = ds/ dt
 The tangent to the curve with equation y = f(x) at Po(xo, yo) is the line through  Acceleration (a) – of a moving point is the time rate of change of the velocity of
Po(xo, yo) with slope f’(xo) the point
Slope of Tangent Line = tan θ = dy /dx = f’(xo) a = dv/ dt = d2s/dt2

The normal to the curve with equation y = f(x) at Po(xo, yo) is the line through Po Differentiation Formulas
which is perpendicular to the tangent line at Po Let u, v be any functions of x
n = any integer
Slope of Normal Line = 1 C= any constant
I. Basic Formulas d du
4. (cotu) = −csc 2 u
d dx dx
1. (c) = 0
dx d du
5. (secu) = sec u tan u
d dx dx
2. (x) = 1
dx d du
6. (cscu) = −cscucotu
d dx dx
3. (x n ) = nx n-1
dx
d du III. Differentiation of Inverse Trigonometric Function
4. (cu) = c d 1 du
dx dx (Arcsinu) =
1.
d du dv dx 1 − u dx
2
5. (u + v) = +
dx dx dx d - 1 du
d udv vdu 2. (Arcosu) =
dx 1 − u 2 dx
6. (uv) = +
dx dx dx d 1 du
du dv 3. (Arctanu) =
v − u dx 1 + u 2 dx
7. d dx dx d − 1 du
(u / v) = 4. (Arcotu) =
dx v2 dx 1 + u 2 dx
du d 1 du
d −k 5. (Arcsecu) =
8. dx dx u u − 1 dx
(k / u) = 2
2
dx u d -1 du
d n du 6. (Arcscu) =
9. (u ) = nu n −1 dx u u 2 − 1 dx
dx dx
d 1 du
10. ( u) =
dx 2 u dx IV. Differentiation of Logarithmic Functions
d - n du d 1 du
11. (1/ u n ) = n+1 1. (lnu) =
dx u dx dx u dx
dy dy du d 1 du
12. = • (The Chain Rule) 2. (logu) = (log e)
dx dx dx dx u dx
dy 1 d log b e du
13. = 3. (log b u) =
dx dx / dy dx u dx
II. Differentiation of Trigonometric Function V. Differentiation of Exponential Functions
d du
1. (sinu) = cosu d u du
dx dx 1. (e ) = e u
dx dx
d du
2. (cosu) = -sinu d u du
dx dx 2. (a ) = a u (ln a )
dx dx
d 2 du
3. (tanu) = sec u d v du dv
dx dx 3. (u ) = u v + lnu
dx dx dx
VI. Differentiation of Hyperbolic Function 6. The tangent to y = x3 – 6x2 + 8x at (3, -3) intersects the curve at another point. Determine
d du this point. Ans: P(0, 0)
1. (sinhu) = coshu 7. For the curve y = x2 + x, at what point does the normal line at (0, 0) intersect the tangent
dx dx line at (1, 2)? Ans: P(3/10, -1/10)
d du 8. Find the angle of intersection between the curves y = x 2 + 2 and
2. (coshu) = sinhu
dx dx y = x + x-1 + 1. Ans: θ = 63.40
d du 9. A boy 3 ft. tall walks away from alight which is on top of post 7.5 ft. high. Find the rate of
3. (tanhu) = sec 2 u change of the length of his shadow with respect to his distance from the lamp post. Ans:
dx dx
2/3 ft/ft
d du 10. Find the rate of change the area of an equilateral triangle with respect to the side of this
4. (cothu) = −csch 2 u
dx dx triangle when the latter is 2 ft. Ans: √3 ft2/ft.
d du 11. If the particle moves according to the law S = t 2 – t3 + 3, find the velocity when
5. (sechu) = −sechutanhu acceleration is zero.Ans: -1/3
dx dx
d du 12. If the motion of the body is described by S = 3t 5 – 30t2 + 5, when will the acceleration be
6. (cschu) = −cschucothu zero? Ans: 1
dx dx 13. Two particles have position at time t given by the equation S 1=t3 + 6t2 – 7t + 1 and S2 = 2t3
– 3t2 – t + Find their position when they have the same acceleration. Ans: S1 = 61; S2 =
VII. Differentiation of Inverse Hyperbolic Function 25
d 1 du 14. Find y’ and y”, given x3y + xy3 = 2 and x = 1. Ans: y’ = -1;y” = 0
1. (sinh −1u) = 15. Find the equation of the tangent and normal to x2 + 3xy + y2 = 5 at (1, 1).
dx u + 1 dx
2
Ans: Tangent: x + y – 2 = 0, Normal: x = y
d 1 du
2. (cosh −1u) = Other Applications of Derivatives
dx u − 1 dx
2
Increasing and Decreasing Functions:
d 1 du  A function y = f(x) is said to be increasing if its value increases as y increases
3. (tanh −1u) =  A function y = f(x) is said to be decreasing if y decreases as x increases
dx 1 − u2 dx
Given a function f(x) differentiable in the interval a ≤ x ≤ b
d -1 du
4. (coth −1u) = 1. If f’(x) > 0, then f(x) is increasing
dx 1 − u2 dx 2. If f’(x) < 0, then f(x) is decreasing
d -1 du 3. If f’(x) = 0, then f(x) is stationary
5. (sech −1u) = Concavity, Critical Points, Inflection Points
dx u 1 − u 2 dx
 Concave Upward
The graph of a function is said to be concave upward if the function is decreasing
d -1 du then increasing.
6. (csch −1u) =
dx u 1 + u dx
2
 Concave Downward
The graph of a function is concave downward if the function is increasing then
Supplementary Problems: decreasing.
1. Find the slope of the curve y = x + 2x-2 at point (1, 2).Ans: -3  Maximum Point
2. Find the point on the curve y = x2 – 6x + 3 where the tangent is horizontal. Ans: P(3, -6) A point where the function from increasing to decreasing and the function is said to
3. At what point on the curve y = x4 + 1 is the normal line parallel to have a relative minimum value.
2x + y = 5?Ans: P(1/2, 17/16)  Critical Point
4. Find the point on the curve y = 7x – 3x2 + 2 where the inclination of the tangent is The point at which y’ = 0 and value of x at this point critical value.
450.Ans: P(1, 6)  Inflection Point
5. Find the point where the normal to y = x + x1/2 + 1 at (4, 7) crosses the A point at which the curve changes its direction of concavity.
y- axis. Ans: P(0,10 1/5) First Derivative Test
Substitute in the expression for the first derivative a value slightly less than and then a value 5. Determine a, b, c and d so that the curve y = ax 3 + bx2 + cx + d will have horizontal
slightly greater than the critical value under consideration. tangents at the points (1, 2) and (2, 3)
1. If f’(x) changes from positive to negative as x increases through the critical value, 6. Find the equation of the line normal to the curve y = 3x 5 – 10x3 + 15x + 3 at its point of
then the critical is a maximum point. inflection. Ans: x + 15y – 45 = 0
2. If f’(x) changes from negative to positive as x increases through the critical value, 7. Find a such that the curve y = 2x 3 – 3ax2 + 12x – 1 will have are of its critical points where
then the critical is a minimum point. x = 2 Ans: a = 3
3. If f’ (x) does not changes sign, the critical is neither a maximum or a minimum point. 8. Find the all values of x where the curve y = x2 – 2x + 5 is increasing. Ans: x > 1
It is the point of inflection with horizontal tangent.
MAXIMA AND MINIMA APPLICATIONS
Step in Solving Problems Involving Maxima and Minima
maximum point 1. Identify the quantity to be maximized or minimized
y’= 0 2. Use the information in the problem to eliminate all quantities so as to have a function
variables. Determine the possible domain of this function.
3. Differentiate this function with respect to the variable whose maximum/minimum value is
Inflection y = f(x) to be determined
Point 4. Equate the derivative of step 4 to zero and solve for the unknown.

Supplementary Problem
y’=0 1. ECE Board Exam 1973
minimum point An open rectangular tank with square base is to have a volume of 10 cu. m and the
material for the bottom is to cost per square meter and that for the sides 6 cents per
square meter. Find the height of the tank if the coil of making the tank is to be a
x=a minimum. Ans: 2.5 cm.
concave concave 2. ECE Board Exam Feb. 1973
downward upward In problem no. 1 above, find the most economical dimension for the tank. Ans: 2m x 2m x
2.5m

3. ECE Board Exam 1981


Second Derivative Test Divide 60 into two parts so that the product of one part and the square of the other is a
1. The function y = f(x) has a maximum value at x = a if f’(a) = 0 and f”(a) < 0. The maximum. Ans: 20, 40
curve is concave downward at that point. 4. ECE Board Exam April 1988
2. The function y = f(x) has a minimum value at x = a if f’(a) = 0 and f”(a) > 0. The curve Find the altitude of the largest circular cylinder that can be inscribed in a circular cone of
is concave upward at that point. radius r and height h. Ans: 1/3 h
3. If at f’(a) = 0, f”(a), then his test fails. Use the first derivative test. 5. ECE Board Exam 1989
Find the greatest volume of a right circular cylinder that can be inscribe in a sphere of
Third Derivative Test radius r. Ans: r = 2.418 r3
A function y = f(x) has an inflection point at x = a if f”(a) = 0 and f”’(a) ≠ 0. 6. ECE Board Exam Nov. 1995
Find the radius of a right circular cylinder that can be inscribe in a cone of a radius R and
Supplementary Problems height H. Ans: r = 2/3 R
1. Find the maximum, minimum and inflection point of the curve 7. ECE Board Exam Feb. 1978
y=-x3 + 2x2 – x +2 Ans:Max.(1, 2); Min. (1/3, 50/27); Inflection (2/3, 52/27) The sum of the two numbers is 36. What are these numbers if their product is to be the
2. Find the value of k such that the curve y = x3 – 3kx2 + 5x – 10 Ans: k = 1 largest possible. Ans: 18 and 18
3. Determine the equation of the parabola y = ax2 + bx + c passing through (2, 1) and be 8. ECE Board Exam Feb. 1978
tangent to the line y = 2x + 4 at point (1, 6) Ans: y = -7x2 + 16x –3 A square sheet of galvanized iron 100cm x 100cm will be used in making an open top
4. What curve of the form y = ax3 + bx2 + cx + d will have critical points at (0, 4) and (2, 0). container by cutting a small square from each corners and bending up the sides.
Ans: y = x3 – 3x2 + 4
Determine how large the square should be cut from each corner in order to obtain the A baseball diamond is a square, 27 m on each side. The instant a runner is halfway from
largest possible volume. Ans: 16.67cm x 16.67 home to first base, he is giving towards first base at 9 m/s. How fast is his distance from
9. ECE Board Exam, Mar. 1989 the second base changing at this instant?
A rectangular field containing a given area is to be fenced off along a straight river. If Ans: -4.025 m/s
no fencing is needed along the river, what should be the dimension of the field so that 2. ECE Board Exam , Sept. 1983
least amount of fencing materials will be used? Ans: L = 2W. A boat is being towed to a pier. The pier is 20 ft. above the boat. The remaining length of
10. ECE Board Exam, Nov. 1989 the rope to be pulled is 25 ft. It is being puled at 6 ft. per second. How fast does the boat
Find the minimum volume of a right circular cylinder that can be inscribe in a sphere approaches the pier? Ans: 10 ft/s
having a radius equal to r. 3. ECE Board Exam, March 1981
11. EE Board Exam , April 1981 Given a conical funnel of radius 5 cm and height 15 cm. The volume is decreasing at the
A telephone company agrees to put up a new exchange for 100 subscribers or less rate of 15 cu. cm/s. Find the rate of change in height when the water is 5 cm from the top.
at a uniform change of P40 each. To encourage more subscriber the company agrees to Ans: 0.43 cm/s
deduct 20 centavos from their uniform rate for each subscriber in excess of 100. If the 4. ECE Board Exam, April 1998 / ECE Board Exam, Oct. 1985
cost to serve each subscriber is P 14, what number of subscriber would give the Sand is pouring from a hole at the rate of 25 cu, ft. per second and is forming a conical
telephone company the maximum net income. Ans: 115 pile on the ground. If the conical formation has an altitude always ¼ of the diameter of the
12. Find the equation of the tangent line to the curve base, how fast is the altitude increasing when the conical pile is 5 ft. high? Ans: 1/12.75
y = x3 – 3x2 + 5x = 2 that has the least slope Ans: 2x – y + 3 = 0 ft/s increasing
5. ECE Board Exam, Feb. 1978
13. Find the area of the largest rectangle with sides parallel to the coordinate axes which A helicopter is rising vertically from the ground at a constant rate of 4.5 m/s. When it is 75
can be inscribe in the bounded by x2 = 28 – 4 and x2 = y – 4. m off the ground, jeep passed beneath the helicopter travelling in a straight line at a
Ans: 64 sq. units constant speed of 80 kph. Determine how fast the distance between them changing after
14. An isosceles trapezoid is 6 cm long on each side. How long must be the longest side if 1 second? Ans: 10.32 m/s (increasing)
the area is maximum. Ans: 12 cm. 6. ECE Board Exam, Feb. 1977
15. Find the dimension of the right circular cone of minimum volume which can be Two boats starts at the same point. One sail due east starting 10 A.M. at a constant rate
circumscribed about a sphere of radius 8 cm. Ans: radius = 8√2 cm ; height = 32 cm. of 20 kph. The other sail due south starting 11 A.M. at a constant rate of 9 kph. How fast
16. Find the dimension of the cylinder of maximum lateral area which can be inscribe in a are they separating at noon?
sphere of radius 6√2 cm. Ans: radius = 6 cm. ; height = 12 cm. Ans: 21.49 kph.
17. Find the ratio of the volume of the right circular cylinder of maximum volume to that of the 7. ECE Board Exam, August 1976
circumscribing cone. Ans: Vcyl/Vcone = 4/9 A dive bomber loss altitude at a rate of 400 mph. How fast is the visible surface of the
18. Find the equation of the line passing through the point (3, 4) which cuts from the first earth decreasing when the bomber is 1 mile high? Ans: 2792
quadrant of a triangle of minimum area. Ans: 4x + 3y – 24 = 0 8. A man lifts a bag of sand to a scaffold 30 m above his head by means of a rope which
19. Find the dimension of the right circular cone of maximum volume which can be inscribe in passes over a pulley on the scaffold. The rope is 60 m long. If he keeps his end of the
a sphere of radius 12 cm. Ans radius=8√2 cm ; height = 16cm. rope horizontal and walks away from beneath the pulley at 4 m/s, how fast is the bag
rising when he id 22.5 m away?Ans: 2.4 m/s
20. Find the area of the largest rectangle that can be inscribe in an ellipse
9. Water is passing through a conical filter 24 cm deep and 16 cm across the top into a
9x2 + 4y2 = 36. Ans: 12π square units.
cylindrical container of radius 6 cm. At what rate is the level of water in the cylinder rising
if when the depth of the water in the filter is 12 cm its level is falling at the rate of 1
cm/min? Ans: 4/9 cm/min
RELATED RATES
10. A particle starts at the origin and travel up the line y = √3 x at a rate of 5 cm/sec. Two
If a quantity x is a function of time t, the time rate of x given expressed as dx/dt.
When two or more time varying quantities are related by an equation, the relation between seconds later, another particle starts at the origin and travels up the line y = x at the rate
their rates of change may be obtained by differentiating both members of the equation with of 10 cm/s. At what rate are they separating 2 seconds after the last particle started?
respect to time t. Ans: 0.37 ft/s
11. A particle travels along a parabola y = 5x2 + x + 3. At what point do its abscissa and
Supplementary Problems: ordinate change at the same rate? Ans: P(0, 3)
1. ECE Board Exam, Sept. 1986
12. At a certain instant the semi major axis and semi minor axis of an ellipse are 12 and 8 C(h,k), Center of Curvature
respectively and the semi major axis is increasing ½ unit each minute. At what rate is the
semi major axis decreasing if the area remains constant? Ans: 1/3 unit/min.
13. A clock hands are 1 and 8/5 inches long respectively. At what rate are the ends Normal
approaching each other when the time is 2’o clock? Ans:0.095 in/min. Circle of
14. An elevated train on a track 30 ft above the ground crosses a street at the rate of 20 ft. θ Curvature
per sec. At the instant that the car approaching at the rate of 30 ft/s and the car are 0 x
separating 1 sec later/ Ans: 2.67 ft/sec
Definition:
Differential Approximation - The curvature K of a curve y = f(x), at any point P(x, y) on it , is the rate of
Approximation of Error change indirection (that is, the angle of inclination - θ of the tangent line per unit
If y = f(x), then dy = f’(x) dx arc length S.
dx = change or error in x
dy = change or error in y K = dθ/ds = lim ∆θ/∆S
dx/x = relative error in x ∆s→0
dy/y = relative error in y
dx/x (100) = percentage error in x
y” - x”
dy/y (100) = percentage error in y
K = -------------------- or K = ----------------------
Supplementary Problems:
[1 + (y’)2]3/2 [1 + (x’) 2]3/2
1. What is the maximum allowable error in the edge of a cube to be used to contain 10
cubic meters if the error in the volume is not to exceed 0.015 cubic meter? Ans: 0.00108
Notes:
 If K > 0, the point P is on the arc that is concave upward
 If K < 0, the point P is on the arc hat is concave downward

2. The semi major axis and semi minor axis of an elliptical plate is measure to be 8 cm and The curvature K is given by:
6 cm respectively. If there is an approximate error of 0.01 cm and .02 cm in measuring
error in computing for the area. Ans: 0.628 g’ h” - g” h’
3. The altitude of a certain circular cone is the same as the radius if the base and is K = ------------------------
measured as 12 cm with a possible error f 0.04 cm. Find approximately the percentage [ (g’)2 + (h)2]3/2
error in the calculated value of the volume. Ans: 1%
4. Find the allowable percentage error in the radius of a circle if the area is to be correct to
within 5%. Ans: 2.5% In Polar form, r = f(θ)
5. What is the percentage error made in the computed surface area of a sphere if the error Where r’ = dr/dθ , r” = d2r / dθ2
made in measuring the radius is 3%.Ans: 6%.

CURVATURE r2 + 2(r’)2 – rr”


K = -------------------------
y tangent [r2 + (r’)2]3/2

y = ƒ(x)
Radius of Curvature
- the radius curvature R for a point P on the curve is the reciprocal of its curvature
at that point
P(x,y) R
R = 1/K
Notes: 1. The hypotenuse of a right triangle is 34 cm. Find the lengths of the two legs if
 If R > 0, the curve is concave upward one leg is 14 cm longer than the other.
 If R <, the curve is concave downward A. 15 and 29 cm.
Circle of Curvature or Oscillating Circle B. 16 and 30 cm.
- The circle of curvature of a curve at a point P on it is the circle of radius R lying C. 17 and 31 cm.
on the concave side of the curve and tangent to it at P D. 18 and 32 cm.
2. The area of a rhombus is 132 sq. m. if its shorter diagonal is 12 m, find the
y longer diagonal.
A. 20 m
B. 22 m
C. 36 m
C D. 28 m
y = ƒ(x) 3. One side of the parallelogram is 10 m and its diagonals are 16 m and 24 m
respectively, find its area.
R
A. 158.7 sq. m
B. 120 sq. m
C. 96 sq. m
P
D. 192 sq. m
x
4. The diameter of two spheres is in the ratio two is to three and the sum of their
volumes is 1260 cu. m. Find the volume of the larger sphere in cu. m.
A. 980
Center of Curvature
B. 972
The center of curvature for a point P(x, y) of a curve y = f(x) is the center C(h, K) of the
C. 960
circle of curvature at P
D. 938
5. The side of the triangle are 5, 7 and 10 respectively. Find the radius of the
y” [1 + (y’) 2] 1 + (y’)2
circumscribed circle.
h = x - ----------------- k = y - --------------
A. 5.39 m
y” y”
B. 6.40
C. 7.20
D. 4.80
EVOLUTE
6. The volume of a sphere is 36π cu. m. The surface area of this sphere in sq. m is:
- The evolute of a curve is the locus of the center of curvature of the given curve.
A. 25π
Supplementary Problems:
B. 42π
1. Find the curvature of ech of the following curves
C.36π
a) y = sin x Ans: -1
D. 54π
b) x2 = 12y at x = 6 Ans: -1
2. Find the point of maximum curvature of the curve y = ln x
Ans: (1/2 √ 2, -1/2 ln 2)
7. How many side does a polygon has if the sum of the interior angles is 2520o?
3. Find the radius of curvature of the curve of x3 + xy2 – 6y2 = 0 at point (3, 3) A. 14
Ans: 5√ 5 B. 16
4. Find the equation of the circle of curvature of the parabola y 2 = 12x at point (3, 6) Ans: (x C. 12
– 15)2 + (y + 6)2 = 288 D. 10
5. Find the center of the curvature of x3 + xy2 – 6y2 = 0 at (3, 3) Ans: C(-7, 8) 8. The first term of an arithmetic progression is 3 and the 15 th term is 45. Find the
sum of the first 15 terms.
A. 260
MATHEMATICS MODULE 4
B. 360 96. Two ferryboats ply back and forth across a river with constant but different
C. 460 speeds, turning at the riverbanks without loss time. They leave opposite shores at the same
D. 560 instant, meet for the first time at 900 meter from one shore and meet for the second time
9. The first term of an arithmetic progression is -2 and the sum of the first 11 terms is 500 meters from the opposite shore. What is the width of the river?
88. The common difference is: A. 2000
A. 4 B. 2200
B .3 C. 2020
C. 2 D. 2002
D. 1 97. How much lead must be added to n alloy which is 50% tin and 25% lead to make
10. An ellipse with major axis 8 and a minor axis 6 is revolved about its minor an alloy which is 60% tin and 20% lead?
axis. Find the volume of the solid revolution. A. 0
A. 201.06 B. 1 kg
B. 150.80 C. 2 kg
C. 1608.50 D. 3 kg
D. 1206.37 98. A pipe can fill up a tank with drain open in 3 hrs. If the pipe runs with the drain
11. Find the eccentricity of an ellipse with major axis 8 and minor 6. open for 1 hr. and then the drain is closed, it ill take 45 more minutes for the pipe to fill up
A. ¾ the tank. If the drain will be closed right at the start of filling how long will it take for the pipe
B.4/2 to fill up the tank?
C. 4/3 A. 1.1 hrs.
D. 0.66 B. 1.125 hrs.
!2. Find the length of the latus rectum of an ellipse with major axis 8 and minor axis C. 1.25 hrs.
6. D. 1.3 hrs.
A. 3.5
B. 4.5 99. Ding can finish the job in 8 hrs. Tito can do it in 5 hrs. If Ding wok for 3 hrs. and
C. 4 then Tito was asked to help him finish it, how long Tito will have to work with Ding?
D. 5 A. 25 hrs
B. 25/13 min
C. 1.923 hrs
94. Find the smallest number which when you divide by 2, the remainder is 1; when D. 30 hrs
you divide by 3, the remainder is 2; when you divide by 5, the remainder is 4 and which 100 Find the volume of the solid generated by revolving the area bounded by x = y2
2
when you divide by 6, the remainder is 5. and x = 2- y about the y – axis.
A. 39 A. 8 pi/3
B. 49 B. 16 pi/3
C. 59 C. 10 pi/3
D. 69 D. 7 pi/3
95. A man bough 20 pcs of assorted calculators for P20, 000. These calculators are
of three types namely: 1B 21.C 41.B 61.D 81.A
a. programmable at P3,000/pc 2. B 22.A 42.C 62.D 82. C
b. scientific at P1,500/pc 3. A 23.B 43.A 63.B 83.
c. household type 4. B 24.B 44.B 64.C 84.A
How many programmable calculators did the man bought? 5. A 25.D 45.D 65.A 85.D
A. 5 6. C 26.A 46.C 66.B 86.A
B. 2 7. B 27.A 47.A 67.D 87.B
C. 13 8. B 28.A 48.A 68.C 88.B
D. 15 9. C 29.C 49.B 69.B 89.A
10 A. 30. 50.C 70.D 90.B
11. D 31.C 51.B 71.A 91.B c. –b/a
12. B 32.B 52.C 72.D 92.A d. –c/a
13. D 33.D 53.A 73.A 93.A
14. D 34.B 54.A 74.A 94.C 7. They are equation whose members are equal only for certain (for possibly) no
15. C 35.B 55.A 75.C 95.B values of unknown.
16. A 36.A 56.C 76.A 96.B a. Conditional equation
17. D 37.C 57.C 77.A 97.A b. Inequalities
18. B 38.B 58.C 78.A 98.B c. fix equation
19. A 39.C 59.B 79.D 99.C d. Temporary equation
20. D 40.A 60.A 80.C1 00.B 8. Roots which are equal to zero are called the:
a. Trivial roots
MATHEMATICS MODULE 2 b. Identical
c. Symmetric
1. When the corresponding elements o two rows of determinant are proportional, d. Rational
then the value of the determinants is: 9. When all x are replaced by y and all y are replaced by x and the equation
a. Multiplled by the ratio remains the same, then the equation is said to be:
b. Zero a. Equivalent
c. Unknown b. Identical
d. One c. symmetric
2. When two rows are interchanged in position, the value of the determinant will be: d. Rational
a. Unchanged 10. The number 0.123123123 … is:
b. Becomes zero a. Irrational
c. Multiplied -1 b. Surd
d. Unpredictable c. Transcendental
3. If every element of a row (or column) are multiplied by a constant k, then the d. Rational
value of the determinant is: 11. To eliminate the surd, we multiply it by its ______________:
a. Multiplied by –nk a. Square
b. k to the n b. Cube
c. Multiplied by k c. Reciprocal
d. Anyone of the above may be true d. Conjugate
4. If the quadratic equation ax2 + bx + c = 0, when b2 is equal than 4ac. then the root 12. The letter D in the Romans Numerals is equivalent to:
are: a. 50
a. Equal b. 500
b. Real and unequal c.5000
c. Imaginary d.50000
d. Extraneous 13. A statement which is accepted without proof:
5. In the quadratic equation ax2 + bx + c = 0, when b2 is greater than 4ac, then the a. Postulate
root are: b. Lemma
a. Equal c. Theorem
b. Real and unequal d. corollary
c. Imaginary 58. There is mo change in the motion of the body unless a resultant force is acting
d. Extraneous on it. This law is known as:
6. In the quadratic equation ax2 + bx + c = 0, if r1 + r2 represent the roots, the r1 = r2 a. the law of Inertia
is equal to: b. Third law of Newton
a. b/a c. Law of resistance
b. c/a d. Doppler’s principle
59. The energy which body possess by virtue of its positions, configuration or internal a. Boyle’s law
mechanism is called ________. b. Young’s law
a. potential energy c. Gay lussac law
b. Kinetic energy d. Charles law
c. Mechanical energy 66. At any two points along a streamline in an ideal fluid in steady flow, the sum of the
d. electrical energy pressure, the potential energy per unit volume, and the kinetic energy per unit volume
60. If the mass of the body is expressed in grams and the velocity in cm/sec, the has the same value. This concept is known as:
kinetic energy is expressed in: a. Bernoulli’s theorem
a. Ergs b. Fluid theory
b. Joules c. Hydraulic theorem
c. Coulombs d. Pascal theorem
d. Slugs
61. Energy is given to a body or systems of bodies when work is done upon it. In this
process there is merely a transfer of energy from one body to another. In such transfer, no Answers:
energy is created or destroyed; it merely changes from one to another. This statement
is known as: 1. B 21 B 41.A 61.A
a. The law of conservation of energy 2. C 22.B 42.A 62.A
b. Energy transformation 3. C 23.B 43.C 63.A
c. Coulomb’s law 4. A 24.C 44.B 64.A
d. law of power 5. B 25.B 45.D 65.A
62. The deformation of elastic body is directly proportional to the applied force, 6. C 26.B 46.C 66.A
provided that the elastic limit is not exceeded. This theory is known as: 7. A 27.A 47.B
a. Hooke’s law 8. A 28.C 48.B
b. Young theory 9. C 29.D 49.A
c. Keppler’s law 10.D 30.C 50.D
d. Bulk modulus 11.D 31.B 51.D
63. If an external pressure is applied to a confined fluid, the pressure will be 12.B 32.A 52.A
increased at every point in the fluid by the amount of external pressure. This theory is 13.A 33.B 53.D
known as: 14.C 34.B 54.C
a. Pascal’s law 15.A 35.A 55.A
b. Hydraulic law 16.D 36.A 56.C
c. Hydrostatic law 17.B 37.D 57.B
d. Brangg’s law 18.D 38.B 58.A
19.A 39.A 59.A
20 A 40.D 60.A

64. A body wholly or partially submerged in a fluid experiences an upward force equal MATHEMATICS MODULE 1
to the weight of the fluid displaced. This theory is known as:
a. Archimedes principle 1. The solid formed by revolving the ellipse about is minor axis is called ________
b. Boyle’s law .
c. Third law of Newton a. spheroid
d. Fluid theory b. oblate spheroid
65. If the temperature of a confined gas does not change the product of the pressure c. prelate spheroid
and volume is constant. This statement is known as: d. ellipsoid
2. When two planes intersect with each other the amount of divergence between the a. The product of the frequency and wavelength
two planes is expressed by measuring the: b. Distance of the crest to the next crest
a. polyhedral angle c. Always equal to 186.00 miles per. sec.
b. plane angle d. The ratio of the frequency to wavelength
c. reflex angle 11. Acceleration is:
d. dihedral angle a. the same as velocity
3. If the product of the slope of any two straight lines is negative. One of this are said b. the same as displacement
to be: c. the rate of change of velocity
a. parallel d. always zero
b. skew 12. Mass is the quantitative measure o
c. perpendicular a. inertia
d. Non-intersecting b. gravity
4. The logarithm of 1 to any base is: c. weight
a. zero d. momentum
b. one 13. A sequence of number where every term is obtain by adding all the preceding terms
c. infinity of a square number series such as 1, 5, 14, 30, 55, 91 …is called _________.
d. intersecting a. Triangular number
5. The maximum displacement of vibration from the equilibrium is called: b. Tetrahedral number
a. frequency c. Euler’s number
b. speed
c. amplitude 98. Square root of the product of two terms of a geometric progression.
d. period a. Median
6. If the velocity of the body is doubled, b. Mean
a. The kinetic energy is quadrupled c. Geometric mean
b. the kinetic energy is halved d. Geometric term
c. the potential energy is halved 99. Which of the following cannot be probability?
d. the potential energy is doubled a. 0.1
b. 0
c. 1
d. 0.232323
7. A body traveling to a circle with constants speed: 100. If a is the number of times that an event will take place and b is the number of times
a. Is accelerated that it will not take place, then the probability that it will take place is:
b. Has constant velocity a. a/b
c. Does no move b. b/a
8. Ivory soap floats in water because: c. a/(b+a)
a. All matter has mass d. ab(a+b)
b. The density of ivory soap is unity
c. The specific gravity of ivory soap is greater than that of water
d. The specific gravity of ivory soap is greater than that of water
9. When two wave of the same frequency, speed and amplitude traveling in opposite Answers:
directions are superimposed:
a. Destructive interference always results
b. Constructive interference always results 1. B 21. C 41. A 61. D 81. C
c. Standing waves are produced 2. D 22. A 42. C 62. C 82. C
d. The phase difference is always zero 3. C 23. A 43. C 63. B 83. D
10. The velocity of a wave is: 4. A 24. A 44. A 64. D 84. C
5. A 25. A 45. D 65. A 85. A Concentric circles – circle having the same center with unequal radii.
6. D 26. B 46. D 66. A 86. B Tangent circles – circles tangent to the same line at the same point.
7. A 27. A 47. D 67. A 87. B Inscribed circle – (in a polygon) when the sides of the polygon are tangent to it.
8. D 28. A 48. 68. A 88. A Circumscribed circle – (about a polygon) when it passes through the vertices of the
9. C 29. A 49. A 69. A 89. B polygon.
10. A 30. A 50. B 70. B 90. D Circular ring/annulus – area included between two concentric circles of unequal radii.
11. C 31. A 51. A 71. B 91. D
12. A 32. B 52. A 72. D 92. A
13. D 33. B 53. A 73. C 93. A POLYGON
14. A 34. C 54. A 74. A 94. B Polygon – a plane closed by broken lines
15. A 35. D 55. B 75. B 95. A Regular polygon – polygon whose angles are equal and all of whose sides are equal
16. A 36. A 56. A 76. C 96. C Similar polygon – polygon whose corresponding angles are equal and their
17. A 37. D 57. B 77. B 97. C corresponding sides are proportional.
18. A 38. D 58. A 78. A 98. C Center of a polygon – common center of its inscribed and circumscribed circle.
19. A 39. A 59. B 79. B 99. C Diagonal of a Polygon – line joining any two non-consecutive vertices.
20. A 40. B 60. A 80. A 100.C Apothem (of a regular polygon) – the perpendicular line drawn from the center of the
inscribed circle to any one of the sides. It is the radius of the inscribed circle.

Classifications of Polygon
PLANE GEOMETRY
Number of Sides Polygon
3 triangle
Definitions of Terms 4 quadrilateral
Axiom – a statement of truth of which is admitted without proof. 5 pentagon
Theorem – a statement of truth which must be established by proof. 6 hexagon
Corollary – a statement of truth of which follows with little or no proof from a theorem. 7 heptagon
Postulate – In construction or drawing of lines and figures of which is admitted without proof. 8 octagon
Hypothesis – part of a theorem which is assumed to be true. 9 nonagon
Conclusion – part of a theorem which is to be proved. 10 decagon
Converse of a Theorem – another theorem wherein the hypothesis and conclusion of the 11 undecagon
first are reversed, i.e. the hypothesis becomes the conclusion and the conclusion becomes 12 dodecagon
the hypothesis.
Trapezoid – a quadrilateral with only two sides of which are parallel.
CIRCLE Parallelogram – a quadrilateral whose opposite sides are parallel.
Circle – locus of points which are at the same distant from a point within, called the center. Rhombus –a parallelogram with equal sides and oblique angles.
Diameter – a line passing thru the center, terminating at both ends on the circle. Rectangle – a parallelogram whose angles are right angles.
Radius – a line drawn from the center to the circle Square – a rectangle with equal sides.
Arc – a part of the circle. Isosceles trapezoid – one whose non-parallel sides are equal.
Chord – a line joining two points on a circle. Isometric figures – figures whose parameters are equal.
Secant – an indefinite line intersecting the circle in two points.
Tangent – an indefinite line touching a circle at only one point.
Segment – position of a circle between chord and its arc. Properties of Plane Figures
Sector – position of a circle between two radii and arc.
Inscribed angle – an angle whose vertex is a point in the circle, the sides of which are 1. The exterior angle of a triangle is greater either non-
chords. β φ adjacent interior angle and is equal to their sum.
Central angle – an angle whose vertex is at the center of the circle the sides of which are α
radii.

d
φ=α+β D E
DE  AB
B
2. A The diagonal of a parallelogram divides the A B DE = ½ AB
parallelogram into two congruent triangles.

∆ ACD ≅ ∆ ABC 8. D C The median of a trapezoid is parallel to the bases


C D and equal to one-half of their sum.
E F EF  AB 
EF = (AB+CD)/2
A
3. The opposite sides and opposite angles of a
A B
parallelogram are equal.
9. C The intersection of the three angle bisectors meet at
P
AB = CD ∠ADC = ∠ABC o Q a common point is called incenter, which is
AD = BCD ∠BAD = BCD equidistant from the three sides of a triangle. The
C inscribed circle is called incenter.
A
R B
4. The diagonals of a rhombus are perpendicular to OP = OQ = OR
A B
each other and bisect the angles through which they
pass.
B
AC⊥BD 10. The intersection of the three perpendicular bisectors
C D
meet at a common point called circumcenter, which
∠ACD = ∠ACB = ½ ∠ABC
is equidistant from the three vertices of the triangle.
∠ADB = ∠CDB = ½ ∠ADC o The circle whose center is 0 touching the three
vertices is called circumcenter.
A C
OB = OA = OC

B
5. A B The diagonals of a parallelogram bisect each other. 11. The intersection of the three altitudes of a triangle
meet at a common point called orthocenter.
A⋅X = XC = ½ AC
X BX = XD = ½ BD
C D A C
C 12. The three medians of any triangle meet at a
B
6. The median of the hypotenuse of a right triangle common point which is two-thirds of the distance
have equal distances from the three vertices. from each vertex to the midpoint of the opposite
Q side. The point of intersection is called the centroid.
A B P M
Mc AMc = BMc = CMc 2/3 BR = BM
2/3 CP = CM
A 2/3 AQ =CAM
7. The line segment which joins the midpoint R C
C

B
(AD) = √(AB)(AC)
B

13. C The altitude upon the hypotenuse of the right triangle 18. The tangent line is a mean proportional between the
is the mean proportional between the segments of entire secant and its external segment.
the hypotenuse.

CD = √(AD)(DB) L1 A1/A2 = (L12) / (L22)


A D B L2
A1
A2
14. A central angle is measured by its intercepted arc.
A
θ θ 19. B The sum of interior angles of any polygon
of n sides is

F ∠A + ∠B + ∠C… = (n–2)180°

15. The inscribed angle is measured by one-half of the


intercepted arc. C
A
B ∠ABC = ½ AC
θ E D
20. Interior angle (θ i) and exterior angle (θ 0 ) of a
C regular polygon of n sides is
θ0
16. An angle formed by two chords intersecting within a θi θI = [(n–2)/n] 180°
circle is measured by one half of the sum of the arcs θo = 360° / n
A intercepted by it and its vertical angle.
B
P (PA)(PB) = (PC)(PD)
D
C

Formulas for Plane Figures


17. The product of one entire secant and its external
segment equals the product of the other entire I. Area of Triangles
D secant and its external segment. I.0 Given sides a, b, and c
A (Hero’s formula) a b
A = √s(s-a)(s-b)(s-c)
Where: 2 sin δ
s = ½ (a+b+c) c
6. Given: sides a, b, and c and inscribed in a circle of radius R
2. Given base b and altitude h A = abc
4R
A = ½ bh
a
Hbbh h R r
b b
b c

7. Given: sides a, b, and c and circumscribed about a circle of radius r


A = rs
Where:
s=a+b+c
2
3. Given: equilateral triangle of side s b
a
A = √ 3 s2
4 r

c
s s II. Area of Parallelogram
1. Given: base b and altitude h
A = bh b
2. Given: sides a and b and their included angle θ
s A = ab sin θ

a
θ
4. Given: two adjacent sides and the included angle
A = ½ bc sin α
A = ½ ac sin β III. Area of a Trapezoid
A = ½ ab sin δ Given: bases a and b and altitude h
b a A = ½ (a+b) a

h
c
5. Given: at least two angles and a side b
A = a2sinβ sinδ
2 sinα
IV. Area of cyclic quadrilateral (Bramaguphta’s Formula)
A = b2sinα sinδ Given: sides a, b, c, and d
2 sin β A =√(s-a)(s-b)(s-c)(s-d) A
d
A = c2 sinα sin β Where: a
D
s = ½ (a+b+c+d) B d2
A+C = 180°
B+D = 180° d1 X. Area of a sector of a circle
c Given: radius r and θ
b
s = rθ r
A = ½ r2 θ s
θ
C

V. Area of Rhombus
Given: diagonals d1d2
A = ½ d1d2 XI. Area of a segment of a circle
d1
d2 A = ½ r2 (θ – sinθ) r
Where θ is in radius
θ
r
VI. Area of Trapezium
1. Given: diagonals d1 and d2 and their included angleφ
A = ½ d1d2 sin φ
XII. Area and Circumference of Ellipse
A d1 Given: major axis a and minor axis b
d2 φ
A = πab
C = 2π √(a2+b2)/2
a
VII. Area of Regular Polygon
R b
1. Given : n sides, each of length s.
A = ½ ns2 cot (π/n)
2. Given: n sides, and apothem a.
A = na2 tan (π/n) a
3. Given: n sides inscribed in a circle of radius R.
A = ½ nR2 sin (2π/n)
a

VIII. Area (A) and Perimeter (P) of a Circle


1. Given: radius r and diameter d
A = πr2 = πd2 r XIII. Area of a parabolic segment
4 Given: base b and height h
P = 2πr = πd A = 2/3 bh

d h
IX. Area of Annulus
Given: circle of radii r1 and r2
A = π(r12 – r22) r1
Where r1>r2 r2 b
Supplementary Problems c) acute angle d) inscribed angle
1. Bisectors of the 3 angles of a triangle meet at a common point 11.Two angles whose sum is 360 degrees are said to be
called the __________ a) supplementary b) complimentary
a. orthocenter b. centroid b) elementary d) explementary
c. incenter d. circumcenter
2. The perpendicular bisector of the sides of a triangle pass 12. All circles having the same center but with unequal radii are
through a common point called the __________ called as
a. orthocenter b. centroid a. eccentric circles b. concentric circles
c. incenter d. circumcenter c. inner circles d. Pythagorean circles
3. Which of the following is not a property of a circle? 13. A circle is _________ outside the triangle if it is tangent to
a) through 3 points not in the straight line one circle and one side and the other two sides prolonged.
only 1 can be drawn a. inscribed b. escribed
b) a tangent to a circle is perpendicular to the radius at the c. circumscribed d. tangent
point of tangency and conversely. 14. A triangle having three sides of unequal length is known as
c) an inscribed angle is measured by ½ of the intercepted a. equilateral triangle b. scalene triangle
arc. c. isosceles triangle d. equiangular triangle
d) the arc of 2 circles subtended by equal central angle are
equal. 15. In a proportion of four quantities, the first and the fourth terms
4. Which of the following is not a property of a triangle? are referred to as the
a) the sum of the 3 angles of the triangle is equal to two a.means b. extremes
right triangles. c. denominators d. axiom
b) the sum of the 2 side of the triangle is less than the 3 rd 16. A statement the truth of which is admitted without proof is
side a. theorem b. corollary
c) if the 2 sides of the triangle are unequal, the angles c. postulate d. axiom
opposite are unequal. 17. The part of the theorem which is assumed to be true is the
d) the altitude of a triangle meets in a point a. corollary b. hypothesis
c. postulate d. conclusion
5. The radius of the circle inscribed in a polygon is called as 18. In Geometry,the construction or drawing of lines and figures,
a) internal radius b) radius of gyration the possibility of which is admitted without proof is called
c) apothem d) hydraulic radius the:
6. A polygon with 12 sides is called as a. corollary b. theorem
a) bidecagon b) dodecagon c. postulate d. hypothesis
c) nonagon d) pentedecagon 19. A statement the truth of which follows with little or no proof from the theorem is
7. A polyhedron having bases 2 polygons in parallel plane and for lateral faces triangles or a. corollary b. axiom
trapezoid with 1 side lying on 1 base and the opposite vertex or side lying on the other base c. postulate d. conclusion
of the polyhedron is 20. A polygon is ______ when no side, when extended, will pass
a) pyramid b) cone through the interior of the polygon.
c) prismatoid d) rectangular parallelepiped a. convex b. equilateral
8. An angle greater than a straight angle but less than 2 straight angles is called as c. isoperimetric d. regular
a) complement b) supplement 21. A circle is said to be _______ to a polygon having the same perimeter with that of the
d) complex d) reflex circle
9. A part of a circle is often called as a. congruent b.isoperimetric
a) sector b) cord c.proportional d. similar
b) arc d) segment 22. The intersection of the sphere and the plane through the center is the
10. An angle whose vertex is appoint on the circle and whose sides are cords is known as a. great circle b. small circle
a) interior angle b) vertical angle c. poles d. polar distance
23. Points that lie on the same plane are said to be 36. Find the area of the folded triangle shown below.
a. collinear b. coplanar
c. dihedral d. parallel

24. What kind of a quadrilateral is always formed by connecting the midpoints of the
consecutive sides of a quadrilateral? Ans. parallelogram

25. The angles of a pentagon are in the ratio 3: 3: 3: 4: 5. Find the largest angle. Ans. 150deg

26. The legs of a triangle are in the ratio 3: 3 and its area is 108 sq. cm. Find the length of the a) 15 b) 20 c) 25 d) 40
legs. Ans. 12 cm., 18 cm 37. The area of a triangle whose angles are A = 69.159˚, B =
34.246˚, C = 84.595˚ is 680.60 m2. The longest side is
27. Find the side of a regular octagon inscribed in a circle of radius 19 cm. a) 15.387 b) 52.431 c) 37.853 d) 64.974
38. Find the area of a decagon that can be inscribed in a circle
28. The upper and lower bases of a trapezoid are 6cm and 12cm respectively, and the with radius 10 cm.
altitude is 4cm. The non-parallel sides of the trapezoid are produced until they meet at P. a)11.387459 b)10.066446 c)37.853527 d)64.974136
Find the altitude of the triangle whose vertex is P and whose base is the lower base of the 39. Find the area of a regular 5-pointed star that can be
trapezoid. Ans. 8cm inscribed in a circle with radius 10 cm.
a) 112.257 b) 56.129 c) 114.535 d) 124.431
29. A secant and a tangent are drawn to circle from the same point. If the internal segment is 40. Find the radius of the largest circle that can be inscribed in
1cm longer than the external segment, and if the tangent is 6cm long, find the length of the the triangle with sides a = 8cm, b = 15cm, and c= 17cm
secant. Aa) 3 b) 1.8 c) 30 d) 8.5
Ans.√145 cm. 41. Find the radius of the smallest circle that circumscribe the
triangle in the previous problem.
30. Two parallel chords of a circle are 16cm and 30 cm long respectively, and the distance a) 8.5 b) 10.2 c) 9.51 d) 7.75
between them is 23cm long. Find the distance each is from the center, and radius of the
circle. 15cm and 8cm., r = 17 cm. 42. Chords AB = 12 cm and BC = 8cm of a circle forms 120˚.
Find the radius of the circle.
31. The diagonals of a rhombus have the ratio of 3:4. The area of the rhombus is 96 sq.cm. a) 10 b) 12.164 c) 14.742 d) 24.634
Find a side of the rhombus. Ans. 10cm

32. The areas of two similar triangles are to each other as 9:25. The perimeter of the first is 43. The radius of the smallest circle that can be circumscribe a
36 cm. What is the perimeter of the second triangle? Ans. 60cm. right triangle of sides a, b, and c.
a. a + b + c b. a + b – c
33. The side of a rhombus is 13cm, and one of its diagonals is 24cm. Finds its area. 120 2 2
sq.cm. C. a – b + c d. c
2 2
34. In a circle whose radius is 10cm, a chord bisects the radius at right angles. Find the area 44. The radius of the largest circle that an be inscribed in a right triangle of sides a, b and c.
of the smaller of the two segments into which the chord divides the circle. 25/3(2π - 3√3)sq a. a + b + c b. a + b – c
cm. 2 2
c. a – b + c d. c
35. From a point P, exterior to circle C, the tangents are drawn to the circle. The distance of P 2 2
from the center of C is 4cm. The length of each tangent is 2cm. Find the diameter of the
circle. Find the area bounded by the tangents and the arc between them. Find CD.
Ans. d = 4√3 cm, A = (4√3 - 2π) sq. cm.
54. Six lines are situated in the plane so that no two are parallel
and no three are congruent. How many points of intersection
are there?
a. 13 b. 14 c. 15 d. 16
55. A triangle is inscribed in ac ircle such that one side of the triangle is a diameter of the
circle. If one of the acute angles of the triangle has a measure of 60˚ and the opposite side of
45. Find the area of the cyclic quadrilateral. that angle has a length 12, then the nearest value of the radius of the circle is.
a. 6.93 b. 1.93 c. 9.6 d. 5.8
56. The hypotenuse of a right triangle is 34 cm. Find the length of the two legs if one leg is 14
cm longer than the other.
a. 18 and 32 cm b. 15 and 29 cm
c. 17 and 32 cm d. 16 and 30 cm
57. The sum of the interior angles of a polygon is 540 degrees. Find the number of the sides.
a. 5 b. 6 c. 8 d. 11
AP = 50cm, BP = 28cm, DP = 56cm, Ө = 30degrees 58. A circle of radius 6 has half its area removed by cutting off a border of uniform width. Find
the width of the border.
a) 456 b) 627 d) 364 d)525 a. 22 b. 135 c. 375 d. 176
46. The internal angle of a polygon is 150˚ greater than its external angle, how many side has 59. A circle is inscribed in a 3, 4, 5 right triangle. How long is the line segment joining the
the polygon? point of tangency of the “3-side” and the “5-side”?
a) 9 b) 6 c) 7 d) 8 a. 1.28 b. 1.35 c. 1.46 d. 1.79
47. The 30˚ angle of a right triangle is bisected. Find the ratio of which opposite side is 60. Two figures having equal perimeter are said to
divided. a. congruent b. isoperimetric
a) 1:2 b) 1:3 c) √2:2 d) 1:4 c. equal d. similar
48. A certain angle has a supplement 5 times the compliment. Find the angle. 61. A right triangle whose length and side may be expressed as ratio of integral units
a. 67.5˚ b. 58.5˚ a. isosceles triangle b. scalene
c. 30˚ d. 27˚ c. pedal triangle d. primitive triangle
49. Find the supplement of an angle whose compliment is 62 degrees 62. The perpendicular bisector of the sides of a triangle intersect at the point known as the
a. 30˚ b. 28˚ a. orthocenter b. cicumcenter
c.152˚ d. 118˚ c. centroid d. incenter
50. The sum of the interior angles of a polygon is 540 degrees. Find the number of the sides. 63. The bisectors of the 3 angles of a triangle meet at a common called the
a. 5 b. 6 c. 8 d. 11 a. circumcenter b. centroid
51. Six lines are situated in the plane so that no two are parallel c. orthocenter d. incenter
and no three are congruent. How many points of intersection 64. An isosceles trapezoid ABCD, AB=5, BC=AD = 3 and CD = 8. Find the length of the
are there? diagonals.
A. 13 b. 14 c. 15 d. 16 a. 7.5 b. 8 c. 7 d. 6
52. The area of a square field exceeds another square by 56 65. The diagonals of rhombus have length 4 and 6 inches. Find the area of the region inside
square meters. The perimeter of the larger field exceeds ½ of the rhombus but outside the circle that is inscribed in a rhombus.
the smaller by 26 m. What are the sides of each field? Ans. a. 3.3 b. 3.4 c. 2.9 d. 2.85
larger field, 9m or 25/3m; smaller field, 5m or 11/3m
66. Two equilateral triangles, each with 12-cm sides, overlap each other to form a 6-point
53 . The sum of the areas of two unequal square lots is 5,200 “Star of David”. Determine the overlapping area, in sq-cm.
square meters. If the lots were adjacent to each other, they a. 36.64 b. 41.57 c. 28.87 d. 49.88
would require 320 meters of fence to enclose the combined
area formed by them. Find the dimensions of each lot. 67. Let D be the set of vertices of a regular dodecagon (12 sided plane polygon). How many
Ans.60m and 40m, or 68m and 24m. triangles may be constructed having D as the vertices?
a. 220 b. 120 c. 240 d. 180
Rectangular Parallelepiped – polyhedron whose six faces are all rectangles.
68. A group of children playing with marble place 50 pieces of the marble inside the
cylindrical container with water filled to a height of 20 cm. If the diameter of aech marble is Prism – polyhedron of which two faces are equal polygon in parallel planes and the other
1.5 cm and that of the cylindrical container 6 cm, what would be the new height of water faces are equal parallelogram.
inside the cylindrical container after the marbles were placed inside?
a. 23.125 b. 24.125 Pyramid – a polyhedron of which one faces is a regular polygon and other faces are triangles
c. 26.125 d. 25.125 which have a common vertex.

69. A horizontal cylindrical tank with hemispherical ends is to be filled with water to a height of Regular Prism – prism whose lateral edges are perpendicular to its bases.
762mm. If the total inside length of the cylinder is 3,600mm, find the volume of water in cubic
meters that will have to filled into the tank up to the required height. If one edge of a cube and Regular Pyramid – pyramid whose base is a regular polygon and whose altitude passes
the volume of the cube respectively in sq.cm. and cu.cm. through the center of base.
a. 864 and 1728 b. 684 and 1728
c. 864 and 1728 d. 864 and 1729 Slant height – altitude of lateral faces.

70. A pyramid with square base has an altitude of 25 cm. If the edge of the base is 15cm, Section – polyhedron is the plane figure formed by a plane passing through the solid.
calculate for the volume of the pyramid.
a. 1885 b. 1875 c. 1785 d. 1958 Convex Polygon – a polygon whose each interior angle of which is less than 18 0.

71. If a right circular cone has a base radius of 35 cm and an altitude of 45cm, solve for the Frustum of Pyramid – section of the pyramid between the base and a section parallel to the
total surface area in square cms. base.
a.10116 b. 10117 c. 11117 d. 12117
Cone – a solid bounded by a conic surface and a plane intersecting all the elements.
72. Two circles of radius 4 and 6.93 are placed in a plane so that their circumference
intersect at right angles. Find the area of the interlapping region. Dihedral Angle – the divergence of two intersecting planes.
a. 14.81 b. 13.92 c. 14.18 d. 15.1
Sphere – a solid bounded by a surface all points of which are equidistant from a point called
73. One side of a regular octagon is 2. Find the area of the region inside the octagon. center.
a. 3.91 b. 13.92 c. 14.18 d. 15.1 Great circle – the intersection and a plane passing through the center.

SOLID GEOMETRY Small circle – the intersection of a sphere and a plane not passing through the center.

Definition of Terms: Quadrant – one-fourth of a great circle.


Polyhedron – a solid bounded by planes.
Regular Polyhedron – polyhedron whose faces are congruent regular polygons and Zone – portion of sphere bounded by a spherical polygon and the plane of its sides.
whose polyhedral angles are regular polygons and whose polyhedral angles are equal.
Lune – portion of a sphere lying between two semi – circles of great circle.
There are only five regular polyhedron:
Spherical pyramid – portion of sphere bounded by a spherical polygon and the plane of its
Tetrahedron – 4 faces
sides.
Hexahedron – 6 faces
Octahedron – 8 faces
Spherical Sector – portion of sphere generated by the revolution of circular sector about any
Dodecahedron – 12 faces
diameter of the circle of which the sector is apart.
Icosahedron – 20 faces
Spherical segment - portion of sphere included between two parallel lines.
Cube – a polyhedron whose six faces are all squares
Spherical wedge – portion bounded by a lune and the planes of two great circles. 1
Focus – a solid formed by revolving a circle about a line not intersecting it. Prismoidal Formula: V = h( b + B + 4M)
6

Formulas in Solid Mensuration


6. Sphere of radius r or diameter D:
1. Cube with edge s:
4 3 1
Volume : V = s 3 Volume : V = πr or V = πD 3
3 6
Surface Area : A = 6s 2
Surface Area : S = 4πr 2 or S = πD 2

7. Right circular cylinder with radius r and altitude h:


2. Rectangular parallelepiped with edges a, b, c and diagonal D:
Volume : V = πr 2 h
Volume : V = abc
Lateral Area : S = 2πrh
Surface Area : A = 2( ab + ac + bc )

Diagonal : D = a 2 + b 2 + c 2 8. Right circular cone with radius r and altitude h:

3. Volume of a prism with base B and altitude h: 1 2


Volume : V = πr h
3

V = Bh Lateral Area : S = πrl

(l = slant height)

4. Volume of a pyramid with base B and altitude h: 9. Frustum of a right circular cone with base radii r and R and altitude h:

1 slant height l:
V= Bh
3
Volume : V =
1
3
(
πh r 2 + R 2 + rR )
5. Volume of a prismatoid with bases b and B, midsection M and altitude h: Lateral Area : S = πl( r + R )

10. Frustum of a pyramid with bases b and B and altitude h:


Volume : V =
1
3
(
πh b + B + bB ) Volume : V =
1
6
2
ZR or V = πR 2 h
3

1 where V = volume of spherical sector


Lateral Area : V= (p + P )l
2

where l = slant height


Z = area of the zone which forms the base of the sector
p = perimeter of base b
R = radius of the sphere
P = perimeter of base B
h = altitude of the sphere
11. Area Z of a zone with altitude h on a sphere of radius R:
15. Ellipsiod

V = 4/3 π abc b
Z = 2πRh
a
For oblate spheroid c
V = 4/3 π ab2

For prolate Spheriod


12. Spherical segment of one base and altitude h on a sphere of radius R: V = 4/3 π a2 b

1 2
Volume : V = πh ( 3R − h)
3 16Paraboloid
h
Total Area : T = πh( 4R − h) V = ½ π a2 h

13. Spherical segment of two bases with radii a and b and altitude h on a sphere
17. Ungula
of radius R: V = 2/3 r2h h
S = 2 rh
Volume : V =
1
6
(
πh 3a 2 + 3b 2 + h 2 )
r
Total Area (
: T = π 2Rh + a + b 2 2
)
14. Spherical Cone = a spherical sector having only one conical surface

Supplementary Problems:
13. Find the volume of a prism having an altitude of 13 cm and a rectangular base of 8 cm
1. The slant height of a regular hexagonal pyramid is 9 cm. and a side of the base is 6 cm. long and 4 cm wide. Ans.416 cm3
Find the volume of the inscribe cone. Ans. 27 √6π cu. cm. 14. The base of a right prism is a rhombus whose sides are each 10 cm long and whose
shorter diagonal is 12 cm. Find its volume if the altitude is 8 cm. Ans. 768 cu cm
2. A rectangle, 4 cm wide and 9 cm long is revolved about its longer side. Find the total
surface area and the volume of the cylinder generated. Ans. T = 104π sq. cm. ; V = 144π 15. The diameter of two spheres are in the ratio 2:3 and the sum of their volumes is 1,260 cu
cu. cm m. Find the volume of the larger sphere. Ans. 972 cu m
If the volume of a cube is 625 m3, find the length of the diagonal. Ans. 14.8 m
3. A solid wooden cone is cut into two parts, the cut being parallel to the base and halfway
between the base and the vertex. Find the ratio of the weights of the two parts. Ans. 7 : 1 16. Find the volume of a regular triangular pyramid whose slant height is 17 cm and whose
altitude is 15 cm.Ans. 960 3 cu cm
4. Find the area of a zone of one base, if the base is a circle of radius 6 cm and is 8 cm
from the center of the sphere. Ans 40 π sq. cm. 17. Find the capacity in liters of a pail in the form of a frustum of a circular cone if the radii of
the bases are 10 and 15 cm and the depth of the pail is 36 cm. Ans. 17.9 L
5. Three spheres of radii a, 2a, and 3a are melted and formed into a new sphere. Find the
surface of this sphere. Ans. 24 3√6 πa2 18. The space occupied by the water in a reservoir is the frustum of a right circular cone.
Each axial section of this frustum has an area of 28 sq m and the diameter of the upper and
6. A sphere whose radius is 10 cm is cut into three parts by parallel planes 8 cm and 6 cm. lower bases are in the ratio 4:3. If the reservoir contains 148 Π/3 cu m, find the depth of the
from the center respectively, and on the opposite sides of the center. Find the total water in the reservoir. Ans. 4m
surface of the part in the middle. Ans. 380 π sq. cm.
19. The lateral surface area of a right circular cylinder is 330 Π sq cm. If its altitude is 20
7. In a frustum of a regular pyramid the bases are squares with sides 6 cm and 12 cm cm, find the diameter of its base. Ans. 16.5 cm
respectively. If the lateral area of the frustum is just half of its total area, what is the
volume? 20. A reservoir is in the form of the frustum of an inverted square pyramid with upper base 24
Ans: 420 m3 m, lower base edge 18 m and altitude 9 m. How many hours will it require for an inlet pipe to
fill the reservoir if water flows in at the rate of 5,000 liters per minute? Ans. 13.32 hr
8. The volume of two similar polyhedron are 64 and 125 cu cm. respectively. If the total
surface of the first is 112 square centimeters, find the total surface area of 21. A horizontal cylindrical tank with diameter of 0.60 m and 3.66 m long is filled with water to
thesecond.Ans.380πsq.cm a depth of 0.46 m. Find the number of liters of water in the tank. Ans. 851 L

9. Find the total surface and the volume of a tetrahedron whose edges are equal to a. 22. A sphere is inscribed in aright circular cone with altitude 15 cm. If the slant height of the
Ans: T = √3 a2 ,V = (√2/12) a3 cone is equal to the diameter of its base, find the surface area of the sphere. Ans. 100 sq cm

23. Use the prismoidal formula to find the volume of the common part of two cylinders, each
10. A sphere of radius 5 cm and a right circular cone of bass radius 5 cm and height 10 cm
with a radius of 6 cm, which intersect at right angles. Ans. 1,152 cu cm
stand on a plane. Find the position of a plane that cuts the two solids in equal circular
sections.
24. If the radius of a sphere is inscribed by 6 cm, its volume is multiplied by 27. Find the
Ans: d = 2 cm and h = 10 cm
radius of the sphere. Ans.3 cm
11. A cylindrical tin can has its height equal to the diameter of its base. Another cylindrical tin
can with the same capacity has its height equal to twice the diameter of its base. Find the
ratio of the amount of tin required for making the two cans with covers. Ans. 0.9524
1. Find the slope of x2y = 8 at point (2, 2)
a. –2 b. 2 c. 8 d. 4
12. Find the volume of the frustum of a regular square pyramid whose base edges are 4 cm
2. If y = x lnx, find y”
and 10 cm and whose slant height is 5 cm. Ans. 208 cu cm
a. 1/x b. ln x c. 1/lnx d. x
3. Evaluate the limit (x - 4)/(x2 – x – 12) as x approaches 4
a. 1/7 b. 0 c. infinity d. indeterminate 19. Find the area bounded by the curves y = 4x – x2, y = 0, x = 1, x = 3.
a. 23/3 b. 22/3 c. 21/4 25/3
4. Evaluate the limit lim x – sin x 20. Find the volume generated by revolving the region bounded by y = x 2 and y = x
x 0 x3 about the y= axis.
a. 1/3 b. ¼ c. 1/5 d. 1/6 a. π/8 b. π/10 c. π/6 d. π/12
5. Find the maximum point on the curve y = x3 – 3x2 – 9x + 5 21. If the first derivative of a function is constant, then the function is said to be:
a. (1, 15) b. (3, -22) c. (-1, 10) d. (-3, 21) a. constant b. linear c. sinusoid d. exponential
6. Determine the velocity of a body which moves according to the law S = 2t 3 – t2 + 4 22. Three sides of the trapezoid are each 8 cm long. How long is the 4 th side when the
where S is displacement in ft. and t is time in sec at t = 1sec. area of the trapezoid has the greatest value?
a. 2ft b. 6ft. c. 4ft. d. 10ft. a. 16 cm b. 12 cm c. 15 cm d. 10 cm
7. Find the equation of the tangent to the curve y = x 4 – x2 + 2 at x = -1. 23. Water is running out from a canonical funnel at a rate of 2 cu. in. per second. If the
a. 2x + y = 0 b. 2x – y = 1 c. 2x – y = 0 d. y + x = 1 radius of the top of the funnel is 4 inches and the altitude is 8 inches, find the rate at
8. Determine the altitude of the largest circular cylinder that can be inscribed in a right which the water level is dropping when it is 2 inches from the top.
circular cone of radius 6 inches and of height 15 inches. a. 2/9π in/sec b. -2/9π in/sec c. -3/2π in/sec d. 5/9π in/sec
a. 12 inches b. 8 inches c. 5 inches d. 10 inches 24. Find the area bounded by the parabola x2 = 8y and its latus rectum.
9. Given a cone of radius R and of altitude H, what percent is the volume of the largest a. 10.67 sq. units b. 32 sq. units c. 48 sq. units d. 16.67 sq. units
cylinder which can be inscribed in the cone to the volume of the cone?
a. 49% b. 44% c. 50% d. 60%
10. Find the most economical proportion for a box with an open top and a square base. 25. What theorem is used to solve for centroid?
a. b = h b. b = 4h c. b = 3h d. b = 2h a. Pappus b. Varignon’s c. Castiglliano’s d. Pascal’s
11. The 5m picture hung on a wall so that its bottom edge is 4m above an observer’s 26. Find the area (in sq. units) bounded by the parabola x 2 – 2y = 0 and x2 + 2y = 8.
eye. How far should the observer stand from the wall so that the angle subtended by a. 11.7 b. 4.7 c. 9.7 d. 10.7
the picture at the eye is a maximum. 27. Find the equation of the curve at every point of which the target line has a slope of
a. 4.9 b. 5.5 c. 6.7 d. 7.2 2x.
12. What is the largest area of a rectangle that can be inscribed within an ellipse whose a. x =-y2 + C b. y = -x2 + C c. y = x2 + C d. x = y2 + C
major diameter is 10ft and whose minor diameter is 6ft. 28. The rate of change of a function of y with respect to x equals 2 – y, and y = 8 when x
a.32.50 sq. ft. b. 28.00 sq. ft c. 30.00 sq. ft d. 34.00 sq.ft = 0. Find y when x = ln 2
13. The hands of the tower clock are 4 ½ ft and 6 ft long respectively. How fast are the b. 5 b. 2 c. –5 d. –2
ends approaching at 4 o’clock in ft per minute? 29. Solve the differential equation (x2 + y2)dx + 2xydy = 0
a. – 0.246 b. – 0.203 c. –0.264 d. –0.256 c. 3xy2 + x3 = C b. 2xy2 = C c. 3xy + 2 = C d. 3x2 + 2y = C
14. A man on the wharf (pier) is pulling a rope tied to a raft at a time rate of 0.60 m/sec if 30. Solve the differential equation y” – 5y’ + 6y = 0
the hands of the man pulling the rope is 3.66m above the level of the water, how fast d. Y = Ae2x + Be3x b. Y = Ae-2x + Be-3x
is the raft approaching the wharf when there are 6.10m of rope out? 5x
c. Y = Ae + Be 3x
d. Y = Ae2x + Be5x
a. –0.75 m/sec b. –0.55 m/sec c. –0.45 m/sec d. –0.65 m/sec 31. Find the equation f the family of orthogonal trajectories of the system of parabolas y 2
15. A balloon is rising vertically over a point A on the ground at the rate 15 ft/sec. A point = 2x + C.
B on the ground level is with the same horizontal plane as A and 30 ft from it, when e. y = ce-x b. y = ce2x c. y = cex d. y = ce-2x
the balloon is 40 ft from A, at what rate is its distance from B changing? 32. The rate of change of a certain substance is proportional to the amount of substance
a. 12 ft/sec b. 15 ft/sec c. 18 ft/sec d. 21 ft/sec is 10 grams at the start and 5 grams at the end of 2 minutes, find the amount of
16. What is the percentage error made in the computed surface area of a sphere if the substance remaining at the end of 6 minutes.
error made in measuring the radius is 3% f. 1.25 grams b. 2.67 grams c. 3.46 grams d. 2.98 grams
a. 3% b. 4% c. 5% d. 6% 33. The rate of population growth of a country is proportional to the number of
17. What is the allowable error in measuring the edge of the cube that is intended to inhabitants. If the population of a country now is 40 million and is expected to double
hold 8 cu.m. if the error of the computed volume is not to exceed 0.03 cu.m.? in 25 years, in how many years will the population be 3 times the present?
a. 0.002 b. 0.003 c. 0.0025 d. 0.001 g. 39.6 yrs b. 39.5 yrs c. 37.9 yrs d. 36.9 yrs
18. Find the radius of curvature of the curve y = x3/3 at x = 1.
a. 2 b. 2 /3 c. 3/ 2 d. 2 /2
34. Water at 100°C is transferred to a room which is at constant temperature of 60°C. a. 1 – i b. 1 + i c. -4(1 + i) d. 4(1 + i)
After 3 minutes the water temperature is 90°C, find the water temperature after 6 52. Find the equation of the family of orthogonal trajectories of the system of the
minutes. parabolas y2 = 2x + C.
h. 82.5°C b. 85.2°C c. 80°C d. 75°C a. y = Ce-x b. y = Ce2x c. y = Cex d. y = Ce2x
53. What is the quotient when 4 + 8i is divided by i3.
35. A body at 90°C cools in 10 mins to 70°C in a room temperature of 25°C. When will its
a. 8-4i b. 8+4i c. -8+4i d. -8-4i
temperature be 40°C? 54. Evaluate the expression (1+i2)10, where i is an imaginary number.
i. 39.8mins b. 38.8mins c. 36.8mins d. 34.7mins a. -1 b. 10 c. 0 d. 15
36. Evaluate ln (3 + j4) 55. Solve for x in (x+yi) (2+4i) = 14 + 8i.
j. 1.77 + j0.843 b. 1.61 + j0.927 c. 1.95 + j0.112 d. 1.46 + j0.102 a. 3 b. 4 c. 14 d. 8
37. Find the value of (1 + i)12 where i is an imaginary number.
k. –64 b. 64 c. 4 d. 4i
38. Simplify the expression i1997 + i1999
l. 0 b. –1 c. 1 + i d. 1- i TRIGONOMETRY
39. Express e0.32 + j0.56 in rectangular form
m. 1.167 + j0.732 b. 1.193 + j1.163 c. 1.452 – j0.315 d. 1.684 –
j1.462 Trigonometry – the branch of mathematics that deals with the solution of triangles.
40. Evaluate cos (0.492+j0.942) Angle – the space between two line meeting at a point called vertex.
n. –1.032 + j0.541 b. 1.302-j0.504 c. 3.12+j1.54 d. 1.48+j0.01
41. Evaluate the value of log (-5)
o. 5+jπ log e b. 5π+j log e c. log 5+jπ log e d. log 5π+j log e Kinds of Angles:
42. Find the Laplace transform of t3 e4t
p. 6/(s+4)4 b. 6/(s-4)4+ c. 6/(s-2)2 d. 6(s+4)4 1. Acute angle – an angle which measures between 0° to 90°
43. Find the Laplace transform of (1 – e-at)
2. Right angle – an angle measuring exactly 90°
q. 1/[s(s+a)] b. 1/(s2 + a2) c. 1/[s(s-a)] d. 1/(s+a)2
44. In complex algebra, we use a diagram to represent a complex plane commonly 3. Obtuse angle – an angle which measures between 90° to 180°
called:
a. De Moivre’s diagram b. Argand diagram 4. Straight angle – an angle measuring exactly 180°
c. Venn Diagram d. Funicular diagram
5. Reflex angle – an angle greater than 180° but less than 360°
45. When the corresponding elements of two rows of a determinant are proportional,
then the value of the determinant is:
a. unknown b. one c. zero d. multiplied by the ratio Two General Classes of Triangles
46. A sequence of number where the succeeding term is greater than the preceeding
term. 1. Right Triangle – a triangle with a right angle
a. isometric series b. divergent series
2. Oblique Triangle – a triangle without a right angle
c. convergent series d. dissonant series
47. In any square matrix, when the two elements of any two rows are exactly the same,
the determinant is:
Oblique Triangles can be further classified as:
a. unity b. positive integer c. zero d. negative integer
48. Which of the following cannot be an operation of matrices? 1. Acute Triangle – a triangle whose all angles are acute.
a. Subtraction b. multiplication c. addition d. division
2. Obtuse Triangle – a triangle with one obtuse angle.
49. Convergent series is a sequence of decreasing numbers or when the succeeding
term is ___________ than the preceding term.
a. ten times more b. equal c. greater d. lesser
50. Find the length of the vector (2, 4, 4).
a. 8.75 b. 7.00 c. 6.00 d. 5.18
51. Find the value of (1 + i)5, where i is an imaginary number.
The Pythagorean Theorem 4. CscA=1/SinA

- states that the sum of the squares of the legs is equal to the square of the
hypotenuse. 5. Versed SinA=1-CosA

Pythagorean Triple – three positive integers satisfying the Pythagorean principle.


Example: 3, 4, and 5; 5, 12, and 13; 20, 21 and 29; 8, 15, 17; 7, 24, 25; etc. 6. Coversed SinA=1-SinA

Supplementary Problems: 7. ExsecantA=SecA-1

1. A storm broke a tree 50 ft high so that its top touched the ground 30 ft from the foot of the
tree. What is the height of the part standing? Ans. 16ft. III. Pythagorean Identities

2. 1. Cos2A+Sin2A=1

3. How far from the center of a circle is its chord 8 inches long if its radius is 5 inches. Ans.
3” 2. 1+Tan2A=Sec2A

4. For what positive value of x will the following lengths be sides of a right triangle 2x + 1, 5x 3. Cot2A+1=Csc2A

– 1, 8x – 3? (The last being the longest) Ans. X = 1


IV. Sum and Difference of Angles Identities

I. Circular Functions 1. Sin(A+B)=SinACosB+CosASinB

1.SinA=y/r 4. CotA=x/y
P(x,y)
2. Sin(A-B)=SinACosB-CosASinB

2.CosA=x/r 5.SecA=r/x 3. Cos(A+B)=CosACosB-SinASinB

3.TanA=y/x 6.CscA=r/y 4. Cos(A-B)=CosACosB+SinASinB


TanA + TanB
5. Tan( A +B) =
II. The Relation Among Functions 1 − TanATanB

1. TanA=SinA/CosA TanA − TanB


6. Tan( A − B ) =
1 + TanATanB

2. CotA=CosA/SinA=1/TanA
V. Double Angle Identities

3. SecA=1/CosA 1. Sin2A = 2SinACosA


2. Cos2A = Cos2A-Sin2A 2. 2CosASinB=Sin(A+B)-Sin(A-B)
2
= 2cos A-1 3. 2CosACosB=Cos(A+B)+Cos(A-B)
= 1-2Sin2A 4. 2SinASinB=Cos(A-B)-Cos(A+B)

2 TanA
3. Tan2 A = IX. Sum and Difference of Sine and Cosine Identities
1 − Tan2 A
1. SinA+SinB=2Sin ½ (A+B)Cos ½ (A-B)

VI. Complementary Angle Identities


2. SinA-SinB=2Cos ½ (A+B)Sin ½ (A-B)
SinA = Cos(90-A)

3. CosA+CosB=2Cos ½ (A+B)Cos ½ (A-B)


CosA = Sin (90-A)

4. CosA-CosB=2Sin ½ (A+B)Sin ½ (A-B)


TanA = Cot(90-A)

X. Sine Law
CotA = Tan(90-A)
a b c
1. = =
sina sinb sinc
SecA = Csc(90-A)

CscA = Sec(90-A)
2. A+B+C = 180

VII. Half Angle Identities


XI. Cosine Law
1 − cosA
1. Sin A / 2 =
2
2
1. a2 = b2+c2-2bcCosA

1 + cosA
2. Cos 2 A / 2 = 2. b2 = a2+c2-acCosB
2

3. c2 = a2+b2-2abCosC
1 − cosA
3. Tan 2 A / 2 =
1 + cosA
4. A+B+C = 180

VIII. Product of Sine and Cosine Indentities


XII. Tangent Law
1. 2SinACosB=Sin(A+B)+Sin(A-B)
π/b – for tangent and cotangent
a − b Tan1/ 2(a − b) – distance of one complete wave of the function
1. =
a + b Tan1/ 2(a + b) 3) Phase Shift  /b c
– property true for any functions
– distance the graph is shifted to the right or to the left from its
a − c Tan1/ 2(a − c) standard position
2. =
a + c Tan1/ 2(a + c) – if c < 0, the graph is shifted to the right
– if c > 0, the graph is shifted to the left
b − c Tan1/ 2(b − c) 4) Vertical translation d – true for all functions
3. =
b + c Tan1/ 2(b + c) – defined as the distance the graph is shifted upward or
downward
4. A+B+C = 180 – if d > 0, the graph is shifted upward
– if d < 0, the graph is shifted downward
5.
Properties of Graph of Circular Functions
Sample Problem:
Determine the maximum value and the period of the function f(x) = –3 sin( π/4 x + 5) - 7
Consider the function
Solution:
a = 3
y = a sin (bx + c) + d
d = –7
a The graph has an amplitude of 3, but it is shifted downward by 7 units. Therefore the
maximum value of the function is 3 – 7 = –4.
Period = 2π/(π/4) ; since b = π/4
d
Period = 8 Answer

c
Supplementary Problems
/b
2π 1. If tan y = 1/3 and tan m = ½, y and m being acute, find y + m.
/b
2. If tan A = ¾ and Sin B = 12/13, A being greater than 180° and B is obtuse , find
a) Sin (A + B)
1) amplitude a – highest value of the function in standard form, a property is true
b) Cos (A + B)
for sine, cosine functions only
c) Tan (A + B)
2) Period 2π/b – for sine, cosine, secant and cosecant functions.
3. Find the value of Cos (A + B) if tan A = ¾ and csc B = 13/15, both angles are acute. Ans: P2 + Q2 = A2 + B2
4. If tan x + tan y = √3 and csc y = √2, x and y are acute, find x.
1 – tan x tan y\ 19. (ECE Board Exam April 6, 1991)
5. Tan x = 3, x is acute. Find Simplify cos A + cosB + sin A + sin B Ans: 0
a) sin 2x Sin A - sin B cos A - cos B

b) tan (x + 45°)
6. If tan (x + y) = 2 and tan x = 1, find tan y. 20. (ECE Board Exam April 6, 1991)

7. Given the acute angles A and B, sin A = 3/5 and sin B = 12/13, find the value of Simplify 2sin B cos B – cos B Ans: 2 sin B – 1

sin (A + B) + cos (A + B). 1 – sin B + sin2 B –cos2 B 1 – sin B

8. Find the value of tan (A + 2B) if cot A = tan B = 2. 21. (ECE Board Exam April 6, 1991)

9. Find sin 2A if tan A = -5/12 and cos A is negative. Simplify sin2 150 + sin2 750 Ans:1

10. Evaluate sin x(cot x/2 + tan x/2).


22. (ECE Board Exam April 6, 1991)
11. Given A, an angle between 0° and 360° such that sin A = - 4/5 and whose tangent is
Simplify sin 00 + sin 10 + sin 20 + . . . + sin 900 Ans: 1
positive, construct A and find the value of sin 2A – cot ½ A.
0 0 0 0
Cos 0 + cos 1 + cos 2 + . . . + cos 90
12. Find the value of cos 2x – sin (90° + x) if tan x = -3/4 and sin x is positive.
23. (ECE Board Exam April 16, 1991)
Find all values of x less than 360° satisfying the equations below.
Simplify: sin20o + sin21o + sin22o + . . . sin290o Ans: 45.5
13. 5sin 2x – 25cos x = 10 – 4sin x
14. cot 2x – tan x = 1
Angles of Elevation and Depression
15. sin 2x + sin x = 0
Angle of elevation – is the angle made with the horizontal by the line of sight from an
16. (ECE Board Exam Nov. 4, 1995)
observer to an object on the higher level than the observer.
1 – cos x sin x
Object
–––––––– + –––––––– Ans: 2cscx
Line of sight
sin x 1 – cos x

17. ECE Board Exam April 3, 1993


Angle of elevation
Solved for θ in the equation: sin 2θ = cos θ Ans: 300 and 1500
18. ECE Board Exam April 3, 199 Observer Horizontal Line
Given the relations P = Asin θ + Bcos θ and
Q = Acos θ - Bsin θ, derive another equation showing the relationship
between P, Q, A and B not involving any of the trigonometric functions of angle θ
Angle of depression – the angle made with the horizontal by the line of sight from an angle of elevation of the top of the lighthouse is 24º. Find the height of the lighthouse.
observer to an object of lower level that the observer. Ans. 95.181ft

Horizontal Line 6. If in right triangle eight times the product of the legs equals the square of the hypotenuse,
Observer find the angles of the triangle.
Angle of depression Ans. [ArcTan (4 + 2√15)], [ArcTan ( 2/4 + 2√15)], 90º

7. Two ladders, one of which is twice as long as the other, rest on the floor and reach the
Line of sight same vertical height on the wall. The shorter ladder makes an angle of 60º with the floor.
What angle does the longer ladder make with the floor? Ans. 25.66º

Object 8. From a point midway between two objects, one being three times as tall as the other, the
Supplementary Problems: angle of elevation of the taller is twice the angle elevation of the shorter. Find this angle
1. Two buildings 450 ft. and 600 ft. in height are opposite each other. From the roof of the of elevation. Ans. . 30° , 60°
higher building, the angle of depression of the edge of the roof of the lower building is
38º40’. How wide is the street? Ans. 187.45ft
9. A man standing at a certain point on a level filed determines the angle of elevation of the
top of a tower 50 feet high, he then finds that by going 90 feet nearer the tower, the angle
2. A tower and a monument stand on a level ground. The angles of depression of the top of
of elevation is increased by 45º. At the first observation how far was he from the tower,
the monument viewed from the top of the tower are 13º and 31º, respectively; the height
and what was the angle of elevation. Ans. 108.443ft., 27.753°
of the tower is 145 m. Find the height of the monument. Ans. 89.3

10. A garage is 12 feet high and fixed on its top is a flagpole 15 feet high. On the opposite
3. A flagpole 25m high stands on the top of a tower, which is 10.5 m high. At what distance
side of the street from the garage at a given point, the garage and the flagpole subtend
from the base of the tower will the flagpole subtend an angle of 3º20’.
equal angles. How wide is the street? Ans. 36ft.
Ans. 34.59 m

11. The angles of a triangle are in the ratio 3:4:5. Express the ratio of the sine of the smallest
4. The angle of depression of a barge from the top of a lighthouse is 16º. After the barge
angle. Ans. √2/2
has traveled 100 m toward the lighthouse, the angle of depression of the barge is 20º.
Find the height of the lighthouse. Ans. 135.147
12. Two parallel chords of a circle of radius 8 inches are on the same side of the center and 5
inches apart. One subtends a central angle twice as large as the other. Find the length of
5. From a boat directly south of a lighthouse, the angle of elevation of the top of the
the shorter chord.
lighthouse is 32º. From a second boat lying directly east of the first and 150 ft. from it, the
13. A man standing at a certain point on a level field determines the angles of elevation of the Two General Forms of Oblique Triangle
top of a tower 50 feet high. He then finds that by going 2/3 of the distance to the base of
the tower, toward it, the angle of elevation has been doubled. At the first observation, A triangle has three angles and three sides. If we are given the measures of three out of
how far was he from the tower, and what was the angle of elevation? this six principal parts, at least one of which is a side, we can possibly find the measures of
the other three parts.
14. A and C, the bases of two towers AB and CD standing on a horizontal plane, are 120 feet
apart. The angle of elevation of D as observed from A is double the angle of elevation of Case I. Given two angles and one side.
B as observed from C. From a point midway between A and C the angles of elevation of Use Law of Sines: B
B and D are complementary. Find the height of the towers.
Ans. AB = 40 ft., CD = 90 ft. a = b = c c a
sin A sin B sin C
15. (ECE Board Exam Nov. 4, 1995)
The angle which the line of sight to the object makes with the horizontal is above the A b C
eye of the observer. Ans. Angle of Elevation
Case II. Given two sides and an included angle.
16. (ECE Board Exam Nov. 4, 1995) Use Law of Cosines:
The angle which the line of sight to object makes with the horizontal is below the eye of
an observer. Ans. Angle of Depressio a2 = b2 + c2 – 2bc cos A
b2 = a2 + c2 – 2ac cos B
Solutions of Oblique Triangles c2 = a2 + b2 – 2ab cos C

C C Case III. Given two sides and the angle opposite one of them.
This is known as the ambiguous case, you can use sine law and examine the possibility
b a b of no solution, one solution or two solutions.
a
Case IV. Given three sides.
A c B A c B
Use cosine law, to solve for each angle.
A = cos-1 b2 + c 2 – a 2 = 45.178 ft. Answer
2bc
2. The sides of a triangle are 3, 5 and 7. Find the largest angle.
Solution :
-1 2 2 2
B = cos c +a –b Using Cosine Law
2ac a2 = b2 + c2 – 2bcCosA
here, a = longest side = 7 ; b = 3; c = 5
C = cos -1 2
a +b –c 2 2
A = largest angle → unknown
2ab Then, b2 + c2 – a2 = Cos A
2bc
Cos A = 32 + 52 – 72 = -0.5
Sample Problem.
2(3)(5)
1. Two angles of a triangle are 52°15′ and 59°30′, and the shortest side is 38.46 ft. long. A = Cos-1(-0.5)
Find the length of the largest side. A = 120° Answer.
Solution :
Using Sine Law (given 2 angles and any side)
Note : • The shortest side is opposite of the smallest angle 38.46 ft. is opposite of Supplementary Problems

52°15′. 1. In an oblique triangle ABC, it is known that tan A = ¾, cos B = 5/13 and AB = 10. Find
• The longest unknown side must be opposite of the largest angle. a) sin C

Largest angle is the third angle : b) side AC

180 – (52°15′ + 59°30′) = 68.25° c) side BC

Thus :
2. A chord, AC, of a certain circle equals 13″, and the angle B of the inscribed triangle ABC
a = b__
Sin A Sin B is 49.35°. Find the radius of the circle.

a = longest side, A = 68.25


b = shortest side = 38.46 ft. B = 52°15′ 3. An observer in a balloon 1 mile high observes the angle of depression of an object on the

Now, a = b SinA ground to be 35°40′. After ascending vertically at uniform rate for 10 minutes, he finds the
SinB angle of depression of the same object is to be 55°20′. Find the rate of ascent of the
o
= (38.46)(Sin 68.25 ) balloon in miles per hour.
Sin52°15’
4. From the top of a lighthouse 85 ft. high standing on a rock, the angle of depression of a 12. The area of a triangle is 50 sq. in. and the lengths of two of its sides are 20 in. and 10 in.
ship was 7°38′ and from the bottom of the lighthouse the angle of depression was 3°. Find the included angle.
What was the height of the rock?

5. A tower 51.63′ high makes an angle of 113°12′ with the inclined plane on which it stands. 13. Two automobiles leave the same town at the same time. One goes north at the rate of 30

The angle subtended by the tower at the same point down the plane from its base is miles per hour, the other goes in the direction 47° E of N at the rate of 20 miles per hour.

23°27′. How far is this point from the base? How far apart are they at the end of 5 hours?
14. The acute angle between the diagonals of a parallelogram is 45°. The diagonals are 8
and 12 inches. Find the lengths of the sides of the parallelogram.

6. A house is situated on a hillside which is inclined 29°39′ to the horizontal plane. A ladder 15. The sides of a triangle are 9, 12, and 14 and the largest angle is bisected, find the length
of the bisector of this angle and the length of the median to the shortest side.
32.75′ long just reaches the bottom of a window 25′ from the ground. What is the
distance of the foot of the ladder from the house, measured down the slope?
Inverse Trigonometric Functions
1. Inverse Sine Function
7. A man standing at a point due to west of a tower 150′ high found the angle of elevation of
y = Arcsin x if and only if x = sin y
the top of the tower to be 68°. He then walked to a second point southwest of the first and
found the angle of elevation of the top of the tower to be 39°. Determine how far he
y y
walked.
π
1 /2
8. A circle is inscribed in a ∆ ABC whose sides are a = 25″, b = 33″ and c = 38″. How long
are tangents to the circle from vertex A? vertex B? vertex C?

–1
9. A circle is inscribed in a triangle whose sides are 5, 12 and 13. Find the radius of the x x
circle.
- π/ 2 π
/2 1

10. The lower base of an isosceles trapezoid is 70.23″. Each of the nonparallel sides is
35.18″. each lower base angle is 81°30′. How long is each diagonal? -1 -π/2

11. If the diameter of a circle is 32.68″, find the angle at the center determined by a chord y = sin x y = Arcsin x
12.9″ long.
2. Inverse Cosine Function
y = Arccos x if and only if x = cos y y = tan x y = Arctan x
y y
where, ∞ ≤ x ≤ ∞, -π/2 < y < π/2 → principal value
1 π

Inverse Identities
π
/2 π x π
/2
1. Arcsin (sin θ) = θ for -π/2 ≤ θ ≤ π/2
2. sin (Arcsin x) = x for –1 ≤ x ≤ 1
3. Arccos (cos θ) = θ for 0 ≤ θ ≤ π
–1 –1 1 x 4. cos (Arccos x) = x for –1 ≤ x ≤ 1
5. Arctan (tan θ) = θ for -π/2 ≤ x ≤ π/2
y = sin x y = Arccos x
6. tan (Arctan x) = x for all values of x
where, –1 ≤ x ≤ 1, 0 ≤ y ≤ π → principal value

Supplementary problems
3. Inverse Tangent Function Solve the following trigonometric equations:
Y = Arctan x if and only if x = tan y
1. 3sinA = 2cos2A , 00 ≤ A ≤ 3600 ans: 300, 1500
1
y y
2. sin2x + 2sin2 2 x = 1 , 0 < x < 1800 ans:300,900, 1500
Π
1 π
/2 3 .Tan -12x +Tan-13x = ans: 1 / 6
4
4. Tan-1[2sin(Cos-1x)] = 600 ans: 1 / 2
0
x 0 x
5. Arctanx + Arctan2x = Arctan3 ans: -1, 1 / 2
-1 1
-1
6. Arcsin(1 – x) + Arccosx = 900 ans: 1 / 2
π
- /2

Solve the following problems


1. If sinA = 0.80, cosA > 0, find cotA. Ans. 0.75
12 13. From the top of the tower 33 m high, the angles of depression of the top and bottom
2. If sin θ = and cos θ < 0, find tan θ . Ans. -12/5
13 of another tower standing on the same horizontal plane are found to be 28.93 and 53.680
0

3. If A – B = 450 and tanA = ¾ , find tanB. Ans. -1/7 respectively. Find the distance between the tops of the towers. Ans. 27.7m

4. If cot θ = -24 / 7, θ in Quadrant II, find sec θ . Ans. -25/24 14. A surveyor at a certain distance measured the angle of elevation of a cliff. He then
walked 20 m on a level ground toward the cliff. The angle of elevation from this second
5. If cosx = 1 / 3, find cos4x. Ans. 17/81 station was then the complement of the former angle. The surveyor again walked 5 m
1 nearer to the cliff in the same line and found the angle of elevation from the third station
6. If cos θ = -1 / 3 and 00 < θ < 3600, find tan θ . Ans. Sqrt(2)
2 to be double the first angle. How high is the cliff? Ans. 22.91m
If sinA = 3 / 5 and A is in Quadrant II, find cos2A. ans. 7/25
15. The minute hand of a clock is 23 cm long while the hour hand is 15 cm long. The plane of
If tanA = 3 / 4 , tanB = -15 / 8, A in Q III, B in Q IV, find rotation of the minute hand is 5 cm above the plane of rotation of the hour hand. Find the
cos(A – B). Ans. 13/85 distance between the tips of the hands of the clock at 2:30 p.m. Ans. 30.9cm

9. Two trains start at the same time from the same station and upon straight tracks 16. A point P within an isosceles right triangle is at a distance of 6, 5 and 4 cm from the
diverging at an angle of 68 degrees. If one train runs at 32 kph and the other at 46 kph, vertices A, B and C respectively. Find the length of the hypotenuse of the right triangle if
how far apart are they at the end of 3 hours? Ans. 135.4km the right angle is at C. Ans. 10.65cm

10. The bearing of B from A is N20 0E; the bearing of C from B is S30 0E; and the bearing of A 17. The angle of elevation of the top of a pole at a point 30 m from the pole is three times the
from C is S400W. If AB = 10 m, find the area of the triangle formed by A, B and C. Ans. angle of elevation of the top of the same pole at a point 150 m from the pole. Find the
13.94sq.m. height of the pole. Ans. 56.7cm

11. An airplane flying an altitude of one km directly away from a stationary observer on the 18. A corner lot of land is 35 m on one street and 25 m on the other street, the angle between
ground, has an angle of elevation of 48 degrees at a certain instant and an angle of two lines of the streets being 80 degrees. The other two lines of the lot are respectively
elevation of 20 degrees one minute later. Find the speed of the plane. Ans. 110.82kph perpendicular to the lines of the streets. What is the worth of the lot at P1,000 per square
meter?
12. From a point on a level ground, the angle of elevation of the top of a building is observed ans: P725,475.00
to be twice the angle of elevation of a window one third of the way up the building. Find
the angle of elevation of the top of the building. Ans. 60degrees 19. At noon, a ship A is sailing on a course eastward at the rate of 20 kph. At the same
instant, another ship B, 100 km east of ship A is sailing on a course N30 0W at the rate of
10 kph. How far away from each other are the ships after one hour? Ans 75.5km
Note: Null set is a subset of all sets
20. A pole casts a shadow 15 cm long when the angle of elevation of the sun is 61 degrees.
If the pole leans 15 degrees from the vertical toward the sun, what is the length of the  Unit Set – a set with only one element
pole? Ans. 54.23cm  Infinite Set – if it is impossible to list down all elements of a set
21. Find the interior angles of a triangle whose sides are 21, 28 and 17.  Finite Set – if it is possible to write all of its elements
ans: 48.400, 94.340, 37.260 • Cardinal Number – refers to the number of elements of a certain set
22. The sides of a triangle ABC are a = 50 cm, b = 64 cm and c = 20 cm. Find the length of
– denoted by n(A) → no. of elements of set A
the median drawn from B to AC. Ans. 20.64cm

Operations on Sets
23. Two stations B and C are situated on a horizontal plane 366 m apart. A balloon is directly
• Union of Sets
above a point A in the same horizontal plane as B and C. At B, the angle of elevation of
The union of sets A and B is another set denoted by A∪B whose elements
the balloon is 62 degrees and the angle at B subtended by AC is 53 degrees and at C,
are in A, or in B or both A and B.
the angle subtended by AB is 72 degrees. Find the height of the balloon. Ans. 799.19m
In symbol, A⋃B = {x x ∈ A or x ∈ B}

PROBABILITY • Intersection of Sets

Introduction to Set Theory The intersection of sets A and B is another set denoted by A ⋂B whose
Definitions elements are common to both A and B.
• Set – a collection of objects called elements of any sort with restriction to In symbol, A⋂B = { x x ∈ A and x ∈ B }

those objects clearly described. (Denoted by capital letters) • Relative Complement


• Element – any object that belong to a set (Denoted by small letters) The relative complement of the set B with respect to A is another set denoted
∈ - symbol used to indicate that an element belong to a given set. by A – B whose elements are all A but not in B.

• Subset – If all elements of set A are in set B or if there are elements of set B not In symbol, A – B = { x x ∈ A and x ∉ B }

belonging to set A, then A is a subset of B denoted by A ⊂ B. • Absolute Complement

• Equal Sets – Two sets are equal if they have exactly the same elements. The absolute complement of set A denoted by A′ is another set whose

A=B elements do not belong to A but in the universal set U

• Equivalent Set – Two sets are equivalent if they have equal number of In symbol A′ = { x x ∈ u and x ∉ A }

elements. A ⇔ B • Product Set or Cartesian Product


The product set denoted by A x B is a set of ordered pairs (x, y) such that x is
• Size of Set
an element of set A and y is an element of set B.
 Null Set or Empty Set – a set without an element
In symbol, A x B = { (x, y) x ∈ A and y ∈ B}
denoted by { } or φ
Note: A x B ≠ B x A n(B⋃S) = 593 total Answer

• Disjoint Set
Two sets A and b are considered disjoint sets if they have no element in Fundamental Counting Principle

common. If an element E, can happen in n1 ways, and for each of these another event E 2 can
happen in n2 ways, and for each of these events, another event E 3 can happen in n3 ways,
In symbol, A⋂B = { } or A⋂B = φ
and so on and so forth, then all in all the events can happen simultaneously in N ways equal
to:
N = n1 n2 n3 …nk no. of ways
Venn Diagram – Pictorial representation of set relations and operations
– Introduced by John Venn, an English Logician
PERMUTATION / COMBINATION
Factorial Notation
Number of elements in a union of Sets: COUNTING FORMULA
- the factorial n, where n is any positive integer is denoted by n!
a.) n(A⋃B) = n(A) + n(B) – n(A⋂B)
Factorial n is defined as the product of positive consecutive integers from 1 to n
b.) n(A⋃B⋃C) = n(A) + n(B) + n(C) – n(A⋂B) – n(A⋂C) – n(B⋂C) + n(A⋂B⋂C)
inclusive
That is,
Sample Problems:
n! = 1(2)(3) …(n-3)(n-2)(n-1)n
1. In a survey, 458 men like basketball, 385 like softball, 250 like both. How many men were
or n! = n(n-1)(n-2) …3.2.1
there? How many like softball only.
Observe that n! = n(n-1)!
Solution:
= n(n-1)(n-2)!
In using Venn Diagram, always prioritize to indicate the number of elements
By definition 0! = 1
in the intersections.

PERMUTATION
u Let n(B) = numbers who like basketball
- is arrangement of all or part of a set of objects in a definite order
n(S) = number of men who like softball
208 250 135 - is any linear ordering of the elements of a set
n(B⋂S)=number of men who like
B S basketball and softball
CASES OF PERMUTATION
n(B⋃S) = total number of men in the survey
CASE 1: Permutation of n different things taken r at a time
Now,
- defined as an arrangement of r out the n objects with attention given to the order of
n(B) – n(B⋂S) = 458 – 250 = 208 like basketball only
arrangement
n(S) - n(B⋂S) = 385 – 250 = 135 like softball only Answer
nPr = n! r<n
n(B⋃S) = n(B) + n(S) - n(B∩S)
(n-r)!
= 458 + 385 –250
CASE 2: Permutation of n different things taken all at a time that is nCa = nCb , if a + b = n
n Pr case whre r = n
n Pr = nPn = n! = n! = n! 2. Number of combination of n things taking 1 or 2, … or n at a time
(n-n)! 0! n C1 + nC2 + nC3 + … + nCn = 2n – 1
CASE 3: Permutation of n things not all different
Supplementary Problems
Let N = number or permutations on n things taken all at a time of which p are alike, q
others are alike, r others are alike, and so on… 1. In how many ways can 7 boys and 6 girls sit in a row if the boys and girls must alternate.
N= n! where p + q + r + …= n Ans. 3, 628, 800
p!q!r!…
CASE 4: Number of Permutation of n distinct objects arranged in a circle 2. In how many ways can 6 people be lined up to get on a car?
N = (n-1)! b.) If certain 3 persons insist on following each other
c.) If certain 2 persons refuse to follow each other how many ways are possible
COMBINATION Ans. a.) 720 b.) 144 c.) 480
- selection of group of objects out of a set irrespective of their order
- combination of n things taken r at a time refers to filling out r place when we have n things at 3. From the digits 0, 1, 2, 3, 4, 5, 6, 7, how many 3 digit number can be formed
our disposal denoted by nCr b.) how many numbers are odd if digits are distinct

n Cr = n!__ c.) how many numbers are even if digits are distinct
(n-r)! r! Ans. a.) 294 b.) 144 c.) 150
Special Cases: 4. In a multiple choice test of 6 questions with 4 possible answers of which only one is
1. When r = n correct

n Cn = n! = n! =1 a.) in how many different ways can a student check off one answer to each
(n-n)! n! 0! n! question?

n Cn = 1 (combination of n things taking all at a time) b.) In how many different ways can a student check off one number to each question
and get all answers wrong.
2. When r = 1 Ans. a.) 4, 096 b.) 729

n C1 = n! = n(n-1)! =n
(n-1)! 1! (n-1)! 5. How many triangles can be formed using the vertices of a dodecagon? Ans. 220

n C1 = n (combination of n things taking one element at a time)


6. In how many ways can you invite one or more of your 5 friends in a certain show?Ans.
31
Other important Combination Formulas
1. nCr = nCn-r
7. How many different three-digit numbers can be made with three 4’s, four 2’s and two 3’s? 15. Solve for n in the equation nP4 = 30[nC5]. Ans. n = 8
Ans. 26

8. A department plays 12 softball games during a season. In how many ways can the team PROBABILITY – measure of the likehood of occurrence of an event
end the season with 7 wins, 3 losses and 2 ties? Ans. 7920
Classical Definition
If an event E can happen in h ways (called the success) and fail to happen in f ways
9. How many distinct permutations are there of the letters of the word “CIRCUIT”.Ans. 1,260
(called failure) out of n possible equally likely ways, then the probability of occurrence is
denoted by
10. How many numbers between 3,000 and 5,000 can be formed with the digits 7, 3, 4, 8, 2
p(E) = h = favorable ways
and 5, no repetitions being allowed. Ans. 48
h+f possible ways

11. In how many ways can 8 boys and 5 girls be arranged in a circle if
The probability q that an event will fail to happen is given by
a.) certain 2 boys will not be together
q(E) = f probability of failure
b.) a certain boy and girl won’t be together
h+f
c.) certain 2 girls won’t be together
d.) certain 3 girls will be together Odd Ratios
Ans. a, b, c, = 399, 168, 000, d = 21, 772, 800 1. Odds in favor
p=h
12. There are 12 ECE’s in a certain telephone company so that 3 will be assigned to q f
switching, 2 will be assigned in planning, 4 will be assigned to transmission and the
remaining will be assigned in some other functions. In how many ways the engineer may 2. Odds against
be divided? q=f
Ans. 277, 200 ways p h
Note: p + q = 1
13. A club has 25 members 4 of whom are engineers. In how many ways can a committee of
3 members be formed in order that at least one member is an engineer? Ans. 970 Axioms of Probability
1. The probability of occurrence of a particular event is a positive real number from zero to
14. From a group of 6 women and 8 men, a committee of 5 is to be formed. In how many one 0≤ p ≤ 1
ways can this be done. If each committee id to consist of a.) exactly 3 men b.) at least 4 2. The probability of occurrence of any events in a sample space is equal to one
men c.) at most 2 men. Ans. a.) 840 b.) 476 c.) 686 The probability of occurrence of certain event is equal to one P(S) = 1
3. The probability of impossible event is equal to zero P(φ) = 0
4. The sum of probability of occurrence of each event in the sample space is equal to one. • P(E1∪E2) = P(E1) + P(E2) for mutually exclusive events
ΣPi = 1 • P(E1∪E2) = P(E1) + P(E2) – P(E1∩E2) for not mutually exclusive
Note: P(E1∪E2) means probability that either E 1 occur only, E2 occur only or E1 and
EVENTS E2 occur both at the same time
• Independent Events • For n mutually exclusive events
Two or more events are said to be independent if the occurrence or non-occurrence P(E1∪E2∪E3∪ … ∪En) = P(E1) + P(E2) + P(E3) + … + P(En)
of any of them does not affect the probabilities of the occurrence of any of the others.
• Dependent Events
3. P(E’) = 1- P(A)
Two or more events are said to be dependent if the occurrence of one events affects Probability of the complement of an event, E’
the probabilities of the occurrence of any of the other.
• Mutually exclusive Events Probability in Repeated Trials – Bernoulli’s Experiment
Two or more events are said to be mutually exclusive if the occurrence of any one of
them excludes the occurrence of the other. P = nCr pr qn-r
(that is, not more than one of them can happen in a single trial)
Where: P= the probability that the event will occur exactly r times in trials
CONDITIONAL PROBABILITY p = the probability that the event will occur in a single trial
For two events E1 and E2, the probability that E2 occurs, given that E1 has occurred is q = the probability that the event will fail to happen (probability of failure)
denoted by =1–p
P(E1/E2) n Cr = the number of ways that the event will occur
For compound events E1E2,
P(E1E2) = P(E1) P(E2/E1) Multinomial Distribution
LAWS OF PROBABILITY If events E1, E2, E3, …, En can occur with probabilities P 1, P2, P3, …, Pn respectively, then
1. Multiplication Law (AND LAW) the probability that E1, E2, E3, …, En will occur x1, x2, x3, …,xn times respectively is given by:
Probability that events E1and E2 will happen both at the same time
• P(E1∩E2) = P(E1) P(E2) for independent events
• P(E1∩E2) = P(E1) P(E2/E1) for dependent events P= N! P1X1 P2X2 P3X3 … PnXn

So that, P(E2/E1) = P(E1∩E2) x1! x2! x3! …xn!


where x1+ x2+ x3+ … +xn = N
P(E1)

2. Addition Law (OR LAW)


Mathematical Expectation 7. An urn contains 3 red pens and a number of blue pens. A second urn contains 5 blue
If p is the probability of success in a single trial of an event, the expected number of pens and an unknown number of red pens. One pen is drawn from the first urn and
successes in n trials is given by : placed unseen in the second urn. Now, two pens are to be drawn form the second urn. If
E = nP the odds in favor of getting 2 blue pens equals 3:5 and the odds in favor of getting pens
of different colors equal 31:29, how many are blue pens in the first urn and how many are
Supplementary Problems red pens in the second urn?
1. In a family of 5 children, what is the probability that the first 3 are girls. Ans. 1/8 *Author’s Hint: No. of blue balls is 3 more than the no. of red balls

2. Four Electronics books and three Communications book are placed on a shelf. What is 8. A pair of dice is thrown. If it is known that one die shows a 4, what is the probability that
the probability that the Communications book will be together? Ans. 0.14286 a. the other dies shows a 5
b. the total of both dice is greater than 7
3. Determine the probability that 7 or 11 comes up in a single toss of a pair of fair dice. Ans. a.) 2/11 b.) 5/11
Ans. 2/9 9. Find the probability of getting 1, 2 or 3 in four tosses of a fair die. Ans. 1/16
4. Tirso and Louie work independently in troubleshooting a circuit. The probability that Tirso
can fix it is 0.6 while the probability that Louie can fix it also is 0.5. What is the probability 10. There are three shooters A,B,C. The probability that probabilities that A, B and C can hit
that the circuit will be fixed? Ans. 0.8 the target are 1/2, 1/3 and 1/4 respectively. If they are simultaneously shooting the target,
find the probability that two of them will hit the target. Ans. 0.25
5. During a certain war, an offensive troop released 4 bombs each of which has probability
of hitting the enemy of 0.25. If 4 bombs are released simultaneously, what is the 11. The probability that Alvin can win in a chess game whenever he plays is 20%. If he plays
probability of hitting the enemy? Ans. 0.685 5 games, find his probability of losing exactly 2 games. Ans. 32/625

6. A box contains an assortment of red, blue and yellow balls. If three balls are drawn from it 12. An urn contains 5 white and 6 green balls. If 4 balls are drawn together, find the
at random, the probability that 2 red balls and 1 blue ball are drawn is 30% of the probability that all are of the same color. Ans. 2/33
probability that 2 blue balls and 1 yellow ball are drawn. Furthermore, the probability that
3 blue balls are drawn is 5/48 times the probability that one ball of each color is drawn. 13. Out of 800 families with 5 children each, how many would you expect to have a.) 3 boys
However, if only two balls are drawn, the probability that one is blue and the other is a b.) 5 girls and c.) either 2 or 3 boys? Ans. a.) 250 b.)25 c.) 500
yellow ball is 8 times the probability that both are red. How many are red balls, blue balls,
and yellow balls are there in the box. 14. A box contains a very large number of red, white, blue and yellow marbles in the ratio
*Author’s Hint: The number of balls are in arithmetic progression. 4:3:2:1 respectively. Find the probability in 10 drawings.
a.) 4 red, 3 white, 2 blue and 1 yellow marble will be drawn and
b.) 8 red and 2 yellow marbles will be drawn Ans. a.) 0.0348 b.) 0.000295
22. A contractor wishes to build five houses, each different in design. In how many ways can
16. a) How many ways can be five people be lined up to pay their electric bills? he place these homes on a street if two lots are on one side and three lots on the
b) If two particular persons refuse to follow each other, how many ways are possible? opposite side?
Ans. a) 120 ways b) 72 ways Ans. 120 ways

17. An engineering freshman must take a chemistry course, a humanities course, and a 22. In how many ways can four boys and three girls sit in a row if the boys and girls must
mathematics course. If they may be select any of 2 chemistry courses, how many ways can alternately be seated?
he arranged his program? Ans. 24 ways Ans. 288 ways

18. In how many different ways can a ten- question true- false examination be answered? 23. In how many ways can seven trees be planted in a circle? Ans. 720 ways
Ans. 1024 ways 24. In how many ways can two mango trees, three chico trees, and two avocado trees be
arranged in a straight line if one does no distinguish between trees of the same kind?
19. How many distinct permutations can be made from the letters of the word “mathematics”? Ans. 210 ways
Ans. 4,989,600
25. A college team plays eight basketball games during an intramural. In how many ways can
20. How many ways can the first five players in a basketball team be filled with twelve men the team end and games with four win, three losses and one tie? Ans. 280 ways
who can play any of the positions?
Ans. 95,040 ways 26. Nine people will be shooting the rapids of Pagsanjan in three bancas that will hold 2, 4
and 5 passengers, respectively. How many ways is it possible to transport the nine
21. How many three- digit numbers can be formed from the digits 0,1,2,3,4 and 5 people to the falls?
a) if each digit is used only once in a given number? Ans. 4,536 ways
b) if digits may be repeated in a given number
27. From a group of three men and seven women, how many committees of five people are
c) How many in (a) are odd numbers?
possible?
d) How many in (a) are even numbers?
a) with no restrictions?
e) How many in (b) are even numbers?
b) with two men and three women?
f) How many are less than 330?
c) with one man and four women if a certain woman must be on the committee?
g) How many are greater than 330?
Ans. a) 252 b) 105 c)60
Ans. a) 100 b)180 c)48 d)52 e)90 f)90 g)89

28. From three red, four green, and five yellow bubblegums, how many selections consisting
of five bubblegums are possibleif two of each color are to be selected? Ans. 390
29. A shipment of 10 Sony Betamax video recorders contains 3 defective sets. In how many 37. Joey prepares 3 cards for his 3 girlfriends. He addresses 3 corresponding envelopes. A
ways can a hotel purchase 4 of these sets and receive at least 2 of the defective sets? brown-out suddenly occurred and he hurriedly placed the cards in the envelope at
Ans. 70 random. What is the probability that
(a) each card is send to its proper addressee?
30. A bag contains four blue, five red, and six yellow plastic chips. (b) no card is sent to the proper address? Ans. a) 5/16 b)9/6
a) If two chips are drawn, find the probability that both are yellow
b) If six chips are drawn, find the probability that there will be two chips of each color 38. A box contains 15 red eggs and 20 white eggs. If 12 eggs are taken on random, what is
c) If nine chips are drawn, find the probability that two will be red, five yellow and two blue the probability that these will have an equal number of red and white eggs? Ans. 7/12
Ans. a) 1/7 b)180/1,001 c) 72/1,001
39. During a fund raising lottery, 250 tickets will sold to the freshman, of which 3 are winners.
31. In a single throw of two dice, what is the probability of throwing not more than five? Marissa, a freshman has 2 tickets. What is her probability of winning something? Ans.
Ans. 5/18 248/ 10375

32. Find the probability that all five cards drawn from a deck are all hearts? Ans. 4.95 X 10 -4 40. If the probability that Nini will go to UP for a certain seminar is 1/3 and the probability that
she will go to UST that seminar is 1/4, find the probability that she will go to collage in
33. a team of 5 students is to be chosen for a math contest. If there were ten male and eight one of the two schools.
female students to choose from, what is the probability that three team members will be Ans. 7/12
male and two will be female? Ans. 20/51
41. If the probability that Ginebra, Alaska, and Shell will win the PBA open conference
34. A bag contains five pairs of socks. If four socks are chosen, what is the probability that championship are 1/5, 1/6, and 1/10, respectively, find the probability that one of theme
there is no complete pair taken? Ans. 8/21 will win the title. Ans. 7/15

35. In the game “spin-a-win” the rim of a wheel is divided in to 30 equal parts with each 42. The probability that Joseph Estrada will be nominated to run fir president is ¼ , and the
marked P10, P20,...,P300. The “win” is indicated by a fixed pointer at the top. If the wheel probability of his election if nominated is 1/3. Find the probability (a) of his being elected
is spun, what is the probability that a three-digit number will be the player’s take home president, (b) of his being nominated and not being elected. Ans. a) 1/12 b)1/6
winning? Ans. 7/10
43. Find the probability of obtaining a 4 in each of two successive tosses of a pair of dice.
36. If eight different books are arranged at random in a shelf, what is the probability that a Ans. 1/1296
certain pair of books (a) will be beside each other? (b)will not be together? Ans. a) ¼
b) ¾ 44. One box contains five black and three white handkerchiefs and another seven black and
five white handkerchiefs. If one handkerchief is drawn from each box, find the probability
that both will be (a) black, (b) white, (c) the same color. Ans. a) 35/96 b) 5/32
c)25/48 52. the probability of an event happening exactly twice in four trials is 18 times the probability
of it happening exactly five times in six trials, find the probability that it will happen in one
45. The probabilities that Marita will win the preliminary, semifinal, and final contest in singing, trial. Ans. 1/3
are 3/8, 1/6, and ½, respectively. Failure in any contest prohibits participation in the
following one. Find the probability that she will (a) reach the final contest (b) win the final 53. If the probability that an event will happen exactly three times in five trials is equal to the
contest. Ans. a) 1/16 b) 1/192 probability that it will happen exactly two times in six trials, find the probability that it will
happen in one trial. Ans. 0.451
46. Three Physics books, five Algebra books, and two Chemistry books are on the shelf. Judd
decides to take two books and selects them at random. Find the probability that the first 54. How many permutations can be formed from the letters of the word “constitution”? Ans.
book drawn will be Physics and the second Chemistry. Ans. 1/15 9,979,000

47. Find the probability of throwing in three tosses of a die, (a) exactly two 4’s (b) at least two 55. How many four- place numbers can be written using the digits from 1 to 9? Ans. 3,024
4’s. 56. Find in if P(n,3)= 6 C(n,5) Ans. n=8
Ans. a) 5/.72 b) 2/27
57. Two dice are rolled. Find the probability that the sum of the two dice is greater than 10.
48. A bag contains three white, four red, and five green candies. Five withdrawals of one Ans. 1/12
candy each are made, and the candy is replaced after each. Find the probability that all
five will be red. Ans.1/243 58. A pair of dice is thrown. Find the probability of having a 7 or 11. Ans. 2/9

49. If the probability that Alaska basketball team will win the PBA Conference Championship 59. A pair of dice is thrown. If it is known that one die shows a 4, what is the probability that
is 2/3, find the probability that it will win exactly three championships in 5 years. the other die shows a 5? Ans. 2/11
Ans. 80/243
60. Nine tickets, numbered to 1 to 9, are in the box. If two tickets are drawn at random,
50. Six Algebra books, four Physics books, and two Chemistry books are on the table. If a determine the probability that both are odd. Ans. 5/18
book is removed and replaced, then another is removed and replaced, and so on until six
removals and replacements have been made, find the probability that an Algebra book 61. A committee of three is to be chosen from a group of 5 men and 4 women. If the
was removed and replaced (a) three times (b) at least three times. Ans. 16/243 selections is made at random, find the probability that two are men. Ans. 10/21

51. If the probability that Imelda will be elected to office is 2/3, find the probability that she will 62. Three balls are drawn from box containing 5 red, 8 black, and 4 white balls. Determine
be elected for successive terms and then defeated on the fifth term. Ans. 16/243 the probability that all are white. Ans. 1/170
63. A bag contains 9 balls numbered 1 to 9. Two balls are drawn at random. Find the
probability that one is even and the other is odd. Ans.5/9

64. How many cars can be given license plates having 5 digits numbers using the digits
1,2,3,4 & 5 with no digit repeated in any license plate? Ans. 120

65. A committee of 4 is selected by lot from a group of 6 men and 4 women. What will be the
probability that will consist of exactly 2 men and 2 women? Ans. 3/7

66. There are 52 tickets in lottery in which there is a first and a second prize. What is the
probability of a man drawing a prize if he owns 5 tickets? Ans. 0.18367

67. There are three candidates for A, B, and C for mayor of a certain town. If the odds are 7:5
that candidate A will win and those of B are 1:3, what is the probability that candidate C
will win? Ans. 1/6

68. A coin is biased so that the head is twice as likely to occur as tail. If the coin is tossed 3
times, what is the probability of getting
a) 2 tails and 1 head? Ans. 2/9 b) at least 2 heads? Ans. 20/27

69. Three man are running for public office. C candidates A & B are given about the same
chance of winning. But candidate C is given twice the chance of either A and B. Find the
probability that:
a) C wins b) A does not win
Ans. a) ¾ b)3/4

70. A player sinks 50% of all his shots. What is the probability that he will make exactly 3 of
his next 4 shots? Ans. 25%

You might also like